1
q
The nurse attends to a patient excluded/excluded from the telemetry station with a diagnosis of acute myocardial infarction. The patient's chest pain began 3 hours ago. Which of the following laboratory tests would be most helpful in confirming the current myocardial infarction diagnosis?
a) Creatine kinase myoglobin (CK-MB) levels
b) CK-MM
c) Troponin C levels
d) Myoglobinspiegel
A
a) Creatine kinase myoglobin (CK-MB) levels
Explanation:
An elevated CK-MB score by mass assay is indicative of acute myocardial infarction; levels begin to rise within a few hours and peak within 24 hours of a heart attack. If the area is reperfused (due to thrombotherapy or PCI), it will peak sooner. CK-MM (skeletal muscle) is not an indicator of heart muscle damage. There are three isomers of troponin: C, I, and T. Troponins I and T are specific to cardiac muscle, and these biomarkers are now recognized as reliable and critical markers of myocardial injury. An increase in myoglobin is not very specific to indicate an acute cardiac event; however, negative results are an excellent parameter to rule out an acute myocardial infarction.
2
q
Which of the following nursing actions should a nurse perform to reduce cardiac workload in a patient diagnosed with myocarditis?
a) Keep the patient on bed rest.
b) Elevate the patient's head.
c) Administer supplemental oxygen.
d) Administer a prescribed antipyretic.
A
a) Keep the patient on bed rest.
Explanation:
The nurse should keep the patient on bed rest to reduce cardiac workload and promote healing. Bed rest also helps reduce heart muscle damage and complications of myocarditis. The caregiver should administer supplemental oxygen to relieve tachycardia that may develop due to hypoxemia. If the patient has a fever, the nurse should administer a prescribed antipyretic along with autonomous nursing measures such as The caregiver should elevate the patient's head to stimulate maximum respiratory potential.
2
q
A nurse cares for a patient who has suffered a myocardial infarction. The patient is prescribed metoprolol (Lopressor). Does the nurse understand which of the following therapeutic effects this medication has?
a) Lowers cholesterol levels
b) Decreases platelet aggregation
c) Increases cardiac output
d) Decreases resting heart rate
A
d) Decreases resting heart rate
Explanation:
The therapeutic effect of beta-adrenergic blockers such as metoprolol is to reduce myocardial oxygen consumption by blocking beta-adrenergic sympathetic stimulation of the heart. The result is decreased heart rate, slower conduction of impulses through the conduction system, decreased blood pressure, and decreased myocardial contractility to balance myocardial oxygen demand and available oxygen. . This helps control chest pain and delays the onset of ischemia during work or exercise. This drug classification also reduces the incidence of recurrent angina, stroke, and cardiac mortality. In general, drug doses are adjusted to achieve a resting heart rate of 50 to 60 beats per minute. Metoprolol is not given to lower cholesterol, increase cardiac output, or decrease platelet aggregation.
3
q
A thoracentesis is performed to obtain a sample of pleural fluid or a biopsy sample of the pleural wall for diagnostic purposes. What does bloody fluid indicate?
a) Emphysema
b) Trauma
c) malignancy
d) infection
A
c) malignancy
Explanation:
A thoracentesis may be performed to obtain a sample of pleural fluid or to biopsy a sample of the pleural wall for diagnostic purposes. The fluid, which may be clear, serous, bloody, or purulent, provides clues to the pathology. Bloody fluid may indicate malignancy, while purulent fluid usually indicates infection. Pneumothorax, tension pneumothorax, subcutaneous emphysema, and pyogenic infection are complications of thoracentesis. Pulmonary edema or cardiac distress may occur after a sudden change in mediastinal content when large volumes of fluid are aspirated.
3
q
What type of oxygen therapy consists of administering oxygen at a pressure greater than 1 atmosphere?
a) Transtracheal
b) Low Flow System
c) High flow system
d) overpressure
A
d) overpressure
Explanation:
Hyperbaric oxygen therapy is the administration of oxygen at pressures greater than 1 atmosphere. As a result, the amount of dissolved oxygen in the plasma increases, which increases the oxygen content in the tissues. Low flow systems contribute in part to the inspiratory gas that the patient breathes, which means that the patient breathes part of the room air together with the oxygen. High flow systems are indicated for patients who need a constant and precise amount of oxygen. During transtracheal oxygenation, patients achieve adequate oxygenation at lower rates, making this method less expensive and more efficient.
3
q
In general, are chest tubes not indicated in a patient undergoing any of the following procedures?
a) Wedge resection
b) segmentectomy
c) Lobectomy
d) Pneumonectomy
A
d) Pneumonectomy
Explanation:
Drains are generally not used in patients who have had a pneumonectomy because fluid accumulation in an empty hemothorax prevents mediastinal displacement. During a lobectomy, two chest tubes are usually used for drainage, the upper one for air and the lower one for fluid. With wedge resection, the pleural cavity is usually emptied due to the possibility of air or blood leakage. Drains are generally used in segmentectomy due to the possibility of air or blood leakage.
3
q
A patient is scheduled for a cholecystectomy. Which nursing finding contributes least to surgical complications?
a) pregnancy
b) Osteoporosis
c) urinary tract infection
d) diabetes
A
b) Osteoporosis
Explanation:
Osteoporosis is unlikely to contribute to any complications associated with a cholecystectomy. Pregnancy reduces maternal reserves. Diabetes increases wound healing problems and the risk of infection. Urinary tract infections weaken the immune system and increase the chance of infection.
3
q
The nurse trains a patient scheduled for elective surgery. Currently, the patient is taking aspirin daily. What education should the nurse provide regarding medication?
a) Continue taking the aspirin as directed.
b) Stop taking aspirin 7 days before surgery, unless your doctor tells you otherwise.
c) Take half a dose of aspirin up to 1 week after surgery.
d) Aspirin should be increased up to 3 days before surgery and discontinued until 3 days after surgery.
A
b) Stop taking aspirin 7 days before surgery, unless your doctor tells you otherwise.
Explanation:
Aspirin should be stopped at least 7 to 10 days before surgery. The other instructions given are incorrect.
3
q
What statement about an institutional ethic
is the commission correct?
1. The ethics committee would be the first option
in overcoming an ethical dilemma.
2. The ethics committee replaces decision making
of patients and health professionals.
3. The ethics committee frees the health system
Specialists in ethical issues.
4. The ethics committee provides education, policy
Case Recommendations and Advice
A
4. The Ethics Committee provides education, policy recommendations, and advice on cases
The Ethics Committee is an additional resource
for patients and health professionals.
3
q
The client most at risk for postoperative wound infection is:
1. 3-month-old baby after pyloric stenosis correction surgery
2. A 78-year-old postoperative hernia.
3. 18-year-old adolescent, drug user, in the postoperative period for bullet extraction in the leg
4. A 32-year-old diabetic in the postoperative period of appendectomy
A
3. Adolescent, 18 years old, drug user, in the postoperative period for the extraction of a bullet in the leg; Everyone is at risk of infection. Answer 3 is higher risk because the bullet is impure and a drug user is at high risk of developing an immunodeficiency.
4
q
A nurse instructs a patient to be discharged after nasal packing surgery. Which of the following discharge instructions would be most appropriate for the patient?
a) Administer saline nasal drops as directed.
b) Reduce the amount of daily fluids.
c) Reduce the amount of daily fluids.
d) Avoid physical activity for 6 weeks.
A
d) Avoid physical activity for 6 weeks.
Explanation:
The nurse instructs the patient to refrain from physical activity for 6 weeks. There is no indication for the patient to refrain from taking oral fluids. Mouthwashes help to hydrate mucous membranes and reduce the odor and taste of dried blood in the oropharynx and nasopharynx. The patient should take pain relievers such as acetaminophen or NSAIDs (ie ibuprofen or naproxen) to relieve nasal discomfort, not aspirin. The patient does not need to use nasal drops if nasal packing is present.
4
q
A nurse cares for a patient in the intensive care unit who has been diagnosed with coronary artery disease (CAD). Which of the following assessment data indicates that the patient is experiencing decreased cardiac output?
a) Disorientation, 20 ml of urine in the last 2 hours
b) Blood pressure 108/60 mm Hg, ascites and crepitus
c) Increased dilation of the jugular vein (JVD) and postural changes in BP
d) Decreased pulse pressure and heart murmur
A
a) Disorientation, 20 ml of urine in the last 2 hours
Explanation:
Evaluation findings associated with reduced cardiac output include reduced pulse pressure, hypotension, tachycardia, reduced urination, lethargy, or disorientation.
4
q
A nurse attends a patient in the Post Anesthesia Care Unit (PACU) with the following vital signs, pulse 115, respiration 20, oral temperature 30.2°C, blood pressure 84/50. What should the caregiver do first?
a) Check the patient's preoperative vital signs.
b) Increase IV fluid rate.
c) Check for bleeding.
d) Notify the doctor.
A
c) Check for bleeding.
Explanation:
The patient is tachycardia with low blood pressure; Therefore, the bleeding test is the priority. Although it is necessary to notify the doctor, the nurse must be able to provide a complete picture of the patient, including bleeding, when calling the doctor. IV fluid administration rate should be adjusted as directed by a physician. The nurse should check previous vital signs, but only after determining an imminent threat of bleeding.
5
q
A patient comes to the clinic complaining of fever, chills, and sore throat and is diagnosed with strep throat. A nurse knows that early diagnosis and effective treatment are essential to prevent which of the following preventable diseases?
a) pericarditis
b) Mitralstenosis
c) rheumatic fever
d) Cardiomyopathy
A
c) rheumatic fever
Explanation:
Rheumatic fever is a preventable disease. Diagnosis and effective treatment of strep throat can prevent rheumatic fever and thus rheumatic heart disease.
6
q
What cell type is believed to play an important role in cutaneous immune system responses?
a) Freszel
b) Langerhans cells
c) melanocytes
d) Merkel cells
A
b) Langerhans cells
Explanation:
Langerhans cells, common to the epidermis, are accessory cells of the afferent process of the immune system. Merkel cells are the receptor cells of the epidermis that transmit stimuli to the axon through a chemical reaction. Melanocytes are special cells in the epidermis that are primarily involved in the production of melanin, which gives hair and skin its color. Phagocytes are white blood cells that engulf and destroy foreign materials.
6
q
The nurse assists a diabetic patient scheduled for cardiac catheterization. Which of the following questions is most important to the caregiver before the procedure?
a) "When was the last time you ate or drank?"
b) "Do you have chest pains?"
c) "What was your blood glucose reading in the morning?"
d) "Are you allergic to shellfish?"
A
d) "Are you allergic to shellfish?"
Explanation:
X-ray contrast media are used to visualize the coronary arteries. Some contrast media contain iodine, and the patient is evaluated prior to the procedure for previous reactions to the contrast media or allergies to iodine-containing substances (such as shellfish). If an allergy to the substance is known or suspected, antihistamines or methylprednisolone (Solu-Medrol) may be given before the procedure. While the other questions are important to the patient, the most important is to determine if the patient has an allergy to shellfish.
6
q
Which of the following statements is true when the nurse measures blood pressure (BP)? Select all that apply.
a) Ask the patient to sit while the blood pressure is measured.
b) The patient's arm should be placed at the level of the heart.
c) Using a blood pressure cuff that is too small will result in higher blood pressure readings.
d) The patient's blood pressure should be measured 1 hour after consuming alcohol.
e) Using an oversized blood pressure cuff will result in higher blood pressure readings.
A
a) Ask the patient to sit while the blood pressure is taken, b) The patient's arm should be placed at the level of the heart, c) Using a blood pressure cuff that is too small will result in a blood pressure reading highest.
Explanation:
All these statements are true when measuring blood pressure. If you wear a blood pressure cuff that is too large, the reading will be lower than your actual blood pressure. The patient should avoid smoking cigarettes or consuming caffeine 30 minutes before the blood pressure measurement.
6
q
A patient is hospitalized with possible acute pericarditis and pericardial effusion. Does the nurse know how to prepare the patient in which diagnostic test the patient's diagnosis was confirmed?
a) computed tomography
b) Chest X-ray
c) cardiac cautery
d) ecocardiograma
A
d) ecocardiograma
Explanation:
Echocardiograms are useful in detecting the presence of pericardial effusions associated with pericarditis. An echocardiogram can detect inflammation, pericardial effusion, tamponade, and heart failure. It can help confirm the diagnosis.
6
q
The nurse changes the bandage on a chronic wound. There are no signs of infection or heavy drainage. How long does the caregiver keep the wound covered?
- 6-12 Standard
- 12-24 Standard
- 24-36 Standard
- 48-72 standard 48
A
48-72 Standard
6
q
An appropriate nursing diagnosis for a client with extensive skin abrasion from scratching an allergic rash is:
1. Risk of compromising the integrity of the skin
2. Compromised skin integrity
3. Compromised tissue integrity
4. Risk of infection
A
2. Compromised skin integrity; The client has actual skin compromise from the rash and scratching, so they are no longer 'at risk'.
As the damage is at the skin level, the integrity of the tissue (option 3) is not compromised as it would affect deeper tissues. Superficial abrasions are also not prone to infection.
7
q
Which of the following causes wrinkles in older adults?
a) Decreased sebum
b) Loss of subcutaneous tissue
c) decreased estrogen production
d) decreased melanin
A
b) Loss of subcutaneous tissue
Explanation:
Loss of subcutaneous tissue causes wrinkles in older adults. Decreased melanin leads to a change in hair color to gray. The decrease in the production of estrogen and sebum does not cause wrinkles in the elderly.
8
q
A patient with a history of alcoholism who is scheduled for urgent surgery asks the nurse, "Why is everyone so worried about how much I drink?" What is the best response from the nurse?
a) "The amount of alcohol you drink will determine the amount of pain medication you will need after surgery."
b) “We may have consultants available after the transaction; if it is determined that you need help drinking.”
c) "This is a mandatory screening question for all patients undergoing surgery."
d) "It is important for us to know how much and how often you drink to avoid surgical complications."
A
d) "It is important for us to know how much and how often you drink to avoid surgical complications."
Explanation:
Alcohol use and alcoholism can contribute to serious postoperative complications. If medical and nursing staff are aware of use or misuse, proactive steps can be taken to prevent complications. Although alcohol can reduce the effectiveness of a medication, it does not determine the amount of pain medication prescribed after surgery. While this is a required assessment question and counselors can be provided for those who want help, these are not the best answers to answer the patient's question.
9
q
The nurse has difficulty seeing a patient's rash. What should the caregiver do to facilitate the assessment? Select all that apply.
a) Gently stretch the skin.
b) Apply an emollient.
c) Pull the skin down.
d) To facilitate assessment of the rash, the caregiver should gently stretch the skin and/or shine a flashlight on the side of the skin. The skin should never be pulled; The use of an emollient increases the difficulty for the caregiver to assess the rash.
e) Shine a flashlight from the side on the affected part.
A
a) Gently stretch the skin., e) Shine a flashlight laterally on the affected part.
Explanation:
To aid in the evaluation of the rash, the caregiver should gently stretch the skin and/or shine a flashlight on the side of the skin. The skin should never be pulled; The use of an emollient increases the difficulty for the caregiver to assess the rash.
10
q
For outpatients and inpatients scheduled for cardiovascular diagnostic procedures, a comprehensive baseline assessment is performed by the nurse to determine accurate baseline data. Which of the following data should be collected if the patient has chest pain?
a) Blood pressure in the left arm
b) description of pain
c) Apical pulse sound
d) Pulse rate in the upper extremities
A
b) description of pain
Explanation:
If the patient has chest pain, a medical history of location, frequency, and duration is required, as well as a description of the pain, whether it radiates to a specific area, what triggers it, and what relieves it. The nurse weighs the patient and measures vital signs. The caregiver can measure blood pressure in both arms and compare the results. The nurse assesses the apical and radial pulses, noting the rate, quality, and rhythm. The nurse also checks the peripheral pulses in the lower extremities.
10
q
A patient complaining of palpitations is diagnosed with atrial fibrillation caused by mitral valve prolapse. To alleviate symptoms, the caregiver should teach the patient which of the following dietary interventions.
a) Eliminate caffeine and alcohol
b) Reduce the amount of sodium and saturated fat.
c) Reduce the amount of acid drinks and fruits.
d) A patient complaining of palpitations is diagnosed with atrial fibrillation due to mitral valve prolapse. To alleviate symptoms, the caregiver should teach the patient which of the following dietary interventions.
A
a) Eliminate caffeine and alcohol
Explanation:
To minimize the symptoms of mitral valve prolapse, the caregiver should advise the patient to avoid caffeine and alcohol. The nurse encourages the patient to read product labels, especially over-the-counter products such as cough medicines, because these products may contain alcohol, caffeine, ephedrine, and epinephrine, which can cause irregular rhythms and other symptoms. The nurse also discusses possible diet, activity, sleep, and other lifestyle factors that may correlate with symptoms.
11
q
The nurse attends to a patient with bruised fingers and toes. Given these findings, which of the following actions should the nurse take?
a) Assess the patient's capillary refill.
b) Obtain an oxygen saturation level.
c) Evaluate the patient for pitting edema.
d) Obtain a 12-lead ECG recording
A
b) Obtain an oxygen saturation level.
Explanation:
Snapping of fingers and toes indicates chronic hemoglobin desaturation (decreased oxygenation) and is associated with congenital heart defects. Clinicians should assess the patient's O2 saturation level and intervene as indicated. Other classifications are not specified.
11
q
The nurse trains the patient on how to use a mini nebulizer. What should the nurse encourage the student to do? (Select all that apply)
- Hold your breath for a few seconds at the end of the inspiration.
- cough frequently
- Breathe quickly and deeply
- Assess progress frequently
- Extends the exhalation phase after using the nebulizer
A
At the end of the inspiration, hold your breath for a few seconds.
cough frequently
Assess progress frequently
12
q
The nurse trains unlicensed staff in gerontological skin considerations. The nurse observes that the participants understand the instructions when they know that older adults are at increased risk of stress injuries due to which of the following causes?
a) Loss of subcutaneous tissue
b) Sun damage over time
c) Reduced capillary loops
d) loss of net strips
A
d) loss of net strips
Explanation:
Elderly patients are at increased risk of shear injury due to loss of straight ridges due to thinning at the junction of the dermis and epidermis. Loss of the red cristae (anchor points between the two skin layers) allows even minor trauma/strain to the epidermis to separate it from the dermis. The other answers do not apply.
12
q
One patient received ketamine (Ketalar) for moderate sedation. What is the priority nursing intervention?
a) Frequent monitoring of vital signs
b) administer oxygen
c) Provide a quiet and dark room.
d) Hallucination test
A
a) Frequent monitoring of vital signs
Explanation:
Vital signs should be monitored frequently for respiratory depression and prompt intervention. Oxygen may be necessary if respiratory depression occurs; Therefore, the monitoring of vital signs is a higher priority care intervention. Providing a dark and quiet room is appropriate after the procedure is complete and the patient has recovered. Hallucinations can occur as a side effect of the drug.
12
q
Which of the following is a physiological effect of
more bed rest?
1. An increase in cardiac output
2. A decrease in lean body mass
3. A decrease in lung expansion
4. A decrease in urinary nitrogen excretion
A
Immobility that causes decreased lung elasticity.
Indentation and accumulation of secretions in portions.
lungs
13
q
A patient is admitted with aortic regurgitation. Which of the following classifications of drugs are contraindicated because they can cause bradycardia and decrease ventricular contractility?
a) Beta-blockers
b) Nitrate
c) ACE inhibitors
d) calcium channel blockers
A
d) calcium channel blockers
Explanation:
The calcium channel blockers diltiazem (Cardizem) and verapamil (Calan, Isoptin) are contraindicated in patients with aortic regurgitation because they can decrease ventricular contractility and cause bradycardia.
13
q
A nurse working in the emergency room examines a patient for signs and symptoms of appendicitis. What patient signs/symptoms should the nurse include in the patient's appendicitis sign/symptom report to the physician?
a) Pain on pressure in the right lower hemiabdomen
b) High fever
c) nausea
d) Pain in the left lower quadrant
A
c) nausea
Explanation:
Nausea is typically associated with appendicitis, with or without vomiting. The pain is usually felt in the right lower quadrant. Rebound pain, or pain felt when pressure is released from the abdomen, may be present with appendicitis. Low-grade fever is associated with appendicitis.
14
q
A 76-year-old man presents to the emergency department complaining of "laryngitis." The triage nurse should ask if the patient has a history that includes:
a) Gastroesophageal reflux disease (GERD)
b) Atemversagen (RF)
c) Chronic Obstructive Pulmonary Disease (COPD)
d) heart failure (CHF)
A
a) Gastroesophageal reflux disease (GERD)
Explanation:
The nurse should ask if the patient has a history of GERD. Laryngitis in older adults is common and may be secondary to GERD. Older adults are more likely to have impaired esophageal peristalsis and a weaker esophageal sphincter. COPD, CHF, and RF are not associated with laryngitis in older adults.
quince
q
An 18-year-old woman is in the emergency room with a fever and cough. The nurse takes her vital signs, listens to her lung sounds, listens to her heart sounds, determines her well-being, and collects blood and sputum samples for analysis from her. What standard of practice is performed?
A. Diagnose
B. Evaluation
C. Classification
D. Implementation
A
C. Classification
Assessment is the collection of comprehensive data on the health status and/or situation of the patient.
quince
q
The factor that best advanced the practice
Nursing in the 21st century was:
1. Urban growth
2. Teachings of Christianity
3. Better training of nurses
4. Better conditions for women
A
3. Better training of nurses
Nursing is a combination of knowledge of the
natural sciences, humanities and social sciences
along with clinical skills
sixteen
q
A nurse prepares to examine a patient for postural changes in blood pressure. Which of the following statements indicates the need for additional training?
a) Obtaining the measurements in the supine position before the measurements sitting and standing
b) Place the patient in the supine position for 10 minutes before measuring baseline blood pressure and heart rate.
c) Wait 30 seconds after each position change before measuring blood pressure and heart rate (HR).
d) Measure the patient's blood pressure while the patient is sitting on the edge of the bed with his feet dangling.
A
c) Wait 30 seconds after each position change before measuring blood pressure and heart rate (HR).
Explanation:
When evaluating patients for postural hypotension, the following steps are recommended: Place the patient supine for 10 minutes before taking baseline blood pressure and heart rate measurements; Put the patient in a sitting position with the legs in a hanging position, wait 2 minutes, and then check the BP and HR measurements; if the patient is asymptomatic or has no significant decreases in systolic or diastolic blood pressure, help the patient to a standing position, take action immediately, and recheck after 2 minutes; Continue measurements every 2 minutes for a total of 10 minutes to rule out orthostatic hypotension. Place the patient in the supine position if orthostatic hypotension is detected or if the patient is symptomatic. Document HR and BP measured in each position (eg, supine, sitting, and standing) and any signs or symptoms accompanying postural changes.
sixteen
q
By measuring the blood pressure in the arm of each patient, does the caregiver recognize that, in the healthy adult, which of the following statements is true?
a) The pressure can vary between the arms by 10 mm Hg or more.
b) The pressures can vary, being the highest pressure in the left arm.
c) The pressure must be equal in both arms.
d) The pressures should not differ by more than 5 mm Hg between arms.
A
d) The pressures should not differ by more than 5 mm Hg between arms.
Explanation:
Usually, in the absence of vascular disease, there is no more than 5 mm Hg difference between arm exercises. The pressures in each arm do not have to be the same to be considered normal. Pressures that vary between the arms by more than 10 mm Hg indicate an abnormal finding. As a normal anatomic variant, the pressure on the left arm is not expected to be greater than on the right.
sixteen
q
After percutaneous coronary intervention (PCI), the patient returns to the nursing unit with large peripheral vascular access sheaths. Does the caregiver understand which of the following methods of inducing hemostasis after sheath placement is contraindicated?
a) Apply a sandbag to the area
b) Direct manual printing
c) Application of a vascular occlusion
d) Application of a mechanical compression device
A
a) Apply a sandbag to the area
Explanation:
Sandbag placement at the sheath insertion site is not effective in reducing the incidence of bleeding and is not an acceptable standard of care. Application of a vascular closure device (Angioseal, VasoSeal), direct manual pressure at the sheath insertion site, and application of a mechanical compression device (C-shaped clamps) are appropriate methods to induce hemostasis after from the removal of the peripheral sheath.
sixteen
q
A patient visits the doctor to determine what type of allergy is causing a rash. What kind of exams does the nurse expect from the doctor?
- skin biopsy
- skin scraping
- Tzanck smear
- patch test
A
patch test
sixteen
q
A patient with an abdominal surgical wound sneezes and says, "Something doesn't feel right about my wound." The nurse notes that the upper half of the surgical wound edges are no longer approximated and the lower half remain approximated. What nursing documentation makes the most sense?
a) After sneezing, the wound pustule.
b) After a sneeze, the wound bled.
c) After a sneeze, the wound dehisces.
d) After a sneeze, the wound was eviscerated.
A
c) After a sneeze, the wound dehisces.
Explanation:
Dehiscence is the partial or complete separation of the edges of the wound. Evisceration is the protrusion of organs through a surgical incision. Pustules refer to the formation of pustules. Bleeding is excessive bleeding.
17
q
A nurse examines a patient who is bleeding profusely from the nose. The nurse documents this finding as which of the following conditions?
a) runny nose
b) Dysphagia
c) nosebleeds
d) Xerostomia
A
c) nosebleeds
Explanation:
Epistaxis occurs due to the bursting of small dilated vessels in the lining of any area of the nose. Xerostomia refers to dry mouth. Rhinorrhea refers to the drainage of a large amount of fluid through the nose. Dysphagia refers to difficulty swallowing.
17
q
If the caregiver observes that the postoperative patient has a consistently low oxygen saturation through the O2 saturation monitor, even though the patient's breathing appears normal, what action should the caregiver take first?
a) Inform the doctor.
b) Assess the patient's heart rate and nail bed.
c) provide oxygen.
d) Document the results.
A
b) Assess the patient's heart rate and nail bed.
Explanation:
A patient may have low oxygen saturation levels due to wearing certain nail polish colors or have an irregular heart rhythm such as atrial fibrillation. These points must be evaluated to ensure the accuracy of the oxygen measurement. Once the reading is confirmed to be accurate, the nurse may need to administer oxygen, notify the physician, and document the results.
18
q
A patient is given succinylcholine and propofol (Diprivan) to induce anesthesia. One hour after administration, the patient has muscle rigidity with a heart rate of 180. What should the caregiver do first?
a) Administer dantrolene sodium (Dantrium).
b) Notify the surgical team.
c) Get cool blankets.
d) Document the results of the evaluation.
A
b) Notify the surgical team.
Explanation:
Tachycardia and muscle rigidity are usually the first signs of malignant hyperthermia. Early detection of malignant hyperthermia increases survival. The nurse would document the findings, administer dantrolene sodium (Dantrium), receive cooling blankets as part of the treatment for malignant hyperthermia, but she would first have to ensure that the surgical team is aware of the findings.
19
q
While tending to a post-op patient, the nurse discovers that the patient is bleeding and in hypovolemic shock. What nursing intervention will control and minimize bleeding and shock?
a) Strengthen the bandage or apply pressure if bleeding is evident.
b) Monitoring of vital signs every 15 minutes
c) raise the head of the bed
d) Encourage the patient to breathe deeply
A
a) Strengthen the bandage or apply pressure if bleeding is evident.
Explanation:
The caregiver should reinforce the bandage or apply pressure if there is open bleeding. Clinicians should keep the head of the bed flat unless there is a contraindication. Encouraging the patient to take deep breaths does not help to control and minimize bleeding and shock. Monitoring vital signs every 15 minutes is an appropriate nursing intervention, but it does not minimize bleeding and shock; this alone will help determine the scope and progression of the problem.
20
q
A nurse cares for a patient after coronary artery bypass grafting (CABG) surgery. The nurse discovers that bloody discharge is constantly oozing from various puncture sites. The nurse is waiting for the doctor to prescribe which of the following drugs to counteract the unfractionated heparin was given to the patient.
a) aspirin
b) Clopidogrel (Plavix)
c) Protaminsulfato
d) Alteplasa (t-PA)
A
c) Protaminsulfato
Explanation:
Protamine sulfate is known to be an unfractionated heparin antagonist (it neutralizes heparin). Alteplase is a thrombolytic. Clopidogrel (Plavix) is an antiplatelet drug given to reduce the risk of thrombus formation after coronary stent placement. The anticoagulant effect of aspirin does not reverse the effect of heparin.
20
q
It is recommended that the patient apply a suspension-type lotion to the dermatosis site. Should the nurse advise the patient how often to apply the lotion for it to be effective?
- all time
- every 3 hours
- every 12 hours
- Every day at the same time
A
every 3 hours
21
q
The nurse reads the doctor's report on the physical examination of an elderly patient. The patient has a xanthelasma, which of the following symptoms is it related to?
a) Dark skin color
b) Bright red birthmarks
c) liver spots
d) Yellowish waxy deposits on the upper eyelids.
A
d) Yellowish waxy deposits on the upper eyelids.
Explanation:
Xanthelasma is a common and benign manifestation of skin aging or can occasionally indicate hyperlipidemia. Symptoms are yellowish waxy deposits on the upper eyelids. Solar lentigo is the term that refers to liver spots. Melasma is the term that refers to a dark skin color. Cherry angioma is the term used to describe a bright red mole.
21
q
A patient goes on to say, "I know everything will be okay." What cognitive coping strategy should the nurse document?
a) music therapy
b) Optimistic self-recitation
c) distraction
d) Photos
A
b) Optimistic self-recitation
Explanation:
When this patient verbalizes this statement, it is an optimistic response. The images are formed when the patient focuses on a pleasant experience or a restorative scene. Distraction occurs when the patient thinks of a funny story or recites a favorite poem or song. Music therapy would be an incorrect answer.
21
q
A patient refuses to remove her wedding ring when preparing for surgery. What is the best course of action for the caregiver?
a) Leave the ring on the patient and cover it with tape.
b) Remove the ring as soon as the patient is sedated.
c) Warn the surgeon to cancel the operation.
d) Discuss the risk of infection from wearing the ring.
A
a) Leave the ring on the patient and cover it with tape.
Explanation:
Most facilities allow the wedding band to remain on the patient during the surgical procedure. The nurse has to secure the ring with tape. Although it is appropriate to discuss the risk of infection, the patient has already refused to remove the ring. The operation must not be canceled and the ring must not be removed without permission.
21
q
A patient undergoes a lumbar puncture. The nurse explains the surgical placement to the patient. Which of the following statements by the nurse is true?
a) "You will lie on your back with the head of the bed at 30 degrees."
b) "You will be lying on the table, face down."
c) "You lie on your side with your knees to your chest."
d) "You lie on your back and the table is at an angle so that your head is below your feet."
A
c) "You lie on your side with your knees to your chest."
Explanation:
During the lumbar puncture, the patient usually lies on their side in a knee-chest position. Lying on the stretcher, face down, the vertebral spaces are not opened to allow access for lumbar puncture. If the patient is in the supine position, access to the surgical site is not possible.
22
q
A nurse cares for an elderly couple at a community nursing home. During the family assessment, she discovers that the couple has many expired medications and multiple medications for their respective chronic conditions. They realize they are going to two different health professionals. The nurse begins to work with the couple to find out what they know about their medications and helps them choose one provider instead of two. This is an example of what QSEN (Quality and Safety in Nursing Education) competency?
A. Non-patient centered care
B. Security
C. Teamwork and collaboration
D. IT
A
B. Security
Helping patients understand the consequences and complications of various medications helps build safety competency.
22
q
What should the nurse documenting an acute open wound include? Select all that apply.
a) Skin around the wound
b) size of the wound
c) Eruptions
d) base of the wound
A
a) perilesional skin, b) size of the wound, d) wound bed
Explanation:
When documenting an acute open wound, the caregiver should consider the size of the wound, the condition of the periwound skin (skin surrounding the wound), and a description of the wound bed. Rash pattern refers to patterns of injury to a patient's skin and does not apply to an acute open wound.
22
q
Which of the following is an autoimmune disease that affects immunoglobulin G?
a) bullous pemphigoid
b) Síndrome de Stevens-Johnson (SSJ)
c) pénfigo
d) Toxic Epidural Necrolysis (TEN)
A
C. pénfigo
Explanation:
Pemphigus is an autoimmune disease that involves immunoglobulin G. NET, SJS, and bullous pemphigoid do not involve immunoglobulin G
23
q
What action should the caregiver take when caring for a client with contact arrangements for an infected, oozing foot ulcer?
1. Wear a mask during dressing changes.
2. Provide disposable trays and cutlery.
3. Follow standard precautions in all customer interactions.
4. Use aseptic surgical technique in any direct contact with the client.
A
3. Follow standard precautions in all customer interactions.
Rationale: Standard accommodations include all aspects of contact accommodations except placing the client in a private room. A mask is indicated when working on a sterile, uninfected wound (option 1). For patients with infected wounds that are not likely to contaminate the patient's hands, disposable food trays are not necessary (Option 2). Sterile technique (surgical asepsis) is not indicated for all client contacts (option 4). The nurse would use clean technique when dressing the wound to prevent the introduction of additional microbes.
24
q
The nurse calculates the patient's smoking history in pack-years. The patient was recently diagnosed with malignant lung cancer. The patient claims to have smoked two packs of cigarettes a day for the past 11 years. Does the nurse correctly document the patient's packing years as which of the following?
a) 11
second) 5
c) 22
d) 10
A
c) 22
Explanation:
Smoking history is usually expressed in pack-years, which is the number of packs of cigarettes smoked per day multiplied by the number of years the patient has smoked. It is important to know if the patient is still smoking or when she stopped smoking. In this situation, the patient's pack year is 22 (2 × 11).
24
q
The nurse educates patients who need surgery for various diseases about the perioperative experience. What kind of nursing training is the most appropriate?
a) Three operational phases and security measures for each phase
b) Intraoperative techniques used to perform the operation
c) Risks and benefits of surgical procedures
d) Expected level of pain and narcotic analgesics used to treat pain
A
a) Three operational phases and security measures for each phase
Explanation:
The perioperative phase includes the preoperative, intraoperative, and postoperative phases. Specific safety guidelines are followed for all surgical patients. The information provided should be general enough to provide information about surgeries and should not focus on individual surgeries as all patients have different surgeries. Intraoperative techniques, expected pain levels, and analgesics are specific to the patient and type of surgery. The risks and benefits of the surgical procedure should be discussed by the physician.
24
q
What nursing intervention has the highest priority for a patient in the immediate postoperative period?
a) Maintain an open airway.
b) Bleeding test
c) Monitoring of vital signs at least every 15 minutes
d) hourly determination of diuresis
A
a) Maintain an open airway.
Explanation:
All the interventions listed are correct. The highest priority intervention is to maintain an open airway. Without a patent airway, the other procedures to monitor vital signs and urination, along with evaluation for bleeding, become secondary to the possibility of oxygen deprivation.
26
q
A nurse assists a patient undergoing a peripheral arteriogram. How should the caregiver assess the adequacy of peripheral perfusion?
a) Observe the patient for bleeding
b) By hemodynamic monitoring
c) Checking peripheral pulses
d) By checking for cardiac arrhythmias
A
c) Checking peripheral pulses
Explanation:
Peripheral arteriography is used to diagnose vascular diseases in smaller arteries. The nurse monitors the patient for bleeding and cardiac arrhythmia and assesses the adequacy of peripheral perfusion by frequently checking peripheral pulses. Hemodynamic monitoring is used to assess the volume and pressure of blood in the heart and vascular system.
26
q
A 35-year-old woman was diagnosed with hypertension. The patient is a stockbroker, smokes daily and is also diabetic. On a return visit, the patient reports that she finds it inconvenient and she takes a long time to go to the doctor regularly just to check her blood pressure (BP). As a nurse, which of the following aspects of patient education would she recommend?
a) Guidance to quit smoking
b) Acquisition of a blood pressure cuff for self-monitoring
c) Discussion of methods to reduce stress
d) administration of glycemic control
A
b) Acquisition of a blood pressure cuff for self-monitoring
Explanation:
Since it takes a long time for this patient to visit the doctor just to measure his blood pressure, as a caregiver, you can suggest him to use an automatic cuff at a local pharmacy or buy a self-monitoring cuff. Discussing methods to reduce stress, recommending smoking cessation, and glycemic monitoring would constitute patient education on how to control hypertension.
26
q
The nurse looks at an African-American patient with a large, enlarged area of scar tissue on his left earlobe. How does the nurse document this finding?
- Atrophy
- scar
- lichenification
- Keloid
A
Keloid
26
q
A surgical patient was transferred to the waiting room. What nursing intervention(s) promote safe and effective nursing? Select all that apply.
a) Maintain an aseptic environment.
b) Identify the patient with two identifiers.
c) Review the surgical site and mark accordingly.
d) Administer oral fluids to the patient.
e) Review medical records.
f) Connect the grounding devices to the patient.
A
b) Identify the patient by means of two identifiers. c) Review the surgical site and mark accordingly. e) Review medical records.
Explanation:
Identifying the patient, reviewing and marking the surgical site, and reviewing medical records promote safe and effective care while the patient is in the waiting room. Maintaining an aseptic environment and applying grounding devices are part of the intraoperative phase. Oral fluids should not be administered while the patient is in the holding area.
27
q
A nurse assists a patient admitted to the emergency room with an uncomplicated nasal fracture. The nasal packing is complete. Which of the following interventions should the nurse include in patient care?
a) Apply pressure on the convexity of the nose.
b) Apply an ice pack
c) Limit your fluid intake
d) Place the patient in lateral decubitus.
A
b) Apply an ice pack
Explanation:
After a broken nose, the nurse applies ice and encourages the patient to keep their head up. The nurse instructs the patient to apply ice packs to the nose to reduce swelling. The pack used to stop bleeding can be awkward and uncomfortable, and the blockage of the nostrils by the pack forces the patient to breathe through the mouth. This, in turn, leads to dehydration of the oral mucosa. Mouthwashes help to hydrate mucous membranes and reduce the odor and taste of dried blood in the oropharynx and nasopharynx. Direct pressure is not indicated in this situation.
27
q
A nurse examines a patient before a magnetic resonance angiography (MRA) of the heart. Which of the following should the caregiver do before the patient undergoes the procedure? Select all that apply.
a) Offer the patient headphones to listen to music during the procedure.
b) Remove the patient's jewelry.
c) Calm the patient before the procedure.
d) Place the patient in the prone position for the procedure.
e) Remove the Transderm Nitro patch from the patient.
A
a) Offer the patient headphones to listen to music during the procedure. b) Remove the patient's jewelry. e) Remove the Transderm Nitro patch from the patient.
Explanation:
Transdermal patches containing a heat-conductive aluminized layer (eg, NicoDerm, Androderm, Transderm Nitro, Transderm Scop, Catapres-TTS) should be removed prior to MRA to avoid skin burns. A claustrophobic patient may need to receive a mild sedative before undergoing MRA. During an MRA, the patient is placed supine on a table that is placed in a closed tube or imager that contains the magnetic field. Patients are instructed to remove jewelry, watches, or other metallic objects (eg, EKG leads). The magnetic coils generate an intermittent ringing or tapping sound that can be annoying, so the patient may be fitted with a headset to listen to music.
28
q
Nurse administers medications in a surgical medical unit. A patient is prescribed to receive 40 mg of Corgard (Nadolol) orally to treat high blood pressure. Which of the following should the caregiver do before administering the medication?
a) Regret or patient
b) Checking the patient's serum K+ level
c) Checking the patient's heart rate
d) check the patient's urine output
A
c) Checking the patient's heart rate
Explanation:
Corgard is a beta blocker. A desired effect of this drug is the reduction of heart rate in patients with tachycardia and arterial hypertension (PA). Physicians should check the patient's heart rate (HR) before administering Corgard to ensure that the patient's heart rate does not fall below 60 (beats per minute (bpm)). The other interventions are not indicated before the administration of a beta-blocker.
28
q
When a patient diagnosed with angina complains that they have more chest pain even at rest, the period of pain is longer and it takes less time to appear, does the caregiver recognize that the patient is describing some type of angina?
a) insoluble
b) variant
c) inestable
d) fireproof
A
c) inestable
Explanation:
Unstable angina, also known as crescendo or pre-infarction angina, indicates the need for a change in treatment. Intractable or refractory angina produces severe, disabling chest pain that does not respond to conventional treatment. Variant angina is described as reversible ST-segment elevation rest pain and is probably due to coronary artery vasospasm. Intractable or refractory angina produces severe, disabling chest pain that does not respond to conventional treatment.
28
q
The nurse is irritating a patient's colostomy when the patient begins to complain of cramps. What is the appropriate role of the caregiver?
a) Increase the shipping fee.
b) Stop watering immediately.
c) Change the irrigation fluid to normal saline.
d) Secure the tube and let the patient rest.
A
d) Secure the tube and let the patient rest.
Explanation:
The nurse should secure the tube and allow the patient to rest if he or she begins to complain of cramping during colostomy irrigation. Once the spasms stop, the caregiver can resume washing.
29
q
A patient is 8 hours after appendectomy surgery and is concerned and says, “Something is wrong. My pain is worse than ever and my stomach is swollen." Blood pressure is 88/50, pulse is 115, and breathing is 24 and labored. The belly is soft and distended. No obvious bleeding was noted. What caregiver measures make the most sense?
a) Inform the patient that this is the normal course after abdominal surgery.
b) Administer morphine on demand.
c) Walk with the patient to reduce abdominal distension.
d) Notify the doctor.
A
d) Notify the doctor.
Explanation:
The results must be communicated to the doctor. The patient may have internal bleeding and may need to be operated on again. The patient may need pain medication, but morphine further lowers blood pressure and can cause further complications. Walking the patient increases the risk of injury because the patient may experience orthostatic hypotension. What the patient experiences is not the normal course after abdominal surgery.
30
q
A newly extubated postoperative patient begins to have nausea and vomiting. What should the caregiver do first?
a) Administer antiemetic.
b) Obtain extraction equipment.
c) Turn the patient onto his side.
d) provide vomit receptacles.
A
c) Turn the patient onto his side.
Explanation:
The caregiver should turn the patient on their side to prevent aspiration. The nurse may need to obtain suction equipment, provide a vomit container, or administer an antiemetic, but the first priority is to protect the patient's airway by preventing aspiration.
31
q
A nurse cares for a patient newly diagnosed with secondary hypertension. Which of the following diseases contributes to the development of secondary hypertension?
a) Calcium deficiency
b) liver function
c) kidney disease
d) Acid imbalance
A
c) kidney disease
Explanation:
Secondary hypertension occurs when a cause of high blood pressure can be identified. These causes include renal parenchymal disease, narrowing of the renal arteries, hyperaldosteronism (mineralocorticoid hypertension), pheochromocytoma, certain medications (eg, hypertension can also occur during pregnancy; women who have high blood pressure during pregnancy have an increased risk of ischemic heart disease, heart attack, stroke, kidney disease, diabetes, and death from heart attack Calcium deficiency or an acid imbalance does not contribute to high blood pressure.
31
q
The nurse cares for an adult patient with normal blood pressure. What the nurse should know would be the approximate imperceptible water loss per day in this patient.
- 250ml/diameter
- 600ml/diameter
- 800ml/diameter
- 1000ml/diameter
A
600ml/diameter
32
q
The nurse prepares the patient's tracheostomy connection with the newly inserted tracheostomy tube. Which of the following, if done by the nurse, indicates a need for further review of the process?
a) Apply clean tracheostomy bands and remove dirty bands after placing new bands
b) put on clean gloves; remove and dispose of soiled bandage in a biohazard bin
c) Clean the wound and the plate with a sterile cotton swab moistened with hydrogen peroxide
d) Dry and reinsert the inner cannula or replace with a new disposable inner cannula
A
a) Apply clean tracheostomy bands and remove dirty bands after placing new bands
Explanation:
With a new tracheostomy, two people must help change the tie. The other actions are correct when performed by the caregiver during tracheostomy care.
32
q
At which of the following anatomical locations does the caregiver hear the PMI (point of maximum impulse)?
a) 1 inch to the left of the xiphoid process
b) Left midclavicular line, fifth intercostal space
c) 2 inches to the left of the bottom of the sternum
d) middle of the sternum
A
b) Left midclavicular line, fifth intercostal space
Explanation:
The left ventricle is responsible for the apical impulse, or point of maximum impulse, which is usually palpable in the left midclavicular line of the chest wall in the fifth intercostal space. The right ventricle is anterior, just below the sternum. The use of inches to identify the location of the PMI is inappropriate due to variations in human anatomy. Auscultation below and to the left of the xiphoid process detects GI sounds but not PMI.
33
q
Which of the following nursing roles might have prescriptive authority in your practice? (Choose all that apply.)
A. Intensivpfleger
b.nurse
C. Certified Clinical Nurse Specialist
D. Head of Nursing
A
BC
34
q
The circulating caregiver is uncertain that proper technique has been followed when placing an object in the sterile field during a surgical procedure. What is the best action of the nurse?
a) Remove the entire field of sterile use.
b) Remove the object from the sterile field.
c) Ask another nurse to verify the technique used.
d) Mark the patient's chart for later review of infections.
A
a) Remove the entire field of sterile use.
Explanation:
If there is any doubt about maintaining sterility, the field should be considered non-sterile. Since the object in question was placed in the sterile field, the sterile field should be taken out of use. Removing the individual element is not appropriate as the field may have been contaminated. Subsequent review of patient records does not reduce the risk of infection. Although another nurse was able to observe the technique used to place objects in a sterile field, this does not solve the immediate problem.
35
q
The nurse explains the DASH diet to a patient diagnosed with hypertension. How many servings of meat, fish and poultry should a patient eat per day?
a) 4 or
b) 7 u 8
c) 2 o 3
d) 2 or less
A
d) 2 or less
Explanation:
On the DASH diet, two or fewer servings of meat, fish, and poultry are recommended.
35
q
A nurse teaches a patient about valve replacement surgery. Which patient statement shows that she understands the benefit of an autograft replacement valve?
a) "The valve is from a tissue donor and I will not need to take any anticoagulant medication when I am discharged."
b) "The valve will be made from my own heart valve and I will not need to take any anticoagulant medication when I am discharged."
c) "The valve is made of porcine tissue and I do not need to take any anticoagulant after discharge."
d) "The valve is mechanical and will not deteriorate or need replacement."
A
b) "The valve will be made from my own heart valve and I will not need to take any anticoagulant medication when I am discharged."
Explanation:
Autografts (ie, autologous valves) are obtained by removing the patient's own pulmonary valve and a portion of the pulmonary artery for use as an aortic valve. Anticoagulation is not necessary because the valve is the patient's own tissue and is not thrombogenic. Autograft is an alternative for children (it can grow as the child grows), women of childbearing age, young adults, patients with a history of stomach ulcers, and people who cannot tolerate anticoagulation. Aortic valve autografts have been viable for more than 20 years.
36
q
A nurse assesses the arterial blood gases of a patient with acute respiratory failure (ARI). Which of the following findings are consistent with this disorder?
a) pH 7,35, PaCO2 48 mmHg
b) pH 7,46, PaO2 80 mmHg
c) pH 7,36, PaCO2 32 mmHg
d) pH 7,28, PaO2 50 mmHg
A
d) pH 7,28, PaO2 50 mmHg
Explanation:
AKI is defined as a decrease in arterial pressure of oxygen (PaO2) to less than 50 mm Hg (hypoxemia) and an increase in arterial pressure of carbon dioxide (PaCO2) to more than 50 mm Hg (hypercapnia) at a lower arterial pH. 7.35.
37
q
Does the decrease in pulse pressure reflect the following?
a) Increased systolic volume
b) Decreased compliance of the arteries
c) Tachycardia
d) Reduced systolic volume
A
d) Reduced systolic volume
Explanation:
The decrease in pulse pressure reflects a decrease in stroke volume and a decrease in ejection velocity or obstruction of blood flow during systole. Elevated pulse pressure would indicate decreased arterial compliance along with bradycardia.
39
q
A 78-year-old woman undergoes surgery on her right hip to repair a hip fracture. What nursing measures are appropriate during the intraoperative phase?
a) Discontinue analgesics due to renal failure.
b) Position the patient correctly with adequate padding and support.
c) Discuss the need for higher doses of anesthetic with the anesthetist.
d) Maintain the operating room temperature at 18°C to avoid hypothermia.
A
b) Position the patient correctly with adequate padding and support.
Explanation:
Adequate padding and support should be used to prevent positioning injuries. The elderly have less bone mass, which increases the risk of intraoperative positioning injuries. Pain relievers can still be used, just in smaller doses, due to reduced liver and kidney function. Lower doses of anesthetics are used in older adults for the same reason as pain relievers. The operating room is normally kept between 20°C and 24°C; 18°C is below the recommended temperature and may promote hypothermia in older people who already have impaired thermoregulation and are prone to hypothermia.
40
q
A PACU nurse cares for a patient with the following assessment data: pale, cool, and clammy skin; thready pulse of 122; blood pressure 78/60; urine output of 25 ml/h; Temperature 99.2°F. What caregiver interventions make sense? Select all that apply.
a) Apply oxygen as directed.
b) Monitor neurological status frequently.
c) Keep the airway open.
d) Elevate the head of the bed 30 degrees.
e) Put on a warm blanket.
f) administration of blood products by order.
A
a) Administer oxygen as directed, b) Monitor neurological status frequently, c) Maintain a clear airway, f) Administer blood products as directed.
Explanation:
The patient shows signs and symptoms of shock. The patient in shock may lose the ability to protect their airways. Often a neurological exam can provide information about reduced oxygen levels in the brain. The administration of blood products can reverse the signs and symptoms of shock. In shock, there is an increased need for oxygen, so oxygen administration is appropriate. The head of the bed should not be raised. The patient should be lying down or in the Trendelenburg position. Applying a heating blanket when the patient is not hypothermic can cause vasodilation, which can further reduce blood pressure and blood flow to vital organs.
41
q
The purpose of postoperative exercises for the legs is:
1. Maintain muscle tone
2. Promotion of venous return
3. Assess range of motion
4. Train tired muscles
A
Promotion of venous return
2. Promotes normal venous return and circulation
blood circulation
42
q
Age-related changes associated with the cardiac system include the following? Select all that apply.
a) Enlargement of the left atrium
b) Fibrous endocardium
c) thinning of the myocardium
d) increase the number of cells in the SA node
A
a) Enlargement of the left atrium, b) endocardial fibrosis
Explanation:
Age-related changes associated with the cardiac system include endocardial fibrosis, enlargement of the left atrium, decreased number of SA node cells, and thickening of the myocardium.
42
q
A nurse cares for a patient who has been prescribed Bumex (bumetanide) to treat stage 2 hypertension. Which of the following statements indicates that the patient is experiencing an adverse drug effect?
a) Electrocardiogram (EGG) tracing showing T-wave spikes
b) Blood sugar level of 160 mg/dL
c) serum potassium level of 3.0 mEq/L
d) Diuresis of 90 cc/ml 1 hour after drug administration
A
c) serum potassium level of 3.0 mEq/L
Explanation:
Bumex is a loop diuretic that can cause fluid and electrolyte imbalances. Low serum potassium levels can occur in patients taking these drugs. ECG changes associated with increased serum potassium include increased T waves. Diuresis is a desired effect after Bumex administration. Serum glucose level is elevated and requires intervention; however, this increase is not related to the administration of Bumex.
42
q
Nurses know that it is important to promote adequate tissue perfusion after cardiac surgery. Which of the following actions should the caregiver take to prevent deep vein thrombosis (DVT) and the possible development of pulmonary embolism (PE)? Select all that apply.
a) Stimulate the crossing of legs.
b) Avoid putting your knees on the bed.
c) Start passive exercises.
d) Place pillows behind the knees.
e) Use antiembolic agents.
A
b) Avoid raising the knees in bed c) Start passive exercises e) Wear anti-embolic stockings.
Explanation:
Preventive measures to avoid venous congestion include: application of sequential pneumatic compression bandages or antiembolic stockings; discourage crossing your legs; Avoid raising your knees in bed; omission of pillows on the back of the knees; Start with passive exercises, followed by active exercises to increase circulation and prevent venous congestion.
42
q
A
43
q
Nurses on a medical floor in intensive care notice an increase in pressure ulcer formation in their patients. A care consultant decides to compare two types of treatment. The first is the method currently used to assess pressure ulcer risk. The second uses a new screening tool to identify patients at risk. Given this information, Nursing Consultant illustrates which career?
A. Clinical Nurse Specialist
B. Nurse administrator
C. Nursing Instructor
D. Research nurse
A
D. Research nurse
The nursing researcher examines issues to improve nursing and to further define and expand the scope of nursing practice. He or she typically works in an academic setting, a hospital, or a nonprofit or independent professional agency.
43
q
A nurse is caring for a patient in the emergency department who has a BNP natriuretic peptide (BNP) level of 115 pg/mL. The nurse understands that this finding is more indicative of which of the following statements?
a) heart failure
b) myocardial infarction
c) pulmonary edema
d) Ventricular hypertrophy
A
a) heart failure
Explanation:
A BNP level greater than 100 pg/mL indicates heart failure. Because this serum laboratory test can be performed quickly, BNP levels are useful for immediate diagnosis of heart failure in settings such as the emergency room. Elevations in BNP can occur from a number of other conditions, including pulmonary embolism, myocardial infarction (MI), and ventricular hypertrophy. Therefore, the physician correlates BNP levels with abnormal findings on physical examination and other diagnostic tests before making a definitive diagnosis of heart failure.
44
q
A nurse provides a nasal tracheal tube to a patient in the intensive care unit. Based on tube placement, the caregiver understands that the patient is at risk of developing which of the following conditions?
a) severe epistaxis
b) orbital cellulitis
c) sinusitis
d) subperiosteal abscess
A
c) sinusitis
Explanation:
Patients with nasotracheal and nasogastric tubes are at increased risk of developing sinus infections. Therefore, it is critical to accurately assess patients with these probes. Removal of the nasotracheal or nasogastric tube as soon as the patient's condition allows allows for sinus drainage, which may prevent septic complications. Severe epistaxis is not a complication of nasotracheal placement. Subperiosteal abscess and orbital phlegmon are complications of chronic rhinosinusitis.
44
q
During a routine exam of a patient, the nurse notices that the nails are concave in shape. Which of the following statements does this finding indicate?
a) fungal infection
b) Cardiopulmonary disease of many years
c) Poor circulation
d) iron deficiency anemia
A
d) iron deficiency anemia
Explanation:
The concave shape of the nails, called the shell, is a sign of iron deficiency anemia. Hitting the nails at an angle of more than 160 degrees indicates a long history of cardiopulmonary disease. Nails thicken with fungal infection and poor blood circulation.
45
q
Which of the following terms refers to a transplant taken from one part of a patient's body and used on another part of the same patient's body?
a) Autotrasplante
b) Aloenxerto
c) Homotrasplante
d) Heteroenxerto
A
a) Autotrasplante
Explanation:
Full-thickness autografts and pedicled flaps are often used for reconstructive surgery months to years after the original injury. An allograft is a transplant that is transferred from one human (living or dead) to another human. A homograft is a transplant that is transferred from one human (living or dead) to another human. A xenograft is a transplant obtained from an animal of a different species than the recipient.
46
q
A 77-year-old woman presents to her local community center for a blood pressure test. Women's blood pressure is recorded as 180/90 mm Hg. The woman has a history of high blood pressure, but she is currently not taking medication. Which of the following questions should the nurse ask the patient first?
a) "Why don't you take your medicine?"
b) "Can you pick up your medication at your pharmacy?"
c) "Do you have problems paying for your medication?"
d) "What medications do you prescribe?"
A
a) "Why don't you take your medicine?"
Explanation:
It is important that the nurse first determine why the patient is not taking their medication. Adherence to the therapeutic program may be more difficult for the elderly. The drug regimen can be difficult to remember and the cost can be challenging. If necessary, monotherapy (treatment with a single active ingredient) can simplify the drug regimen and make it less expensive. The other questions are appropriate, but the priority is to determine why the medication regimen is not being followed.
47
q
A nurse performs chest auscultation on a patient with asthma. How does the caregiver describe the high-pitched, hissing sounds of music being heard?
- rattles
- cracks
- panting
- snores
A
panting
48
q
A patient is receiving mechanical ventilation in the intensive care unit. Ventilator alarms begin to sound. Which of the following actions should the caregiver do first?
a) Ventilate the patient manually.
b) Inform the physiotherapist.
c) Troubleshooting to identify the malfunction.
d) Reposition the endotracheal tube (ET).
A
c) Troubleshooting to identify the malfunction.
Explanation:
Caregivers should first try to identify and correct the problem immediately, and if the problem cannot be identified or corrected, the patient should be manually ventilated with an ambu bag. The physiotherapist may be warned, but this is not the first action of the nurse. Clinicians should not change the position of the ET tube as a first response to an alarm.
49
q
The nurse performs a physical examination of a patient's windpipe. The nurse inspects and palpates the trachea for which of the following?
a) mucous cords
b) Deviation from the center line
c) detection of exudate
d) signs of muscle weakness
A
b) Deviation from the center line
Explanation:
During the physical examination, the caregiver should inspect and gently palpate the trachea to assess its location and deviation from the midline. The trachea is usually in the midline as it enters the thoracic inlet behind the sternum, but may be deviated by masses in the neck or mediastinum. Lung diseases such as pneumothorax or pleural effusion can also displace the trachea. The nurse examines the posterior pharynx and tonsils with a tongue depressor and light, noting any signs of swelling, inflammation, or oozing and changes in the color of the mucous membranes. The nurse also examines the anterior, posterior, and lateral chest walls for signs of muscle weakness.
49
q
The nurse notes round, papular, red lesions on the patient's back that blanch with light pressure. Which of the following actions is the appropriate caregiver action?
a) Inform the doctor.
b) Document findings
c) Apply protective cream.
d) Rotate and reposition the patient.
A
b) Document findings
Explanation:
Lesions that are red, papular, and round on a patient's trunk that turn whitish under pressure are typically cherry angiomas. Since this lesion is not clinically significant, the appropriate nursing action is to document the findings.
51
q
The nurse attends to a patient who complains of chest discomfort. The patient's admission diagnosis was left lower lobe pneumonia. Which of the following strategies will the nurse instruct the patient to use to alleviate the symptoms?
a) Perform deep breathing exercises when chest discomfort occurs
b) Request anesthesia if pain occurs
c) Lying on the right side
d) Lie on the left side of the bed.
A
c) Lying on the right side
Explanation:
Pleuritic pain from irritation of the parietal pleura is sharp and seems to "take" on inspiration; Patients often describe this as "like stabbing with a knife." Patients are more comfortable lying on the affected side because it immobilizes the chest wall, limits expansion and contraction of the lungs, and reduces friction between the injured or diseased pleura on that side. The pain associated with coughing can be manually reduced with immobilization of the chest. The nurse would instruct the patient to lie on the left side, not the right side, to reduce pain. Although pain relievers can be administered, non-drug therapies and non-narcotic interventions must first be administered. Breathing exercises would not help relieve pain but would slow the patient's breathing and expand the lungs.
51
q
The nurse attends to a patient with allergic rhinitis. The patient asks the nurse about measures to alleviate allergic symptoms. Which of the following is the best response from the nurse?
a) “You should try to reduce exposure to irritants and allergens.”
b) "Make sure you get a flu shot every year."
c) "You should visit your otolaryngologist every month."
d) "Take over-the-counter (OTC) nasal congestion if you experience symptoms."
A
a) “You should try to reduce exposure to irritants and allergens.”
Explanation:
The caregiver instructs the patient with allergic rhinitis to avoid or reduce exposure to allergens and irritants such as dust, mold, animals, smoke, odors, dusts, aerosols, and tobacco smoke. Annual influenza vaccination is recommended for patients with infectious rhinitis. To avoid potential drug interactions, the patient is advised to read drug labels before taking over-the-counter medications. Patients with nasal septal deformities or nasal polyps may be referred to an ear, nose, and throat specialist.
51
q
A patient with infective endocarditis (IE) and fever is admitted to the intensive care unit (ICU). Which of these doctor's orders should the nurse implement first?
a) Create a transesophageal echocardiogram.
b) Request blood cultures from two sites.
c) Administer acetaminophen (Tylenol) pro re nata (PRN) for fever greater than 48.3 degrees.
d) Administer ceftriaxone (Rocephin) 1 g IVPB every 12 hours.
A
b) Request blood cultures from two sites.
Explanation:
Blood cultures (each set comprises one aerobic and one anaerobic culture) should be collected from different venipuncture sites over a 24-hour period (each set should be at least 12 hours apart) or every 30 minutes if the patient's condition is unstable before of antimicrobial administration. It is important to obtain blood cultures prior to initiating antibiotic therapy to obtain accurate sensitivity results.
52
q
A patient is brought into the operating room for elective surgery. What is the current priority action?
a) Acquisition of requested blood products.
b) Obtain a sponge and syringe count.
c) Verify consent.
d) Document the start of the operation.
A
c) Verify consent.
Explanation:
Surgery without consent is not allowed. The start of the operation can only be documented after the start of the operation. Blood products must be administered within a specific time period and therefore should be purchased only when necessary. The count of sponges and syringes is a safety issue that must be carried out prior to surgery and while the wound is sutured, but the patient does not give his consent, surgery must not be performed.
53
q
A nurse reviews discharge instructions with a patient who underwent a cardiac catheterization in the left groin 8 hours ago. Which of the following should the caregiver include?
a) “Do not bend at the waist, strain, or lift heavy objects for the next 24 hours.”
b) "You can shower or shower when you get home."
c) "If a discharge occurs at the puncture site, call 911 immediately."
d) "Contact your GP if you develop a temperature above 102°F."
A
a) “Do not bend at the waist, strain, or lift heavy objects for the next 24 hours.”
Explanation:
The nurse should instruct the patient to do the following: if the groin artery was used, not bend at the waist, strain, or lift heavy objects for the next 24 hours; Contact first aid personnel if any of the following occur: swelling, new bruising, or pain at the procedure puncture site, temperature of 101°F or higher. If bleeding occurs, lie down (in the groin area) and apply firm pressure to the puncture site for 10 minutes. Notify the primary provider as soon as possible and follow their instructions. If the bleeding is profuse, call 911. The patient should not drive to the hospital by car.
55
q
Which of the following findings indicates that hypertension progresses to end-organ damage?
a) Urine production of 60 cc/ml in 2 hours
b) Chest X-ray showing pneumonia
c) blood urea nitrogen (BUN) level of 12 mg/dl
d) damage to the retinal blood vessels
A
d) damage to the retinal blood vessels
Explanation:
Symptoms that indicate that hypertension is progressing to the point of target organ damage must be identified early so that appropriate treatment can be initiated. All body systems should be examined for evidence of vascular damage. An ocular examination with an ophthalmoscope is important because damage to the retinal blood vessels indicates similar damage to other parts of the vasculature. The patient is asked about blurred vision, spots before the eyes, and decreased visual acuity. The heart, nervous system, and kidneys are also carefully examined. BUN and 60 cc/mL in 2 hours are normal findings. The presence of pneumonia does not indicate end-organ injury.
55
q
A postoperative patient begins to cough violently from eating Jell-O. The nurse notices an evisceration of the intestines. What should the caregiver do first?
a) Notify the surgeon.
b) Cover the intestine with sterile, moist dressings.
c) Place the patient in deep Fowler's position.
d) Document the event.
A
c) Place the patient in deep Fowler's position.
Explanation:
Placing the patient in the deep Fowler position further reduces bulging of the viscera. The nurse should cover the intestine with a sterile, moist bandage; notify the surgeon and document the event; but above all, the caregiver must minimize further protrusion of the intestines.
57
q
A patient undergoes perineal surgery. Which of the following caregiver actions is appropriate?
a) Place the patient in the Sims position.
b) Place the patient in the supine position.
c) Place the patient in the lithotomy position.
d) Place the patient in the Trendelenburg position.
A
c) Place the patient in the lithotomy position.
Explanation:
The lithotomy position is used in almost all perineal, rectal, and vaginal surgeries. The Trendelenburg position is typically used for operations on the lower abdomen and pelvis. The Sims or lateral position is used for kidney surgery. The supine position is the usual position for surgical procedures.
57
q
The nurse performs an examination of an immobilized patient. What assessment leads the nurse to act?
1. Heart rate 86
2. Red area on the sacrum
3. Unproductive cough
4. Urinary output of 50 ml/hour
A
2. Red area on the sacrum
Reason: Reddened skin area may cause skin damage. The other options are within normal limits.
58
q
The caregiver understands that a general goal of hypertension control includes:
a) There are no complaints of sexual dysfunction.
b) There is no evidence of target organ damage.
c) There are no complaints of postural hypotension.
d) The patient maintains a normal blood pressure value.
A
b) There is no evidence of target organ damage.
Explanation:
Prolonged increases in blood pressure gradually damage blood vessels throughout the body, especially in target organs such as the heart, kidneys, brain, and eyes. The general objective of the treatment is that the patient does not suffer damage to the target organs. The desired effects of antihypertensive drugs are to maintain normal blood pressure. Orthostatic hypotension and sexual dysfunction are side effects of certain antihypertensive drugs.
59
q
Which of the following oxygen supply devices has the advantage of delivering a high concentration of oxygen?
a) Mask without rebreather
b) Venturi mask
c) Catheter
d) face shop
A
a) Mask without rebreather
Explanation:
The non-rebreathing mask provides a high concentration of oxygen, but often fits poorly. However, if the non-rebreathing mask fits snugly on the patient and both side ports have one-way valves, it is possible for the patient to receive 100% oxygen, making the non-rebreathing mask a high-pressure oxygen system. flow. The Venturi mask provides small amounts of supplemental oxygen. The catheter is an inexpensive device that delivers a variable rate of inspired oxygen and may cause distension of the stomach. A front tent provides a fairly accurate ratio of inhaled oxygen, but it is bulky and uncomfortable. It would not be the device of choice for delivering a high concentration of oxygen.
60
q
The nurse assesses the cutaneous system of a patient with Cushing's syndrome. Which of the following findings does the nurse expect?
a) hyperpigmentation
b) jaundice
(c) Hirsutism
d) Alopecia
A
(c) Hirsutism
Explanation:
The nurse expects to find hirsutism, or excessive hair growth, since Cushing's syndrome causes hirsutism, especially in women. Alopecia, jaundice, and hyperpigmentation are not typical findings in patients with Cushing's syndrome.
61
q
A patient presents to the emergency department with an exacerbation of left ventricular failure and complains of shortness of breath. Which of the following is the primary nursing intervention?
a) Hemmer verabreichen angiotensin converting enzyme
b) administration of angiotensin II receptor blockers
c) Evaluate the level of oxygen saturation
d) administer diuretics
A
c) Evaluate the level of oxygen saturation
Explanation:
Evaluation is a priority to determine the severity of the exacerbation. It is important to assess the oxygen saturation of a patient with heart failure because oxygen saturation levels below normal can be fatal. Treatment options vary depending on the severity of the patient's condition and may include supplemental oxygen, oral and intravenous medications, major lifestyle changes, cardiac device implantation, and surgical approaches. The overall goal of heart failure treatment is to relieve the patient's symptoms and decrease the workload on the heart by reducing afterload and preload.
62
q
A patient with a myocardial infarction develops acute mitral regurgitation. Does the nurse know which of the following manifestations to evaluate, indicating that the patient is developing pulmonary congestion?
a) Tachycardia
b) high blood pressure
c) shortness of breath
d) A loud murmur and thump
A
c) shortness of breath
Explanation:
Chronic mitral regurgitation is usually asymptomatic, but acute mitral regurgitation (eg, secondary to myocardial infarction) often manifests as severe congestive heart failure. Dyspnea, fatigue, and weakness are the most common symptoms. Palpitations, shortness of breath on exertion, and cough from lung congestion also occur. A loud murmur is often heard at the apex of the heart during ventricular systole. Hypertension can develop when reduced cardiac output triggers the renin-angiotensin-aldosterone cycle. Tachycardia is a compensatory mechanism when stroke volume decreases.
62
q
The PACU nurse comments that the patient is
Shake off. This is most commonly caused by:
1. Cold water washes during surgery
2. Side effects of certain anesthetics
3. Malignant hypothermia, a serious disease
4. The use of a reflective roof on the farm.
room table
A
Side effects of certain anesthetics
It is not always a sign of hypothermia, but rather a
Side effect of certain anesthetics
63
q
The nurse instructs the patient on how to apply corticosteroid cream to lesions on the arm. What intervention can the caregiver indicate to the patient to increase the absorption of the drug?
- After application, place an occlusive dressing over the area.
- Make sure the skin is slightly hydrated to allow the medicine to absorb through the cracks in the skin.
- Apply a thick layer of cream to the lesions so that if anything comes out, more of it will be absorbed.
- Apply the medicine every 2 hours.
A
After application, apply an occlusion bandage over the area.
64
q
A patient comes to the emergency room with chest pain. Patient arrangements include the following items. What job should the nurse do first?
a) Troponinspiegel
b) 2 liter nasal oxygen cannula
c) 12 channel ECG
d) Aspirina 325 mg oral
A
c) 12 channel ECG
Explanation:
The caregiver must first do the 12-lead ECG. The priority is to determine if the patient is suffering from an acute myocardial infarction and to initiate appropriate interventions as soon as possible. The remaining orders must be completed after the ECG.
Sixty-five
q
What gerontological alterations of the respiratory system should the nurse look for when examining the elderly patient?
- Decreased diameter of the alveolar duct
- Increased presence of mucus.
- Decreased gag reflex
- Increased presence of collagen in the alveolar walls
- Decreased presence of mucus.
A
Decreased gag reflex, increased presence of collagen in the alveolar walls, decreased presence of mucus
66
q
The nurse gives a patient a TEN. What evaluation data suggest that the patient may be progressing to keratoconjunctivitis (select all that apply)
- Flaking skin on the eyelids
- itchy eyes
- Irritated eyes
- dry eyes
- erased optical discs
A
itchy eyes, burning eyes, dry eyes
67
q
Nurse teaches acne patient using isotretinoin (Accutane) therapy. Which of the following statements should the nurse make?
a) Contraceptives are not necessary during treatment.
b) It is teratogenic in humans.
c) Side effects are irreversible.
d) The patient should take vitamin A supplements.
A
b) It is teratogenic in humans.
Explanation:
Accutane is teratogenic in humans, which means it can have an adverse effect on the fetus, causing defects in the central nervous system and cardiovascular system and structural abnormalities of the face. Contraceptives are needed during treatment. The patient should not take vitamin A supplements while taking this medication. Side effects are reversible upon discontinuation of the drug.
69
q
A patient comes to the emergency room complaining of chest pain that worsens when he takes a deep breath and lies down. After ruling out a myocardial infarction, a nurse would check which of the following diagnoses?
a) rheumatic fever
b) pericarditis
c) Cardiomyopathy
d) Mitralklappenstenosis
A
b) pericarditis
Explanation:
The main symptom of pericarditis is pain, which is assessed by assessing the patient in different positions. The caregiver tries to determine if the pain is affected by respiratory movements while holding your breath or holding your breath; by flexion, extension, or rotation of the spine, including the neck; by movements of shoulders and arms; from cough; or swallowing. Recognizing the events that trigger or exacerbate pain can help make a diagnosis and differentiate pericarditis pain from myocardial infarction pain.
69
q
A patient asks about the purpose of withholding food and fluids before surgery. What is the appropriate caregiver response?
a) Prevents aspiration and respiratory complications.
b) Reduces the risk of high blood sugar levels and slow wound healing.
c) Decreases urine production so a catheter would not be necessary.
d) Prevents hyperhydration and hypertension.
A
a) Prevents aspiration and respiratory complications.
Explanation:
The main purpose of withholding food and fluids before surgery is to prevent aspiration, which can lead to respiratory complications. Preventing overhydration, reducing urine output, and lowering blood sugar levels are not the primary goals of withholding food and fluids before surgery.
70
q
Which of the following comfort techniques does a nurse teach a patient with pleurisy to help immobilize the chest wall?
a) Using a prescription pain reliever
b) Raise the head of the bed
c) Using a heat application
d) Roll to the affected side
A
d) Roll to the affected side
Explanation:
The nurse instructs the patient to immobilize the chest wall by turning the affected side to reduce stretch on the pleura and relieve pain.
71
q
Which of the following is a possible complication of low endotracheal (ET) tube cuff pressure?
a) Drucknekrose
b) tracheal hemorrhage
c) Aspiration, pneumonia
d) tracheal ischemia
A
c) Aspiration, pneumonia
Explanation:
Low cuff pressure can increase the risk of aspiration pneumonia. High cuff pressure can cause tracheal hemorrhage, ischemia, and pressure necrosis.
72
q
A nurse assists a patient diagnosed with unstable angina and receiving i.v. receive heparin. The patient is placed on bleeding precautions. Which of the following are part of the bleeding precautions?
a) Avoid subcutaneous injections (SQ)
b) Avoid using nail clippers
c) Use of electric toothbrush
d) avoid continuous BP monitoring
A
d) avoid continuous BP monitoring
Explanation:
The patient receiving heparin should take precautions for bleeding, which may include: applying pressure to the needle puncture site for a longer period of time than usual, avoiding intramuscular injections, avoiding tissue damage and bruising from trauma or constrictive devices (eg, automatic blood pressure cuff devices). SQ injections are allowed; A soft toothbrush should be used and nail clippers may be used by the patient, but with caution.
73
q
What actions should a public health nurse include when planning ways to reduce the incidence of rheumatic fever in the community?
a) Educate people in the community to seek medical treatment for strep throat.
b) Administer prophylactic antibiotics to people with a family history of rheumatic fever.
c) Educate people in the community about the importance of temperature control to detect infections.
d) Encourage vulnerable groups in the community to receive the strep vaccine.
A
a) Educate people in the community to seek medical treatment for strep throat.
Explanation:
Prevention of acute rheumatic fever depends on effective antibiotic treatment of strep throat. Family history is not a risk factor for rheumatic fever. There is no vaccine that is effective in reducing the incidence of rheumatic fever. Temperature control education will not reduce the incidence of rheumatic fever
75
q
Apt complains of intense itching, worse at night. The nurse decides to examine her skin with a magnifying glass and flashlight to look for the "itch mite." What skin condition does the caregiver expect?
- contact dermatitis
- pediculosis
- sarna
- ringworm of the body
A
sarna
76
q
Why is a customer with a fever often prone to pressure sores?
1. The perception of pain is reduced.
2. Drugs given to relieve fever cause edema.
3. Client may be too weak to change position.
4. Increased metabolism causes a higher demand for oxygen that cannot be met.
A
4. Increased metabolism causes increased oxygen demand that cannot be met; An increase in metabolism causes a higher demand for oxygen that cannot be met; Therefore, a customer with a fever is prone to pressure sores. Answers 1 and 2 are false statements. Answer 3 can be a cause of pressure sores and can occur in clients with a fever, but is not directly related.
77
q
Black sores are treated with debridement. What type of debridement is the most selective and least damaging?
1. Debridement with scissors
2. Debridement with wet to dry dressings
3. Mechanical debridement
4. Chemical debridement
A
4. Chemical debridement; Chemical debridement is performed with enzymatic agents or autolytic agents. Answer 1 is a kind of acute debridement. Answers 2 and 3 are mechanical and less precise than the chemical ones.
78
q
A nurse examines a patient at risk of developing Acute Respiratory Distress Syndrome (ARDS) after major surgery. What early and most common sign of ARDS will the caregiver look for?
a) sibilancia bilateral
b) Rapid onset of severe dyspnea
c) Inhalation of cracklings
d) Zianosis
A
b) Rapid onset of severe dyspnea
Explanation:
The acute phase of ARDS is characterized by a rapid onset of severe dyspnea, usually occurring less than 72 hours after the initial event.
79
q
The nurse educates the patient on the correct use of concentrated topical corticosteroids. Which of the following qualities belong to the nurse? Select all that apply.
a) Avoid prolonged use.
b) Hypertrichosis is normal.
c) Apply on the face.
d) Apply in intertriginous areas.
A
a) Avoid prolonged use. c) Apply on the face.
Explanation:
Clinicians should instruct the patient to avoid prolonged use which can lead to hypertrichosis (excessive hair growth) and/or steroid-induced acne. The caregiver should also advise the patient not to apply the corticosteroid to the face and intertriginous areas.
80
q
The nurse administered the preanesthetic medication. What action should the caregiver take next?
a) Inform the patient about the discharge instructions.
b) Consult the list of medications at the patient's home.
c) Obtain the patient's signature on the informed consent form.
d) Place the patient on bed rest with the side rails elevated.
A
d) Place the patient on bed rest with the side rails elevated.
Explanation:
Pre-anesthetic medication can make the patient feel dizzy and dizzy. Safety is priority. The informed consent form must be signed before the patient is medicated. Consents signed after the patient has been treated with medication are not legal. Home medication review and patient education should be performed prior to medicating the patient.
81
q
The nurse attends to a patient who is due to undergo a bronchoscopy. The nurse understands that it is important to provide the patient with respiratory disease with the necessary information and adequate explanation for each diagnostic procedure. Which of the following actions should he perform?
a) Support the patient's caregivers
b) Provide adequate rest periods
c) Coping with declining energy levels
d) control of shortness of breath
A
c) Coping with declining energy levels
Explanation:
In addition to administering individual nursing tests, patients with respiratory diseases also need a meaningful and appropriate explanation of all the diagnostic procedures they will undergo. Caregivers should be aware that many of these patients may have some degree of shortness of breath and reduced energy levels. For this reason, explanations should be short but complete and may need to be repeated after a rest period. The caregiver must also ensure adequate rest periods before and after procedures. After invasive procedures, the caregiver should carefully monitor for signs of difficulty breathing.
81
q
What caregiver actions best cover the preoperative period?
a) Educate patients about the signs and symptoms of infection.
b) Documentation of the use of Sequential Compression Devices (SCD)
c) Shaving the patient with a razor
d) Monitoring of vital signs every 15 minutes
A
a) Educate patients about the signs and symptoms of infection.
Explanation:
The patient is informed about the prevention or detection of complications already in the preoperative period. Application of the SCD and frequent monitoring of vital signs occur after the preoperative period. Only electric scissors should be used for hair removal.
83
q
The nurse runs a hypertension service project for a local senior community. Which of the following are among the risk factors and cardiovascular problems associated with high blood pressure? Select all that apply.
a) Adiposity (BMI ≥ 30 kg/m2)
b) Age ≥55 in men
c) Decreased levels of low-density lipoproteins (LDL).
d) Elevated high-density lipoprotein (HDL) cholesterol
e) smoke
A
a) obesity (BMI ≥ 30 kg/m2), b) age ≥ 55 in men, e) smoking
Explanation:
The main risk factors (in addition to hypotension) are smoking, dyslipidemia (high LDL cholesterol, low HDL cholesterol), diabetes mellitus, renal failure, obesity, sedentary lifestyle, age (over 55 years for men, 65 years for women ) and a family history of circulatory heart disease.
85
q
Choose the statements that correctly correspond to the high blood pressure medication with its side effects. Select all that apply.
a) Check for bradycardia with ACE inhibitors.
b) Monitor serum potassium for thiazide diuretics.
c) Direct vasodilators can cause headache and tachycardia.
d) Coughing is a common side effect of adrenergic inhibitors.
e) Beta-blockers can cause sedation.
A
b) Monitor serum potassium for thiazide diuretics. c) Direct vasodilators can cause headache and tachycardia.
Explanation:
Thiazide diuretics can reduce potassium; Many clients require potassium supplements. Angiotensin-converting enzyme (ACE) inhibitors can cause mild to severe dry cough. Beta blockers can cause a slow heart rate; Pulse rate should be assessedb
86
q
The nurse distinguishes between a macule and a papule when evaluating a patient's skin lesion. The nurse determines that the lesion is a papule when which of the following are true?
a) Greater than 1 cm in diameter
b) smooth skin color change
c) Sublime and detailed
d) Circumscribed limit
A
c) Sublime and detailed
Explanation:
The nurse determines that the lesion is a papule rather than a macule by noting that the lesion is elevated and palpable. Macules are flat, non-palpable changes in skin color. Both the spots and the papules have circumscribed borders. The spots are less than 1 cm in diameter and the papules are less than 0.5 cm in diameter.
87
q
The nurse must collect a sputum sample to identify the presence of tuberculosis (TB). What care(s) is(are) indicated for this type of sample? Select all that apply.
1. Collect the sample at night.
2. Send the sample to the laboratory immediately.
3. Ask the client to spit into the sputum container.
4. Mouth care before and after collecting the sputum sample.
5. Collect a sample on 3 consecutive days.
A
2. Send the sample to the laboratory immediately.
4. Mouth care before and after collecting the sputum sample.
5. Collect a sample on 3 consecutive days.
Rationale: The sputum sample should be sent to the laboratory immediately. The client must receive oral care before and after sample collection. The sputum sample must be collected on three consecutive days. Option 1 is incorrect because the sputum sample is collected in the morning and not at night. Option 3 is incorrect because the term saliva indicates that saliva is being examined. The client should cough up or expectorate phlegm or sputum.
88
q
A nurse attends to a patient after a thoracentesis. Which of the following signs, if observed in the patient, should be reported to the physician immediately?
a) "The patient becomes restless and complains of pleuritic pain."
b) "The patient has subcutaneous emphysema around the needle insertion site."
c) "The patient has an oxygen saturation of 93%."
d) "The patient is sleepy and complains of a headache."
A
c) "The patient becomes restless and complains of pleuritic pain."
Explanation:
After a thoracentesis, the nurse monitors the patient for pneumothorax or recurrence of the pleural effusion. The signs and symptoms associated with a pneumothorax depend on its size and cause. The pain is usually sudden and may be pleuritic. The patient may have minimal shortness of breath with mild chest discomfort and tachypnea with a simple or uncomplicated small pneumothorax. As the pneumothorax increases, the patient may become anxious and develop dyspnea as accessory muscles become more activated.
88
q
Melanin production is controlled by a hormone secreted by which of the following glands?
a) parathyroid gland
(b) Hypothalamus
c) adrenal gland
d) thyroids
A
(b) Hypothalamus
Explanation:
Melanin production is controlled by a hormone secreted by the brain's hypothalamus called melanocyte-stimulating hormone. Melanin production is not controlled by the thyroid, adrenal, or parathyroid glands.
89
q
The nurse assists a hospitalized patient with a diagnosis of bacterial pharyngitis. The nurse assumes that the patient will receive which of the following medications?
a) Tylenol with codeine
b) Robitussina DM
c) Tylenol
d) Penicillin
A
d) Penicillin
Explanation:
The treatment of choice for bacterial pharyngitis is penicillin. Taking penicillin V potassium for 5 days is the regimen of choice. Traditionally, penicillin is given as a single injection; However, oral forms are more commonly used and are just as effective and less painful than injections. Penicillin injections are recommended only if there is concern that the patient will not comply with therapy. Robitussin DM can be used as a cough suppressant. For severe sore throats, aspirin or Tylenol or Tylenol with codeine may be given.
90
q
A nurse attends to a patient who has undergone balloon aortic valvuloplasty. For which of the following complications would the nurse closely examine the surgical site?
(a) expand
b) bleeding and dehiscence of the wound
c) thrombosis and infection
d) bleeding and infection
A
d) bleeding and infection
Explanation:
Possible complications of balloon aortic valvuloplasty include aortic regurgitation, embolism, ventricular perforation, annulus rupture, ventricular arrhythmia, mitral valve injury, infection, and bleeding at catheter insertion sites.
91
q
A patient presents to the emergency room complaining of anxiety and chest pains after shoveling heavy snow that morning. The patient says he hasn't taken nitroglycerin in months, but he took three nitroglycerin tablets, and although the pain is mild, "they didn't work very well." The patient shows the bottle of nitroglycerin to the nurse and the prescription was filled 12 months ago. Which of the following medical orders is the nurse waiting for?
a) chest x-ray
b) Ativan 1 mg oral
c) Serum electrolyte
d) Nitroglycerina SL
A
d) Nitroglycerina SL
Explanation:
Nitroglycerin is volatile and is inactivated by heat, moisture, air, light, and time. Nitroglycerin must be renewed every 6 months to ensure its full effectiveness. The patient's pills had expired and the nurse would have to wait for nitroglycerin to be administered to see if the chest pain subsided. The other options can be combined later, but the priority is to relieve the patient's chest pain.
93
q
Under which of the following conditions does high or increased adherence occur?
a) Neumothorax
b) Pleuraerguss
(c) SDRA (atemnot acute syndrome)
d) Epilepsy
A
d) Epilepsy
Explanation:
High or increased compliance occurs when the lungs lose elasticity and the chest stretches too far, as in emphysema. Conditions associated with decreased compliance include pneumothorax, pleural effusion, and ARDS.
95
q
A
96
q
Evidence-based practice is defined as:
A. Tradition-Based Care
B. Literature on scientific research in nursing and biomedical
C. A problem-solving approach that integrates current best knowledge into clinical practice Correct
D. Quality nursing care delivered in an efficient and commercially reasonable manner
A
C. A problem-solving approach that integrates current best knowledge into clinical practice Correct
Evidence-based practice integrates the best current evidence with clinical experience and patient/family preferences and values for the delivery of optimal medical care.
97
q
A client has an infected wound. What local human response should the keeper expect to see?
A
Edema; Chemical mediators increase the permeability of small blood vessels, causing fluid to enter the interstitial compartment, resulting in localized edema.
98
q
A patient comes to the ER complaining of stabbing pain when taking a deep breath. What is this type of pain likely to tell the nurse?
- bacterial pneumonia
- carcinoma bronchogenico
- pulmonary infarction
- pleuresía
A
pleuresía
100
q
A nurse cares for a patient with an acute myocardial infarction (STEMI). The nurse assumes that the doctor will prescribe Alteplase (Ativase). Which of the following are the most important questions a caregiver should ask a patient before administering this medication?
a) "How many sublingual nitroglycerin tablets have you taken?"
b) "At what time did your chest pain start today?"
c) "How severe is your pain on a scale of 1 to 10?"
d) "Do your parents have a history of heart disease?"
A
b) "At what time did your chest pain start today?"
Explanation:
The patient may be a candidate for thrombolytic (fibrolytic) therapy. These medications are given when the patient's chest pain lasts more than 20 minutes and is not relieved by nitroglycerin, ST-segment elevation in at least 2 leads targeting the same area of the heart less than 6 hours after pain onset. The most appropriate question for nurses refers to the appearance of chest pain. The other questions would not help determine if the patient is a candidate for thrombolytic therapy.
101
q
One patient underwent a 12-lead ECG as part of an annual physical examination. The nurse notices an abnormal Q wave on a normal electrocardiogram. Does the nurse acknowledge that this finding indicates this?
a) Variant angina
b) An IM in development
c) A past MI
d) Cardiac arrhythmia
A
c) A past MI
Explanation:
An abnormal Q wave may be present with no ST segment and T wave changes, indicating a non-acute old infarct
101
q
The nurse works seriously with parents.
sick newborn. Surgery was suggested for this
baby, but the chances of success are unclear. in the
help parents resolve this ethical conflict that
The nurse knows that the first step is:
1. Explore appropriate courses of action
2. Identify the people who can solve the difficulty
3. Clarification of amounts related to the cause of the
Dilemma
4. Gather all available information about the
Lage.
A
4. Gather all available information about the
lage..
Include as much information as possible
from a variety of sources such as laboratory and
Test results; the clinical condition of the patient; currently
literature on the condition; and the patient's
religious, cultural and family situation.
101
q
What is the proper technique for performing a wound culture?
1. Cleaning the wound prior to sample collection.
2. Dry the sample in the area of greatest drainage accumulation.
3. Removal of scabs or scabs with sterile forceps and then culture of the underlying site.
4. Wait 8 hours after the antibiotic dose to obtain the sample.
A
1. Cleaning the wound before sampling; Wound culture samples should be collected from a clean area of the wound. The microbes responsible for the infection are more likely to be found in viable tissue.
The collected wastewater contains ancient and mixed organisms. An adequate sample can be obtained without the client bothering the debridement. The caregiver does not usually debride a wound to obtain a sample. Once systemic antibiotics are started, the post-dose interval will not significantly affect the concentration of organisms in the wound.
102
q
A nurse cares for a patient who has been prescribed oral warfarin (Coumadin). The nurse checks the patient's prothrombin time (PT) to assess the effectiveness of the medication. Which of the following laboratory values should the nurse also assess?
a) International Normalization Ratio (INR)
b) Partial thromboplastic time (PTT)
c) sodium
d) Complete blood count (CBC)
A
a) International Normalization Ratio (INR)
Explanation:
The INR, reported with the PT, provides a standard method for reporting PT values and eliminates variation in PT results from different laboratories. The INR is used instead of the PT only to monitor the effectiveness of warfarin. The therapeutic range for the INR is 2 to 3.5, although the specific ranges vary depending on the diagnosis. The other laboratory values are not used to assess the effectiveness of Coumadin.
103
q
The nurse cares for a patient in the intensive care unit who is on mechanical ventilation. Which of the following nursing interventions are performed to reduce the patient's risk of developing ventilator-associated pneumonia (VAP)?
a) Keep the patient in Fowler's high position
b) Daily cleaning of the patient's mouth with chlorhexidine
c) Rotate and reposition the patient every 4 hours
d) Ensure that the patient remains sedated during intubation.
A
b) Daily cleaning of the patient's mouth with chlorhexidine
Explanation:
The five key elements of the VAP package include the following: elevation of the head of bed (30 to 45 degrees: half-Fowler position), daily "sedation vacation," and assessment of readiness for extubation (see below); gastric ulcer prophylaxis (with histamine-2 receptor antagonists such as ranitidine [Zantac]); deep vein thrombosis (DVT) prophylaxis; and daily oral hygiene with chlorhexidine (0.12% mouthwash). The patient should be rotated and repositioned every 2 hours to avoid complications of immobility and atelectasis and to optimize lung expansion.
104
q
What actions during surgery require immediate intervention by the circulating caregiver?
a) The surgeon arrives at the sterile field to obtain the equipment
b) The operating room nurse calls the blood bank to obtain blood products
c) The anesthesiologist monitors blood gas values
d) First aid of the graduate nurse suturing the surgical wound
A
b) The operating room nurse calls the blood bank to obtain blood products
Explanation:
The scrub nurse is "washed down" and should only come into contact with sterile equipment. Using the telephone to call the blood bank is the responsibility of the home nurse and would compromise the sterility of the operating room nurse. The surgeon "rubbed" and should only touch sterile fields. The anesthetist should monitor blood gas levels as needed, and it is appropriate for the first responder to suture the surgical wound.
106
q
Which of the following vitamin deficiencies would the nurse identify to prevent the complication of bleeding during surgery?
a) magnesium
b) Five
c) Vitamin K
d) Vitamin A
A
c) Vitamin K
Explanation:
Vitamin K is important for normal blood clotting. A deficiency in vitamin A and zinc would damage the immune system, while a deficiency in magnesium would delay wound healing.
107
q
A nurse examines a patient admitted with infective endocarditis. Which of the following statements would the nurse expect?
a) Involuntary muscle movements of the extremities
b) Raised red rash on the trunk and face
c) Bruises on the palms and soles of the feet
d) Small painful lesions on the tips of the fingers and toes
A
d) Small painful lesions on the tips of the fingers and toes
Explanation:
The primary symptoms of infective endocarditis are fever and heart murmur. In addition, small painful nodules (Osler's nodes) may appear on the pads of the fingers or toes.
108
q
The nurse cares for a patient with herpes zoster. The nurse describes the injuries in the patient's chart as which of the following?
a) pustules
b) Gallbladder
c) papules
(d) cysts
A
b) Gallbladder
Explanation:
Herpes zoster lesions are vesicles, defined as circumscribed, elevated, palpable lesions containing serous fluid and measuring less than 0.5 cm in diameter. Papules are raised masses with transient, irregular borders. Pustules are pus-filled lesions. Cysts are encapsulated, fluid-filled, or semisolid masses in the subcutaneous tissue or dermis.
109
q
A nurse attends to a patient after a wedge resection. As the nurse examines the patient's chest drainage system, a constant bubbling is observed in the airtight chamber. Which of the following problems does this finding indicate?
a) Expanding pneumothorax
b) air leak
c) mareas
d) Greater drainage
A
b) air leak
Explanation:
The nurse must be alert for air leaks in the drainage system; they are indicated by constant bubbling in the sealed chamber or by the air leak indicator on dry systems with a check valve. Tidal movement is a fluctuation in the water level in the water column that shows an effective connection between the pleural cavity and the drainage chamber and indicates that the drainage system remains open.
111
q
Does the nurse teach the patient with herpes zoster (shingles) about the following?
a) Once a patient has shingles, they will not get it a second time.
b) The infection results from a reactivation of the varicella virus.
c) No known medication affects the course of herpes zoster.
d) A person who has had chickenpox can become infected again if exposed to a person with shingles.
A
b) The infection results from a reactivation of the varicella virus.
Explanation:
Herpes zoster is thought to represent a reactivation of the latent varicella (varicella) virus and reflects reduced immunity. Varicella-zoster virus is thought to lie dormant in nerve cells near the brain and spinal cord and is reactivated in immunosuppressed conditions and cancer. A person who has had chickenpox is immune and therefore not at risk of contracting it after being exposed to patients with shingles. Some evidence suggests that if oral antivirals are given within 24 hours of the initial outbreak, the infection stops.
113
q
A nurse listens to a patient's lung sounds during a routine exam. The sounds produced are harsh and crackling, as if two pieces of leather were rubbing together. Would the nurse correctly document this finding as follows?
a) crackle
b) audible wheezing
c) wheezing whistle
d) Frictional atritus of the pleura
A
d) Frictional atritus of the pleura
Explanation:
Pleural rub is heard secondary to inflammation and loss of lubricating pleural fluid. Crackles are soft, sharp, discontinuous clicks that occur during inspiration. Audible wheezes are low-frequency rumbles heard primarily during expiration. Whistles are continuous, musical, high-pitched whistles heard during inhalation and exhalation.
115
q
Which of the following interventions does a nurse perform on patients with empyema?
a) Do not allow visitors with respiratory infections
b) Stimulate breathing exercises
c) Take precautions against droplets
d) Place suspected patients together
A
b) Stimulate breathing exercises
Explanation:
The nurse instructs the patient in lung expansion breathing exercises to restore normal respiratory function.
115
q
A patient admitted to the coronary care unit (CCU) with a STEMI is anxious and distressed. Which of the following medications will the nurse administer to relieve the patient's anxiety and reduce cardiac workload?
a) Norvasc (Amlodipine)
b) Tenormina (atenolol)
(c) IV-Morphine
d) Nitroglycerine IV
A
(c) IV-Morphine
Explanation:
IV morphine is the analgesic of choice to treat acute myocardial infarction. It is given to relieve pain and treat anxiety. It also reduces preload and afterload, which reduces the workload on the heart. IV nitroglycerin is given to relieve chest pain. The administration of Tenormin and Norvasc is not indicated in this situation.
115
q
A patient has contact dermatitis on the hand and the nurse notices a thick, rough area between the thumb and forefinger. How does the nurse know that this is important for repeated scratching and rubbing?
- Atrophy
- lichenification
- Keloid
- Scale
A
lichenification
116
q
The scope of nursing practice is legally defined by:
1. State Nursing Practice Laws
2. Nursing professional organizations
3. Hospital Policies and Procedures Manuals
4. Employed health professionals
institutions
A
1. State Nursing Practice Laws
Defines the legal boundaries within each state
117
q
A nurse prepares to assist a healthcare professional in removing a patient's chest tube. Which of the following instructions will the nurse correctly give to the patient?
a) "Do not move while removing the chest tube, as this will make removal more painful."
b) "Exhale vigorously as the chest tube is withdrawn."
c) "As the chest tube is removed, raise your arms above your head."
d) "When the tube is removed, take a deep breath in, exhale and push down."
A
d) "When the tube is removed, take a deep breath in, exhale and push down."
Explanation:
When assisting with chest tube removal, instruct the patient to perform a gentle Valsalva maneuver or to breathe calmly. The chest tube is then clamped and quickly withdrawn. Simultaneously a small bandage is applied and sealed with Vaseline gauze covered with 4×4 inch gauze and completely covered and sealed with non-porous tape. The other options are incorrect patient instructions.
118
q
A patient comes to the emergency room complaining of a severe coughing fit. The patient states that the “episodes are more intense at night”. Based on the patient's main complaint, which of the following should the nurse suspect?
a) Bronchitis
b) Left heart failure
c) emphysema
d) Chronic Obstructive Pulmonary Disease (COPD)
A
b) Left heart failure
Explanation:
Night cough may indicate the onset of left ventricular failure or bronchial asthma. A morning cough with sputum may indicate bronchitis. A cough that worsens when the patient lies on their back suggests postnasal drip (rhinosinusitis). Coughing after eating may indicate aspiration of material into the tracheobronchial tree. A recent cough is usually due to an acute infection. A cough that is more common at night is not associated with COPD, pulmonary emphysema, or bronchitis.
119
q
What is the cricoid cartilage in relation to the structure of the larynx?
a) the largest cartilage structure
b) The cartilaginous flap that covers the opening of the larynx during swallowing
c) Used in the movement of the vocal cords with the thyroid cartilage
d) The only complete ring of cartilage in the larynx.
A
d) The only complete ring of cartilage in the larynx.
Explanation:
The cricoid cartilage is the only complete ring of cartilage in the larynx (below the thyroid cartilage). The arytenoid cartilages are used in the movement of the vocal cords with the thyroid cartilage. The thyroid cartilage is the largest of the cartilaginous structures; part of it forms the Adam's apple. The epiglottis is the cartilaginous flap that covers the opening of the larynx during swallowing.
120
q
The nurse performs a preoperative assessment. The nurse notices that the patient is constantly crying and wringing his hands. The patient says: “I am very nervous about this operation. Do you think he's going to be okay?" What is the best response from the nurse?
a) “You don't have to worry; They have the best surgical team.”
b) "What family support do you have after the operation?"
c) "What are your concerns?"
d) "No one has ever died from the procedure you performed."
A
c) "What are your concerns?"
Explanation:
Asking the patient about his concerns is an open therapeutic technique. Allow the patient to lead the conversation and address her emotional state. Asking for family support changes the subject and is not therapeutic. Discussing the surgical team and the low mortality rate associated with a procedure downplays the patient's feelings and is not therapeutic.
121
q
Does target organ damage caused by untreated or undertreated hypertension include the following? Select all that apply.
a) Hiperlipidemia
b) cerebrovascular accident
c) Diabetes
d) damage to the retina
i) heart failure
A
b) stroke, d) retinal damage, e) heart failure
Explanation:
Target organ systems include the heart, the cerebrovascular system, the peripheral vascular system, the kidney, and the eye. Hyperlipidemia and diabetes are risk factors for the development of hypertension.
123
q
A nurse takes a medical history of a patient with a primary diagnosis of mitral stenosis. Which of the following patient-reported conditions is the most common cause of mitral stenosis?
a) heart failure
b) rheumatic endocarditis
c) atrial fibrillation
d) myocardial infarction
A
b) rheumatic endocarditis
Explanation:
Mitral stenosis is most often caused by rheumatic endocarditis, which progressively thickens the mitral valve leaflets and tendon sheaths. Ruffles often merge with each other. Eventually, the opening of the mitral valve narrows, further restricting the flow of blood into the ventricle.
123
q
The anesthesiologist performed a transacral line block. Which of the following nursing records is consistent with the anesthesia administered?
a) No movement in the lower part of the right leg
b) Denied sensations in the perineum and lower abdomen
c) Shouting and throwing devices
d) Does not respond to verbal or tactile stimuli
A
b) Denied sensations in the perineum and lower abdomen
Explanation:
A transacral block produces anesthesia of the perineum and sometimes the lower abdomen. Shouting and pulling of equipment may be related to the awakening phase of general anesthesia. Lack of response to verbal or tactile stimuli and lack of movement in the lower right leg are inconsistent with a transacral conduction block.
123
q
You are at the scene of an accident and discover that the victim has a bleeding wound on the lower leg. After rinsing the wound with water and covering it with a clean bandage, you notice that the bandage is saturated with blood. Which of the following would be the best course of action in this case?
1. Lower the limb while applying pressure to the wound.
2. Remove the first bandage and apply another clean or sterile bandage.
3. Place your hands around the client's ankle and apply pressure.
4. Reinforce the first layer of dressing with a second layer of dressing.
A
4. Reinforce the first layer of dressing with a second layer of dressing; To control excessive bleeding, apply direct pressure to the wound and elevate the extremity. When the dressing is saturated, apply a second coat. Removing the initial bandage can stop blood clots and increase bleeding.
124
q
A patient is on the third postoperative day for surgical correction of an open abdominal wound and traumatic amputation of the right leg after a motorcycle accident. What is the highest priority nursing intervention?
a) Evaluation of the white blood cell count, temperature and appearance of the wound
b) Educate the patient on safe transfer procedures from bed to chair.
c) Administration of analgesics within 1 hour at the request of the patient
d) Obtain nutritional advice to improve wound healing
A
a) Evaluation of the white blood cell count, temperature and appearance of the wound
Explanation:
The patient is at higher risk of infection associated with a surgical wound classified as dirty. Evaluation of the white blood cell count, temperature, and appearance of the wound allows the caregiver to intervene at the first sign of infection. The patient will have special nutritional needs for wound healing and will require education on safe transfer procedures, but the need to monitor for infection is a higher priority. The patient should receive pain medication as soon as possible upon request, but recent literature suggests that pain medication should be given on a schedule rather than "as needed."
125
q
An asymptomatic patient questions the nurse about the diagnosis of mitral regurgitation and asks about continuing an exercise routine. Which of the following is the most appropriate nursing response?
a) Continue with the exercise routine unless you have symptoms such as shortness of breath or fatigue.
b) Avoid strenuous cardiovascular exercise.
c) Continue your exercise routine, but get plenty of rest after exercising.
d) Avoid any type of exercise.
Show response
A
127
q
Which of the following is called the ability of the heart muscle to shorten in response to an electrical impulse?
a) Repolarization
b) diastole
c) contractility
d) Depolarization
A
c) contractility
Explanation:
Contractility is the ability of the heart muscle to shorten in response to an electrical impulse. Depolarization is the electrical activation of a cell caused by sodium entering the cell while potassium leaves the cell. Repolarization is the return of the cell to a resting state caused by potassium re-entering the cell while sodium leaves the cell. Diastole is the period of ventricular relaxation that results in ventricular filling.
128
q
Which of the following is an age-related change associated with the respiratory system?
a) Decreased size of the airways
b) Decreased thickening of the alveolar membranes
c) Increased elasticity of the alveolar sacs.
d) Increased pectoral muscle mass
A
a) Decreased size of the airways
Explanation:
Age-related changes that occur in the respiratory system include a reduction in the size of the airways, a decrease in pectoral muscle mass, an increase in the thickening of the alveolar membranes, and a reduction in the elasticity of the alveolar sacs. .
128
q
Which of the following lifestyle modification guidelines for hypertension does the nurse teach the patient?
a) Make sure you eat enough fruits and vegetables.
b) Stop drinking alcohol.
c) Limit aerobic physical activity to 15 minutes three times a week
d) Reduce tobacco consumption to no more than four cigarettes a day.
A
a) Make sure you eat enough fruits and vegetables.
Explanation:
The guidelines include adopting the Dietary Approaches to Stop Hypertension (DASH) eating plan: Eat a diet rich in fruits, vegetables, and low-fat dairy products with reduced levels of saturated and total fat, Reduce dietary sodium: Reduce dietary sodium intake to zero or more than 100 mmol/day (2.4 g sodium or 6 g sodium chloride) and physical activity: Participate in regular aerobic physical activity, such as brisk walking (at least 30 min/day, most days of the week), Moderate Alcohol Consumption: Limit consumption to no more than two drinks (eg, 24 oz. of beer, 10 oz. of wine or 3 ounces of 80 proof whiskey) per day for most men and no more than 1 drink per day for women and lighter individuals. Tobacco – should be avoided as anyone with high blood pressure is already at increased risk of heart disease and smoking increases that risk.
129
q
Thirty minutes after application, the client asks the caregiver to leave the heating pad in place. The nurse explains the following to the client:
1. The application of heat for more than 30 minutes can produce the opposite effect (constriction) of the desired one (dilation).
2. It is acceptable to leave the pad in place for an additional 30 minutes.
A
1. The application of heat for more than 30 minutes can produce the opposite effect (constriction) of the desired one (dilation); Heating pads must be removed. After 30 minutes of heat application, the blood vessels in the region begin to rebound, resulting in vasoconstriction.
Lowering the temperature while adding heat, dry or wet, does not prevent the rebound effect. The visual appearance of the site after inspection (Option 3) does not indicate whether kickback is occurring.
131
q
The nurse educates a community group on the types of surgeries. A group member asks the nurse to describe a type of surgery that is curative. Which of the nurse's answers is correct?
a) A facelift
b) Placement of the gastrostomy tube
c) a biopsy
d) Excision of a tumor
A
d) Excision of a tumor
Explanation:
An example of a curative surgical procedure is the excision of a tumor. Biopsy, facelift, and gastrostomy tube insertion are not examples of curative surgical procedures.
132
q
A patient comes to the emergency room with a suspected allergic reaction. The patient has laryngeal edema causing obstruction and has cervical retractions during inspiration. Which of the following is the primary nursing intervention?
a) Prepare for endotracheal intubation with mechanical ventilation.
b) Prepare to administer epinephrine and subcutaneous corticosteroids.
c) Apply 100% oxygen through a face mask.
d) Prepare an immediate tracheotomy.
A
b) Prepare to administer epinephrine and subcutaneous corticosteroids.
Explanation:
The use of accessory muscles to maximize airflow, often manifested by constricting the throat during inspiration, is an ominous sign of impending breathlessness. The patient's obstruction is caused by edema secondary to an allergic reaction, and treatment should include immediate administration of subcutaneous epinephrine and a corticosteroid. The other interventions may be indicated in patients with laryngeal obstruction; However, in this case, drug administration is the most appropriate intervention to treat the patient's laryngeal edema.
133
q
When performing endotracheal suctioning, the caregiver suctions while gently withdrawing and rotating the catheter 360 degrees during which of the following periods?
a) 20 to 25 seconds
b) 30 to 35 seconds
c) 10 to 15 seconds
d) 0 to 5 seconds
A
c) 10 to 15 seconds
Explanation:
In general, the nurse should not suction for more than 10-15 seconds, as hypoxia and arrhythmia can occur, which can lead to cardiac arrest. Sucking for 30-35 seconds is dangerous and can lead to hypoxia in the patient, which can lead to arrhythmia and eventually cardiac arrest. Sucking for 20-25 seconds is dangerous and can lead to hypoxia in the patient, which can lead to arrhythmia and eventually cardiac arrest. A suction time of 0 to 5 seconds would not provide sufficient time for effective suction of secretions.
134
q
The nurse understands that the patient's antihypertensive medication report should include all of the following except which of the following?
a) Avoid hot baths, exercise and alcohol within 3 hours of taking vasodilators.
b) Avoid over the counter cold, weight loss and sinus medicines.
c) If a dose of medication is forgotten, double the next one to make up for it.
d) Do not abruptly suspend antihypertensive medication.
A
c) If a dose of medication is forgotten, double the next one to make up for it.
Explanation:
Doubling of doses may cause severe hypotension (HBP) and is not recommended. Medications must be taken as prescribed. Hot baths, strenuous exercise, and excessive alcohol consumption are vasodilators and should be avoided. Many over-the-counter preparations can precipitate hypertension. Abrupt discontinuation of antihypertensive drugs may induce a severe hypertensive reaction and is not recommended.
135
q
What is the maximum volume of air that can be inspired after a maximum expiration?
a) Volume at rest
b) Expiratory reserve volume
c) Inspiratory reserve volume
d) Corresponding volume
A
c) Inspiratory reserve volume
Explanation:
The maximum volume of air that can be inspired after a normal inspiration is called the inspiratory reserve volume. The inspiratory reserve volume is usually 3000 mL. Tidal volume is the volume of air that is inhaled and exhaled with each breath. The expiratory reserve volume is the maximum volume of air that can be expelled after a normal expiration. Residual volume is the volume of air remaining in the lungs after maximal expiration.
136
q
Which of the following skin conditions is caused by staph, strep, or multiple bacteria?
(a) impetigo
b) sarna
c) lice
d) Giftefeu
A
(a) impetigo
Explanation:
Impetigo occurs at any age, but it is particularly common in children living in unsanitary conditions. Scabies is caused by the itch mite. Pediculosis capitis is caused by head lice. Poison ivy is a contact dermatitis caused by the oleoresin secreted by a certain form of ivy.
137
q
One patient underwent uneventful hernia surgery. Which of the following nursing actions is the most appropriate in the immediate postoperative period?
a) Monitor vital signs every 15 minutes
b) Assessment of pupillary response every 5 minutes
c) urine output measurement every 15 minutes
d) Obtaining arterial blood gases every 5 minutes
A
a) Monitor vital signs every 15 minutes
Explanation:
Pulse rate, blood pressure, and respiratory rate are recorded at least every 15 minutes for the first hour and every 30 minutes for the next 2 hours. Obtaining arterial blood gases every 5 minutes is painful for the patient unless a special device is inserted to obtain arterial blood samples. Without complications, it is not indicated for the patient. Urine output is monitored frequently, but is usually measured every hour. Although it may be necessary to assess pupillary response during the immediate postoperative period, it is not necessary to do so every 5 minutes.
137
q
All of the following measures are used for evaluation
venous thrombosisexcept:
1. Check for a positive Homans sign
2. Ask the patient about the presence of a calf
efforts
3. Observation of the dorsal aspect of the lower extremities
for redness, heat and sensitivity
4. Measure the circumference of each leg daily,
Place the tape measure in the middle of the
Knee
A
4. Measure the circumference of each leg daily and place the tape measure at the middle of the knee.
Calf circumference should be measured on both sides.
139
q
A nurse examines the skin around an African American man's wound for inflammation. The nurse will determine that inflammation is present when which of the following determinations is made?
a) Blue-green hue
b) White spots
c) red dye
d) Lila-Graustich
A
d) Lila-Graustich
Explanation:
The nurse will determine inflammation when a purple-gray discoloration is detected. Inflammation in fair-skinned people is characterized by erythema or reddening of the skin.
141
q
A patient with restrictive cardiomyopathy (RCM) is taking digoxin. Due to the risk of increased sensitivity, which of the following manifestations should the patient's caregiver carefully review?
a) tachypnea and dyspnea
b) anorexia and confusion
c) edema y ortopnea
d) abdominal pain and diarrhea
A
b) anorexia and confusion
Explanation:
Patients with RCM have increased sensitivity to digoxin and the caregiver should anticipate low-dose prescription and assess for digoxin toxicity. The most common manifestations of digoxin toxicity are gastrointestinal disorders (anorexia, nausea, and vomiting), cardiac disorders (arrhythmias, and heart block), and central nervous system (CNS) disorders (confusion, headache, weakness, dizziness, and blurred vision or yellow) . . 🇧🇷
143
q
The nurse takes a respiratory history from a patient who came to the clinic with a chronic cough. What information should the nurse get from the doctor? (Select all that apply)
- Financial ability to pay the bill
- social care
- Patient or family history of lung disease
- Occupational and environmental influences
- history of smoking
A
History of lung disease in the patient or family, occupational and environmental influences, history of smoking
144
q
A doctor will order a wound to be covered with a wet or damp gauze bandage. What should the nurse explain to the patient as the main reason for this type of dressing?
A
Wrapping the wound with moist-to-moist dressings allows epidermal cells to migrate through the wound surface bed more rapidly than with dry dressings, thus facilitating healing.
Moist dressings also direct exudate up and away from the wound bed, helping to increase the wound's resistance to infection. 🇧🇷
145
q
A patient hospitalized after a car accident suffered a stroke. A nurse examines the patient to determine the most common clinical manifestation of flail chest.
a) Zianosa
b) high blood pressure
c) Paradoxical movement of the thorax
d) whistle
A
c) Paradoxical movement of the thorax
Explanation:
As the ribcage expands during inspiration, the severed portion of the rib segment (unstable segment) moves paradoxically (air oscillation) as it contracts during inspiration, reducing the amount of air that can be expelled towards the lungs. As intrathoracic pressure exceeds atmospheric pressure, the unstable segment bulges out during expiration, affecting the patient's ability to exhale. The mediastinum then shifts to the affected side. This paradoxical effect leads to increased dead space, reduced alveolar ventilation, and decreased compliance.
147
q
A caregiver caring for a patient with pulmonary embolism understands that there may be a high ventilation/perfusion ratio. What does this mean for PT?
- Perfusion trumps ventilation.
- There is a lack of perfusion and ventilation.
- Ventilation exceeds blood flow.
- Ventilation corresponds to perfusion.
A
Ventilation exceeds blood flow.
148
q
A patient with a skull fracture after falling down a ladder requires surgery. How long should the caregiver wait to transport the patient to surgery?
a) immediately
b) A label 1
c) In 48-72 hours
d) In 1 week
A
a) immediately
Explanation:
Emergency surgeries occur when the patient needs immediate care. A skull fracture is an indication for emergency surgery. Urgent surgery is performed when the patient needs immediate attention, usually within 24 to 30 hours. Any surgery scheduled beyond 30 hours is classified as necessary or elective, and a skull fracture does not qualify for elective or mandatory surgery.
149
q
Nurse attends to a patient after a thoracotomy. Which of the following findings requires immediate caregiver intervention?
a) Moderate amounts of colorless sputum
b) Chest drainage of 190 ml/h
c) pain of 5 on a pain scale of 1 to 10
d) Heart rate: 112 bpm
A
b) Chest drainage of 190 ml/h
Explanation:
The caregiver should monitor and record the amount and type of drainage every 2 hours. The caregiver notifies the rescuer if the drainage is 150 mL/h or greater. Other findings are normal after thoracotomy; No intervention required.
150
q
A family doctor will perform a thoracentesis. What action will include the role of the nurse?
1. Place the patient supine in the Trendelenburg position.
2. Position client in a sitting position with elbows resting on the nightstand.
3. Instruct the UAP to measure vital signs.
4. Administer an opioid analgesic.
A
2. Position the patient in a sitting position with elbows on the nightstand.
Justification: The puncture site is usually in the posterior part of the thorax. The client should be positioned leaning forward. This allows the ribs to be separated to expose the site. Variant 1 is incorrect. The client should not be placed in the Trendelenburg position, as the site will not be exposed. Option 3 is incorrect because vital sign changes are not routine in this procedure. Option 4 is wrong. The patient does not need to be treated for pain with this procedure.
152
q
Which of the following sedatives is effective in treating pruritus?
a) Benzoyl peroxide
b) tetracycline
c) Hydroxyzine (Atarax)
d) Fexofenadine (Allegra)
A
c) Hydroxyzine (Atarax)
Explanation:
Atarax is a sedative drug that is effective in treating itching. Benzoyl peroxide, Allegra, and tetracycline are not effective in treating itching
153
q
The client is comfortably on the right or left side only (not on the back or stomach). List at least four possible pressure ulcer sites for caregiver assessment.
A
These are important areas to evaluate. Possible ulcer sites for clients lying on their side include:
1. ankle
2. knee
3. Trochanter
4. Ilia
5. Shoulders
6. Ears
154
q
Which of the following are primary risk factors for pressure ulcers? Select all that apply.
1. Low protein diet
2. Insomnia
3. Prolonged surgical procedures
4. Fever
5. Sleep on a waterbed
A
1, 3 and 4; Risk factors for pressure ulcers include a low-protein diet, lengthy surgical procedures, and fever.
Protein is necessary for proper skin health and healing. During surgery, the client is on a hard surface and may not be well protected from pressure on bony prominences. Fever increases skin moisture, which can damage the skin, and exposure of the body to the cause of the fever can affect circulation and skin integrity. Insomnia (option 2) would generally be associated with restless sleep, transferring pressure to different parts of the body and reducing the likelihood of skin damage. A waterbed (option 5) distributes pressure more evenly than a regular mattress, which actually reduces the chance of skin damage. 🇧🇷
156
q
A nurse attends to a patient in the ICU who required emergency endotracheal intubation (ET) with mechanical ventilation. The nurse will be instructed to obtain arterial blood gases (ABG) after the procedure. Does the nurse recognize that ABGs should be obtained at what point after starting mechanical ventilation?
a) 15 minutes
b) 20 minutes
c) 10 minutes
d) 25 minutes
A
b) 20 minutes
Explanation:
The nurse records minute volume and obtains ABG to measure partial pressure of carbon dioxide (PaCO2), pH, and PaO2 after 20 minutes of continuous mechanical ventilation.
156
q
When interpreting the results of a Mantoux test, the nurse explains to the patient that if the intradermal injection site shows which of the following signs, a reaction will occur?
a) Redness and induration
b) hematomas
c) detachment of tissue
d) dehydration
A
a) Redness and induration
Explanation:
A reaction occurs when there is induration and erythema (redness)
156
q
A client's family asks you to explain some keloid scars the client has developed. The best explanation for keloid scars would be that keloid scars are:
1. Due to a relatively rare hereditary predisposition.
2. Caused by an abnormal amount of collagen deposited as scars form.
3. More common in fair-skinned people of Northern European ancestry.
4. Caused by repeated early and abrupt cessation of scab formation.
A
2. Caused by an abnormal amount of collagen deposited in the formation of scars; Keloid scars are due to an abnormal amount of collagen deposited during the maturation stage of scar formation and are more likely to occur in dark-skinned people.
157
q
A patient has a moisture-retaining bandage to treat a pressure ulcer on the sacrum. How long should the caregiver leave the bandage on before putting it back on?
- 4-6 standard
- 8 pattern
- 12-24 Standard
- 24-36 Standard
A
12-24 Standard
158
q
If you are caring for an obese patient 4-5 days after surgery who has occasional nausea and vomiting and is not good at fluid control, what would you be most concerned about?
1. Postoperative bleeding and anemia
2. Dehiscence and evisceration of wounds
3. Compromised skin integrity and pressure ulcers
4. Loss of mobility and paralytic ileus
A
2. Dehiscence and evisceration of wounds; Wound dehiscence is most likely to occur 4 to 5 days after surgery, and risk factors include obesity, poor diet, multiple trauma, suture failure, excessive coughing, vomiting, and dehydration.
159
q
The nurse examines a patient and finds a herpes simplex/zoster skin lesion. How does the caregiver document this injury?
- Mancha
- papules
- gallbladder
- papule
A
gallbladder
161
q
The nurse discovers that a patient has round red spots on his lower extremities. The nurse documents this finding as which of the following?
a) Petechiae
b) Espinongioma
c) Teleangiektasien
d) equimose
A
a) Petechiae
Explanation:
Petechiae are associated with bleeding tendencies or skin emboli. Spider angioma is associated with liver disease, pregnancy, and vitamin B deficiency. Ecchymosis is associated with trauma and a tendency to bleed. Telangiectasias are associated with venous pressure conditions.
163
q
A nurse from the surgical team was designated as the surgical nurse. What OR Nursing Actions Are Appropriate?
a) Delivery of instruments to the surgeon and assistants
b) Lead the surgical team in a debriefing
c) Keep all records and adjust lights.
d) Coordinate the activities of other employees.
A
a) Delivery of instruments to the surgeon and assistants
Explanation:
The duties of a surgical nurse include assisting the surgical team by handing out instruments to the surgeon and assistants, preparing sutures, collecting specimens for laboratory testing, and counting sponges and needles. The responsibilities of a circulating nurse include guiding the surgical team through an interview, keeping records, adjusting lighting, and coordinating the activities of other team members.
164
q
A nurse assesses a patient postoperatively for protrusion of abdominal organs through the surgical incision. Which term, documented by the nurse, best describes the findings?
a) Erythema
b) Hernia
c) Dehishenz
d) expand
A
d) expand
Explanation:
Evisceration is a surgical emergency. A hernia is a weak spot in the abdominal wall. Dehiscence refers to the partial or complete separation of the wound edges. Erythema refers to redness of the tissue.
166
q
The nurse performs an assessment of a patient with CHF. The caregiver asks if the patient has difficulty breathing in a position other than upright. What does the nurse mean?
- Dyspnoea
- Ortopnoe
- tachypneia
- BradipnoeO
A
Ortopnoe
167
q
The nurse cares for a client who is receiving 25 mg of oral hydrochlorothiazide in addition to his high blood pressure (HBP) medication regimen. Which of the following instructions should the nurse give the patient?
a) "You may develop a dry mouth or nasal congestion while taking this medication."
b) "You can drink alcohol while taking this medication."
c) "Increase the amount of fruits and vegetables you eat."
d) "Take this medication before bedtime."
A
c) "Increase the amount of fruits and vegetables you eat."
Explanation:
Thiazide diuretics cause sodium, potassium, and magnesium depletion. The patient should be encouraged to eat potassium-rich fruits and vegetables. Diuretics cause increased urination; The patient should not take the medication at bedtime. Thiazide diuretics to prevent dry mouth or nasal congestion. Alcohol can increase orthostatic hypotension (a side effect).
168
q
The nurse examines a dark-skinned patient with cherry-red fingernails, lips, and oral mucosa. What do these assessment data indicate that the patient may be experiencing?
- Anamie
- CO positioning
- Polyzythäm
- OC shock
A
CO poisoning
170
q
A medical student assigned to observe an operation enters the operating area without restraint wearing jeans, a T-shirt, and sneakers. What is the best action of the nurse?
a) Inform the medical student about the clothing required for each surgical area.
b) Provide the medical student with a cap and mask.
c) Immediately escort the medical student out of the area.
d) No action is required.
A
a) Inform the medical student about the clothing required for each surgical area.
Explanation:
It would be best to educate the medical student on the attire required for each surgical area. While the student is observing an operation, she should dress appropriately in each zone to reduce the risk of introducing pathogens. The unrestricted zone allows street clothing; therefore, the student does not need to be removed. If the nurse does nothing, the student may enter the restricted or restricted area without proper attire. The provision of a cap and mask does not eliminate the need to remove street clothes to observe the operation.
172
q
A
173
q
The patient is trained in the use of incentive spirometry prior to surgery. What should nurses include in their training?
- Have the patient lie supine while using spirometry
- Encourage the patient to try to stop coughing during and after using spirometry
- Inform the patient that the use of the spirometer is not necessary if the patient is in pain
- Encourage the patient to take about 10 breaths per hour while awake.
A
Encourage the patient to take approximately 10 breaths per hour while awake.
174
q
What is the term for the transfer of heat from the body to a cooler object in contact with it?
a) line
b) evaporation
c) radiation
d) lichenification
A
a) line
Explanation:
Conduction is the transfer of heat from the body to a cooler object in contact with it. Radiation is the transfer of heat to another object that is more distant and has a lower temperature. Lichenification is the leathery thickening of the skin. Convection is the movement of hot air molecules away from the body.
175
q
A patient with suspected acute respiratory distress syndrome (ARDS) experiences anxiety and agitation due to increased hypoxemia and dyspnea. Which of the following would a nurse do to improve oxygenation and make the patient more comfortable?
a) Energy flows for the next 24 hours
b) Administer pancuronium (Pavulon) in small doses
c) Place the patient in the prone position.
d) Help the patient to sit in a chair
A
c) Place the patient in the prone position.
Explanation:
The patient is extremely anxious and agitated due to increased hypoxemia and dyspnea. It is important to reduce the patient's anxiety, as anxiety increases oxygen consumption. Oxygen delivery in ARDS patients is sometimes improved in the prone position. Rest is important to limit oxygen consumption and reduce oxygen demand.
176
q
When the nurse examines the skin of a patient with tinea pedis, she finds a linear laceration. Is this documented by the caregiver as follows?
A scale
b) ulcer
c) crack
d) erosion
A
c) crack
Explanation:
Linear tears in the skin are documented as fissures. Scales are scales secondary to dead, squamous epithelium. Erosions are defined as loss of the superficial epidermis that does not extend into the dermis. Ulcers are skin loss that extends beyond the epidermis.
177
q
Nurse completed training in self-administration of sublingual nitroglycerin. Which of the following patient statements shows that patient education was effective?
a) "Side effects of nitroglycerin include hot flashes, throbbing headache, and high blood pressure."
b) "I can take nitroglycerin before sexual intercourse so I don't feel chest pain."
c) "After taking two pills with no relief, I must call an ambulance."
d) "I can put the nitroglycerin tablets in my daily pill box with my other medications."
A
b) "I can take nitroglycerin before sexual intercourse so I don't feel chest pain."
Explanation:
Nitroglycerin can be taken before any activity that may cause pain. Since nitroglycerin, when taken prophylactically (i.e., before angina-inducing activities such as exercise, climbing stairs, or sexual intercourse) increases exercise and stress tolerance, it is best taken before angina develops. pain. The client is instructed to take three pills 5 minutes apart, and if the chest pain is not relieved, EMS should be contacted. Nitroglycerin is very unstable; it must be transported safely in its original packaging (for example, a dark glass bottle with a lid); Pills should never be removed and stored in metal or plastic cases. Side effects of nitroglycerin include: hot flashes, throbbing headache, hypotension, and rapid heartbeat.
178
q
A nurse listens to the heart of a patient diagnosed with mitral valve prolapse. What is often the first and only manifestation of mitral valve prolapse?
a) Additional heart sound
b) Fatigue
(c) Syncope
d) foolishness
A
a) Additional heart sound
Explanation:
Often the first and only sign of mitral valve prolapse is identified when a physical exam of the heart reveals an additional heart murmur called a mitral valve click. Fatigue, dizziness, and syncope are other symptoms of mitral valve prolapse.
180
q
The nurse cares for a patient with a suspected malignant skin disease. The nurse anticipates that the patient will undergo which of the following diagnostic tests?
a) Patch tests
b) skin scrapes
c) Tzanck smear
d) Biopsy
A
d) Biopsy
Explanation:
Biopsies are performed on skin nodules, plaques, blisters, and other lesions to rule out malignancy and provide an accurate diagnosis. Skin scrapings are used to diagnose spores and hyphae. A Tzanck swab is a test used to examine the cells of blistering skin conditions, such as shingles. A patch test is used to identify the substances to which the patient has developed an allergy.
181
q
The patient is admitted to the Intensive Care Unit (ICU) with a diagnosis of emergency/hypertensive crisis. The patient's blood pressure (BP) is 200/130 mm Hg. The nurse prepares to administer Nitropress (nitroprusside) intravenously. After assessment, which of the following patient findings requires immediate caregiver intervention?
a) Chest pain score of 3/10 (on a scale of 1 to 10)
b) Diuresis of 40 cc/ml in the last hour
c) Nausea and severe headache
d) Numbness and weakness of the left arm
A
d) Numbness and weakness of the left arm
Explanation:
Hypertensive emergencies are acute, life-threatening increases in blood pressure that require immediate treatment in an intensive care unit because of the severe damage that can occur to end organs. Findings of numbness and weakness in the left arm may indicate that the patient has neurological symptoms associated with ischemic stroke due to very high blood pressure and requires immediate intervention. A urine output of 40 ml/h is within the normal range. The other findings are likely caused by high blood pressure and require intervention, but do not require as urgent action as neurological changes.
183
q
What finding by the nurse might indicate that the patient has chronic hypoxia?
- cracks
- peripheral edema
- tapping fingers
- Zianose
A
tapping fingers
184
q
The nurse cares for a patient newly diagnosed with high blood pressure. Which of the following patient statements indicates the need for additional training?
a) "When I get out of bed, I sit down for a moment before getting up."
b) "If I measure my blood pressure and it's normal, I don't need to take my blood pressure medication."
c) "I think I'll sign up for a yoga class twice a week to relieve my stress."
d) "I'm going to see a nutritionist to control my weight."
A
b) "If I measure my blood pressure and it's normal, I don't need to take my blood pressure medication."
Explanation:
The patient needs to understand the disease process and how lifestyle changes and medications can control high blood pressure. The patient must take the medication as directed. Normal blood pressure indicates that the medication is having the desired effect. The remaining responses do not indicate the need for additional training.
185
q
The nurse cares for a patient with a lung disease. Which observation by the nurse indicates a very late symptom of hypoxia?
- Zianose
- Dyspnoea
- concern
- confusion
A
Zianose
187
q
A nurse assists a patient who presents to the emergency room (ER) complaining of chest pain. Which of the following electrocardiographic (ECG) findings would the nurse be most concerned about?
a) ST elevations
b) Frequent premature atrial contractions (PACs)
c) Isolated premature ventricular beats (PVCs)
d) sinus tachycardia
A
a) ST elevations
Explanation:
The first signs of acute myocardial infarction are usually seen in the T wave and ST segment. The T wave is inverted; the ST segment is elevated (usually flat). ST-segment elevation in two adjacent leads is a key diagnostic indicator of MI (ie, ST-elevation myocardial infarction, STEMI). This patient requires immediate invasive therapy or fibrinolytic drugs. While the other ECG findings require intervention, ST-segment elevation requires immediate and definitive intervention.
188
q
When the balloon at the distal end of a pulmonary artery catheter is inflated and the pressure is measured, which of the following is called the measurement obtained?
a) heart gas
b) pulmonary arterial pressure
c) central venous pressure
d) Pulmonalarterienkeildruck
A
d) Pulmonalarterienkeildruck
Explanation:
When the balloon is inflated, the tip of the catheter floats into smaller branches of the pulmonary artery until it can no longer pass, and the pressure is recorded, reflecting left atrial pressure and left ventricular end-diastolic pressure. Central venous pressure is measured in the right atrium. Pulmonary artery pressure is measured when the tip of the balloon is deflated. Cardiac output is determined by thermodilution, in which fluid is injected into the pulmonary artery catheter.
189
q
The nurse is assigned to care for a patient with a chest tube. The nurse should ensure that which of the following items are kept at the patient's bedside?
a) A bottle of sterilized water
b) A set of hemostats.
c) An incentive spirometer
d) An Ambu bag
A
a) A bottle of sterilized water
Explanation:
It is important for the caregiver to ensure that a sterile water bottle is available at the patient's bedside. If the chest tube and drainage system are disconnected, air can enter the pleural space and cause a pneumothorax. To prevent the development of a pneumothorax, a temporary water seal can be created by placing the open end of the chest tube in a bottle of sterile water. No need to keep an ambu bag, incentive spirometer, or set of hemostats at the bedside.
190
q
Which of the following is the primary nursing diagnosis for the patient undergoing laryngectomy?
a) Ineffective airway clearance
b) Impaired verbal communication
c) anxiety and depression
d) Unbalanced diet: Less than what the body needs
A
a) Ineffective airway clearance
Explanation:
The main nursing diagnosis is Ineffective Airway Clearance using ABC. Imbalanced diet: Less than physical needs, impaired verbal communication, and anxiety and depression are possible nursing diagnoses, but are not a priority diagnosis.
190
q
The nursing license exam is exactly the same in every state in the United States. THIS EXAM:
A. Ensure safe care for all patients
B. Ensure standard care for all patients
C. Ensures the provision of honest and ethical care.
D. Provides a minimum standard of knowledge for a registered nurse in practice
A
D. Provides a minimum standard of knowledge for a registered nurse in practice
Registered Nursing (RN) candidates must pass the NCLEX-RN® administered by each State Board of Nursing. Regardless of educational preparation, the RN licensing exam is exactly the same in every state in the United States. This provides a standardized minimum knowledge base for nurses.
190
q
The nurse gives instructions to a patient with acne. Which of the following foods does the caregiver avoid? Select all that apply.
an ice cream
b) banana
c)chocolate
d) onion
A
a) ice cream c) chocolate
Explanation:
The caregiver should be encouraged to avoid foods associated with acne breakouts, including chocolate, cola, fried foods, and dairy products.
191
q
A nurse prepares to perform chest physiotherapy (CPT) on a patient. Which of the following patient statements would indicate that the procedure is contraindicated?
a) "I just had lunch, I'm ready for my CPT."
b) "I got my pain pills 10 minutes ago, let's do my CPT now."
c) “I just put on my coat; we can do my CPT now.
d) "I've been coughing all morning and I can hardly expel anything."
A
a) "I just had lunch, I'm ready for my CPT."
Explanation:
When performing CPT, the caregiver ensures that the patient is comfortable, is not wearing restrictive clothing, and has not eaten anything else. The nurse will administer pain medication as prescribed prior to percussion and vibration, splint each incision, and provide support pads if needed. One of the objectives of the CPT is for the patient to be able to mobilize the secretions; the patient with an unproductive cough is a candidate for PTC.
192
q
A patient diagnosed with acute respiratory distress syndrome (ARDS) is restless and has low oxygen saturation. If the patient's condition does not improve and oxygen saturation continues to fall, what procedure is the caregiver likely to use to help the patient breathe more easily?
a) Increase oxygen supply
b) Administer a large dose of furosemide (Lasix) IVP stat
c) Schedule the patient for lung surgery
d) Intubate the patient and control breathing with mechanical ventilation
A
d) Intubate the patient and control breathing with mechanical ventilation
Explanation:
A patient with ARDS may require mechanical ventilation to support breathing while the underlying cause of the pulmonary edema can be addressed. The other options are not suitable.
194
q
A nurse cares for a patient newly diagnosed with coronary artery disease (CHD). When developing a patient curriculum to address modifiable risk factors for CHD, the caregiver will consider which of the following? Select all that apply.
a) obesity
b) Decreased LDL level
c) alcohol consumption
d) drug use
e) Increased blood pressure
A
a) obesity, e) increased blood pressure
Explanation:
Hypertension, obesity, hyperlipidemia, smoking, diabetes mellitus, metabolic syndrome, and physical inactivity are modifiable risk factors for CHD. Alcohol and drug use are not included in the list of modifiable risk factors for CHD.
195
q
An obese patient undergoes abdominal surgery. A surgeon says during the procedure: "The amount of fat we have to cut is disgusting." What is the best caregiver response?
a) Inform the resident to the treating surgeon.
b) Ignore the comment.
c) Discuss any concerns about the comments with the responsible caregiver.
d) Inform the resident that all communication must be professional.
A
d) Inform the resident that all communication must be professional.
Explanation:
The nurse must speak for the patient, especially when the patient cannot speak for himself. By informing the resident that all communication must be professional, the caregiver attends to the comment at that time and advocates for the patient. Ignoring the comment is not appropriate. Concerns about unprofessional communication may need to be addressed by the nurse with the charge surgeon or charge nurse if the behavior persists. The best course of action is to address the behavior when it occurs.
196
q
When a patient is diagnosed with pulmonary embolism, which of the following ventilation-perfusion relationships is present?
a) Silent unit
b) Low ventilation/perfusion ratio
c) Tótraum
d) Normal ratio of perfusion to ventilation
A
c) Tótraum
Explanation:
When ventilation exceeds perfusion, a dead space (high ventilation to perfusion ratios) is created. Examples of dead space are pulmonary embolism, pulmonary infarction, and cardiogenic shock. With pneumonia or mucus plugs, there is a low ventilation-perfusion ratio. A silent drive occurs in pneumothorax or ARDS.
198
q
The nurse is caring for a 56-year-old male patient who had an implantable cardioverter-defibrillator (ICD) implanted 2 days ago. The patient tells the nurse, "My wife and I will never be able to have sex now that I have this ICD." Which of the following is the best response from the nurse?
a) "Sex is allowed after ICD implantation."
b) "You really should talk to your wife about your concerns."
c) "I will definitely discuss your concerns with the doctor."
d) "He seems concerned about resuming sexual activity."
A
d) "He seems concerned about resuming sexual activity."
Explanation:
The patient treated with an electronic device experiences not only physical and lifestyle changes, but also emotional changes. At various times during the healing process, the patient may experience anger, depression, anxiety, anxiety, or a combination of these emotions. It is imperative that the caregiver monitor the patient's response to the device and provide emotional support and counseling to the patient and their family members, as appropriate. Observing that the patient appears to have concerns about resuming sexual activity acknowledges the patient's concerns and allows for further discussion. The remaining responses ignore the patient's feelings and do not facilitate continued discussion or address the patient's concern.
199
q
A college student comes to the health center with signs and symptoms of viral rhinitis (common cold). The patient says: "I have been feeling very bad all week, what can I do to feel better?" Which of the following responses does the caregiver do best?
a) "Your symptoms should go away soon, just try to get some rest."
b) "You should rest, drink more and take ibuprofen."
c) “Have you ever tried a decongestant nasal spray? they work well.”
d) "Antibiotics will be prescribed to make you feel better."
A
b) "You should rest, drink more and take ibuprofen."
Explanation:
Treatment of viral rhinitis consists of symptomatic therapy, including adequate fluid intake, rest, prevention of hypothermia, and use of expectorants when necessary. Gargling with warm salt water soothes a sore throat, and nonsteroidal anti-inflammatory drugs (NSAIDs), such as aspirin or ibuprofen, ease discomfort and pain. Antibiotics are not prescribed because they do not affect the virus that is causing the patient's signs and symptoms. Topical decongestant nasal sprays should be used with caution. Viral rhinitis symptoms can last 1-2 weeks.
200
q
The nurse cares for a patient who is scheduled for a lobectomy. After the procedure, the caregiver plans care based on which of the following?
a) The patient returns to the ward with two chest tubes.
b) The patient should be sedated until the chest tubes are removed.
c) The patient will return from the operation without drainage tubes.
d) The patient requires mechanical ventilation after the operation.
A
a) The patient returns to the ward with two chest tubes.
Explanation:
The caregiver should plan to return the patient to the nursing unit with two chest tubes intact. During a lobectomy, the lobe is removed and the remaining lobes of the lungs are re-expanded. Usually, two chest catheters are inserted for drainage. The upper tube is used to remove air; the lower one is used to drain liquids. Sometimes only a catheter is needed. The chest tube is connected to a chest tube for several days.
202
q
Which of the following didactic interventions should the nurse include in the treatment plan for a valvular patient with pulmonary congestion?
a) Teach the patient to take nitroglycerin if he has difficulty breathing
b) Teach patients to report a weight gain of 3 pounds in 1 week
c) Teach the patient to rest and sleep in a chair or sit in bed with his head raised
d) Teach the patient to drink at least 2 liters of fluids a day and monitor diuresis
A
c) Teach the patient to rest and sleep in a chair or sit in bed with his head raised
Explanation:
It is recommended that patients experiencing symptoms of lung congestion rest and sleep sitting up in a chair or bed with their head elevated. In addition, the nurse trains the patient to weigh himself daily and report an increase of 3 pounds in a day or 5 pounds in a week to the primary caregiver. The nurse can help the patient plan activities and rest times to achieve an acceptable lifestyle.
204
q
The nurse assists the patient after tonsillectomy and adenoidectomy. Two hours after the procedure, the patient begins to vomit large amounts of dark blood at frequent intervals, tachycardic, and febrile. What will the nurse do after notifying the surgeon?
a) Prepare a needle aspiration.
b) Stay with the patient and monitor her closely.
c) Prepare a needle aspiration.
d) Get light, mirror, gauze, curved hemostatic forceps.
A
d) Get light, mirror, gauze, curved hemostatic forceps.
Explanation:
If the patient vomits large amounts of dark blood at short intervals, the pulse and temperature increase, or the patient becomes restless, the nurse will immediately inform the surgeon. The nurse should have the following items ready to examine the surgical site for bleeding: a lamp, mirror, gauze, curved hemostats, and a waste container. It is not necessary for the caregiver to remain at the patient's bedside. Needle aspiration is a considered procedure for patients with a peritonsillar abscess. While oral suction may be necessary at a point of care, it is not a priority at this time.
205
q
A postoperative patient is discharged home after minor surgery. The PACU nurse will review the discharge instructions with the patient and the patient's spouse. What actions are appropriate for the caregiver? Select all that apply.
a) Inform about activity restrictions.
b) Discuss wound care.
c) That the patient sign his living will.
d) Have the spouse verify by notifying the doctor.
e) Availability of information on health promotion issues.
A
a) Inform about activity restrictions. b) Discuss wound care. d) Have the spouse verify by notifying the doctor. e) Provide information on health promotion issues.
Explanation:
The caregiver should educate about activity limitations and wound care and review complications that require physician notification. The caregiver should also provide information on topics related to health promotion, such as weight control and smoking cessation. The patient should not make important decisions or sign any legal forms due to the effects of anesthesia.
207
q
A patient has been receiving 100% O2 therapy with a mask without rebreathing for several days. The patient now complains of tingling fingers and shortness of breath, is extremely restless, and describes pain below the sternum. What does the caregiver expect?
- Oxygen-induced hypoventilation
- oxygen toxicity
- Oxygen-induced atelectasis
- hypoxia
A
oxygen toxicity
208
q
A 55-year-old man, newly diagnosed with high blood pressure, returns to his physician's office for a routine follow-up visit after several months of treatment with Lopressor (metoprolol). During the initial assessment by the nurse, the patient's blood pressure (BP) is recorded as 180/90 mmHg. The patient states that he is not taking the prescribed medication. Which of the following is the best response from the nurse?
a) "Be sure to discuss your non-compliance with your medication schedule with your doctor."
b) “Your high blood pressure needs to be treated with medication; You must take your Lopressor every day.
c) "It is very important that you take the medication as prescribed, otherwise you could have a stroke."
d) “The medication you were prescribed may cause sexual dysfunction; Are you having this side effect?"
A
d) “The medication you were prescribed may cause sexual dysfunction; Are you having this side effect?"
Explanation:
The caregiver needs to understand why the patient is not taking the medication. Lopressor is a beta blocker. All patients should be informed that beta-blockers can cause sexual dysfunction and that other medications are available if problems with sexual function occur. While the other statements are true, they are not therapeutic and would not determine why the patient was not taking the prescribed medication.
209
q
The nurse assists a male patient diagnosed with rhinosinusitis. The doctor prescribed the patient four sprays of budesonide (Rhinocort) in each nostril each morning. The nurse informs the patient that a common side effect of this medication includes which of the following?
a) teary eyes
b) headache
c) nosebleeds
d) Artralgia
A
c) nosebleeds
Explanation:
Common side effects of budesonide include epistaxis, pharyngitis, coughing, nasal irritation, and bronchospasm.
210
q
Which of the following diagnostic tests is used to confirm the diagnosis of maxillary and frontal sinusitis?
a) X-rays of the paranasal sinuses
b) Aspiring sinus
c) TRM
d) Computed tomography
A
b) Aspiring sinus
Explanation:
Sinus aspirates may be obtained to confirm the diagnosis of inflammation of the maxillary and frontal sinuses and to identify the pathogen. For this, flexible endoscopic culture techniques and paranasal sinus smears were used. Sinus radiographs and CT scans may be performed in patients with frontal headache, in refractory cases, and when complications are suspected.
211
q
What type of fan has a predetermined volume of air that must be released with each inspiration?
a) Zeitzyklus
b) negative pressure
c) pressure cycles
d) volume cycled
A
d) volume cycled
Explanation:
With volume-gated ventilation, the volume of air to be delivered with each inspiration is available. Negative pressure ventilators apply negative pressure to the outside of the chest. Timed fans stop or control inspiration after a preset time. When the pressure controlled ventilator is turned on, it provides airflow (inspiration) until it reaches the current pressure, then it is turned off and exhalation occurs passively.
213
q
The caregiver acknowledges that the treatment of a non-ST-segment elevation myocardial infarction (NSTEMI) differs from the treatment of a patient with a STEMI in that a STEMI is most commonly treated with which of the following agents?
a) IV-Heparin
b) Perkutane Koronar Intervention (PCI)
c) Nitroglycerine IV
d) Thrombolytic
A
b) Perkutane Koronar Intervention (PCI)
Explanation:
Often, the patient with STEMI is taken directly to the cardiac catheterization laboratory for immediate PCI. Superior results have been reported using PCI compared to thrombolytics. IV heparin and IV nitroglycerin are used to treat NSTEMI.
215
q
Hypertension is officially diagnosed when the patient has a systolic blood pressure greater than ______ mm Hg and a diastolic blood pressure greater than _____ mm Hg for a prolonged period of time.
a) 110, 60
b) 130, 80
c) 120, 70
d) 140, 90
A
d) 140, 90
Explanation:
According to the categories of blood pressure values established by the Joint National Committee (JNC) VI, stage 1 hypertension is indicated by a systolic pressure of 140-159 or a diastolic pressure of 90-99. Systolic pressures of 130 and diastolic pressures of 80 fall within the normal range for an adult. A systolic pressure of 110 and a diastolic pressure of 60 is within the normal range for an adult. A systolic pressure of 120 and a diastolic pressure of 70 are within the normal range for an adult.
216
q
Which of the following actions would put a patient at increased risk of skin shear injury?
1. Walking without shoes
2. Sitting in the Fowler position
3. Lie on your back in bed
4. With a heating pad
A
2. Sitting in the Fowler position; None of the other movements or situations produce the combination of friction and pressure with downward movement seen in bedridden patients in Fowler's position.
217
q
A nurse gives discharge instructions to a patient after throat surgery. The nurse instructs the patient to avoid which of the following?
a) Wear a tissue over the stoma
b) tos
c) Wear a plastic bib when showering
d) Nadar
A
d) Nadar
Explanation:
Swimming is not recommended because a person with a laryngectomy can drown without submerging their face. Special precautions are needed when bathing to prevent water from entering the stoma. Wearing a loose plastic bib over the tracheostoma or simply placing your hand over the opening is effective. The nurse also suggests that the patient wear a tissue over the stoma to make the opening less obvious. The nurse encourages the patient to cough every 2 hours to promote effective gas exchange.
218
q
The purposes of the medical history for the patient
to be operated on include all of the following
except:
1. Decide if surgery is indicated
2. Identify the patient's perception and
operating expectations
3. Obtaining information about the patient's past
experience with surgery
4. Understand the impact of an operation on the
emotional health of the patient and his family
A
Deciding if surgery is indicated
This is the doctor's decision and responsibility.
from the provider
220
q
A nurse tries to help a patient who had shoulder surgery earlier that day. The patient refuses to walk. What caregiver measures make the most sense?
a) Use several workers to get the patient out of bed.
b) Delegate the task to unlicensed support staff.
c) Highlight the importance of early mobility to avoid complications.
d) Document the patient's refusal.
A
c) Highlight the importance of early mobility to avoid complications.
Explanation:
The patient may refuse to walk out of fear or pain. Education on the importance of mobility to avoid complications can encourage the patient to walk. The caregiver must take all reasonable steps (pain control, education) before documenting the patient's refusal to ambulate. If the patient already refuses to walk, delegating the task to unlicensed support staff is not an appropriate response. The patient should not be forcibly lifted out of bed.
221
q
For which diagnosis would the nurse expect decreased tactile emotion and hyperresonant percussion sounds when assessed before admission?
- Bronchitis
- emphysema
- atelectasis
- pulmonary edema
A
emphysema
222
q
The nurse examines an African-American patient and notices a pigment stain on the patient's fingernails. Does the nurse determine that this finding indicates the following?
a) chronic anemia
b) Normal fluctuation
c) smoker's nails
d) Melanomas
A
b) Normal fluctuation
Explanation:
Fluctuations in skin pigment levels in dark-skinned patients such as B. Pigmented nail ridges, pigmented spots on the white of the eye, and pigmented wrinkles over the bridge of the nose are considered normal color deviations.
223
q
What statement, if made by the client or a family member, would indicate the need for additional training?
1. If an area of skin turns red, but the redness goes away after turning, I should tell the nurse.
2. Placing foam pads under your heels or other bony areas can help relieve pressure.
3. If a person cannot turn over in bed, someone should help them change position every 4 hours.
4. The skin should only be washed and lubricated with warm (not hot) water while it is still slightly damp.
A
3. If a person cannot turn over in bed, someone should help them change position q4h; Immobile and dependent people must be repositioned at least every 2 hours, not every 4, so this client or family member needs school tutoring.
Warm water and moist, moist skin are the proper skin care techniques. Areas of redness that do not return to normal skin color should be reported. It would also be nice to use a foam pad to relieve pressure.
224
q
A confused patient who fell over the side of the bed
The rails were not used is an example of what kind of
Responsibility?
1. Crime
2. Battery
3. Attack
4. Negligence
A
4. Negligence
4. Conduct within due diligence standards
226
q
A 66-year-old man presents to the emergency department complaining of a severe headache and mild nausea for the past 6 hours. On evaluation, the patient's blood pressure is 210/120 mm Hg. The patient has a history of hypertension for which he is taking clonidine (Catapres) 1.0 mg twice daily. Which of the following questions is the most important for the nurse to ask the patient?
a) "Do you have chest pain or shortness of breath?"
b) "Have you taken your prescribed Catapres today?"
c) "Do you have a dry mouth or stuffy nose?"
d) "Did you take any medication for your headache?"
A
b) "Have you taken your prescribed Catapres today?"
Explanation:
The nurse should ask if the patient has taken the prescribed Catapres. Patients should be informed that rebound hypertension can occur when antihypertensive drugs are suddenly discontinued. A particular side effect of Catapres is the rebound or withdrawal of high blood pressure. While other questions may be asked, the most important one is to ask if the patient is taking the medication prescribed for hypertension because of the patient's severely elevated blood pressure.
228
q
A nurse answers a male patient's call light. The patient complains of an irritating tickling sensation in the throat, a salty taste, and a burning sensation in the chest. Upon further examination, the nurse notices a handkerchief covered in bright red frothy blood on the bed. The nurse can assume that the blood source is likely to come from which of the following sources?
a) the stomach
b) the rectum
c) the lungs
d) Or nose
A
c) the lungs
Explanation:
The blood in the lungs is usually bright red, frothy, and mixed with sputum. Initial symptoms include a tickle in the throat, a salty taste, a burning or bubbling sensation in the chest, and possible chest pain. In this case, the patient tends to immobilize the bleeding side. This blood has an alkaline pH (>7.0). Blood from the stomach is vomited up rather than spit out, can mix with food, and is usually much darker in color; often referred to as "vomiting coffee grounds". This blood has an acidic pH (
229
q
A thick yellow material leaks from a client's wound. The nurse correctly describes the drain as follows:
1. bloody
2. Serous blood
3. serous
4. Purulent
A
4. purulent; The discharge is described as purulent. Bloody and serous-sanguinous contains blood. Serosa is clear and watery.
230
q
A patient is on the first postoperative day of abdominal surgery. The patient receives 0.9% saline solution at 75 mL/h, has a nasogastric tube with 200 mL of light yellow fluid every 8 hours, and wound drainage with 50 mL of dark red drainage every 8 hours. The 24-hour urine output totals 2430 mL. Which nursing measurements make the most sense?
a) Evaluate edema.
b) Document the results and reassess within 24 hours.
c) Stop nasogastric suction.
d) evaluation of signs and symptoms of dehydration.
A
d) evaluation of signs and symptoms of dehydration.
Explanation:
The patient's 24-hour intake is 1800 mL (75 x 24). The patient's 24-hour production is 3180 mL [(200 × 3) + (50 × 3) + 2430]. Because delivery is significantly greater than uptake, the patient is at risk of fluid volume deficit. The nurse should not stop nasogastric suctioning without medical advice. Findings should be documented and reassessed, but the caregiver should take additional steps to prevent complications. Edema is usually associated with excess fluid volume.
232
q
A patient is being evaluated for acute laryngitis. Which of the following symptoms are among the clinical manifestations of acute laryngitis?
a) A non-edematous uvula
b) A throat that gets worse at night.
c) hoarseness
d) A wet cough
A
c) hoarseness
Explanation:
Signs of acute laryngitis are hoarseness or hoarseness and a severe cough. Other signs of acute laryngitis include a dry cough and a sore throat that is worse in the morning. In allergies, the uvula is noticeably swollen.
233
q
A nurse gives discharge instructions to a patient with pulmonary sarcoidosis. Does the nurse conclude that the patient understands the information if the patient correctly identifies which of the following early signs of an exacerbation?
a) lose weight
b) shortness of breath
c) fever
d) headache
A
b) shortness of breath
Explanation:
Early signs and symptoms of pulmonary sarcoidosis may include breathlessness, cough, hemoptysis, and congestion. General symptoms include anorexia, fatigue, and weight loss.
235
q
A new cleaning technician is introduced into the operating room. The OR tech tells the nurse, "You can skip the fire safety information, as I've worked in hospitals for 10 years." What is the best response from the nurse?
a) "The operating room presents unique circumstances that increase the risk of fire."
b) "I know this information is not exciting, but I need to discuss this information with you."
c) "Okay, but you should check the hospital's fire safety policy yourself."
d) "This is a requirement of your job, just persevere."
A
a) "The operating room presents unique circumstances that increase the risk of fire."
Explanation:
The operating room environment has some unique characteristics that increase the risk of fire, such as: B. Curtains that allow oxygen concentration. By having the new employee understand the logic behind fire safety in the operating room, the new employee develops a greater understanding and appreciation of fire safety. If fire safety is presented only as a job requirement, the employee may not understand the importance of fire safety. The hospital's fire safety policy is comprehensive; The employee would need to review the specific operating room fire safety guidelines.
236
q
A nurse watches as a patient signs a consent form for surgery. After the patient signs the informed consent form, she begins to ask questions about the risks and benefits of the surgery. What caregiver measures make the most sense?
a) Place the consent form in the patient's medical record.
b) Inform the nursing supervisor about the patient's doubts.
c) Answer the patient's questions.
d) Ask the surgeon to come and answer the questions.
A
d) Ask the surgeon to come and answer the questions.
Explanation:
It is the doctor's responsibility to provide information about the risks and benefits of surgery. It is not the caregiver's or carer's responsibility to discuss the risks and benefits. The consent form should not be included in the record until all of the patient's questions have been fully answered.
237
q
A patient is scheduled for invasive surgery. What are the priority documents needed for the procedure?
a) The Medication Registration Form
b) An informed consent form signed by the patient
c) A clinical history taken by the general practitioner.
d) Prescriptions for postoperative medications
A
b) An informed consent form signed by the patient
Explanation:
The signed consent is necessary and important to start invasive procedures. The nurse should therefore look for the informed consent form signed by the patient. Medical history, medication reconciliation, and post-op prescriptions are good things, but they are not required records before performing an invasive procedure.
238
q
Advanced General Practice Nurses:
A. Operate independently
B. Role as Head of Unit
C. Work in intensive care
D. Work at the university level
A
A. Operate independently
The certified advanced practice nurse acts independently as a physician, educator, case manager, consultant, and researcher in their practice to plan or improve the quality of care for patients and their families.
239
q
A postoperative patient with an open abdominal wound is taking corticosteroids. The doctor orders a culture from the abdominal wound even though there are no signs or symptoms of infection. What actions are appropriate for the caregiver?
a) Request to interrupt the order without receiving the sample.
b) Suspend the order until purulent discharge is observed.
c) Obtain a culture sample from the wound.
d) Use an antibiotic detergent before collecting the sample.
A
c) Obtain a culture sample from the wound.
Explanation:
Corticosteroids can mask the presence of an infection by interfering with the normal inflammatory response. The culture should be taken even if the patient does not show traditional signs and symptoms of infection. The regimen should not be interrupted or continued until purulent discharge is detected, as the infection may worsen and the patient may develop sepsis. Antibiotic detergent should not be used prior to sample collection, as it will alter the growth of organisms.
240
q
The nurse performs an admission history and physical examination on a patient with a history of contact dermatitis. The nurse assesses whether the patient is using which of the following drug classifications?
a) saline washes
b) Virostatic
c) Corticosteroid
d) antimicotica
A
c) Corticosteroid
Explanation:
Corticosteroids are used for contact dermatitis. Antifungals, antivirals, and saline solutions are not used in the treatment of contact dermatitis.
242
q
When examining an African American patient for cyanosis, which of the following areas should the nurse evaluate?
a) Nails
b) mucosa oral
c) we
d) Sclera
A
b) mucosa oral
Explanation:
In a person with dark skin, the skin often takes on a grayish tint. To detect cyanosis, look at the oral mucosa, conjunctiva, and nail bed.
243
q
What is the incubation period for the herpes simplex virus type 1 (HSV-1) that causes cold sores (cold sores)?
a) 2 to 12 days
b) 3 a 6 meses
c) 20 to 30 days
d) 0 a 3 meses
A
a) 2 to 12 days
Explanation:
HSV-1 is mainly transmitted through direct contact with infected secretions. The incubation period is approximately 2 to 12 days. The period from 0 to 3 months exceeds the incubation period. The period of 20 to 30 days exceeds the incubation period. The period from 3 to 6 months exceeds the incubation period.
244
q
The nurse leads a community awareness program on basal cell carcinoma (BCC). Which of the following statements should the nurse make?
a) It is a malignant growth that originates in the epidermis.
b) Begins as a small waxy knot with translucent, pearly, ruffled edges.
c) Metastasis through the blood or lymphatic system.
d) It is more invasive than squamous cell carcinoma (SCC).
A
b) Begins as a small waxy knot with translucent, pearly, ruffled edges.
Explanation:
BCC usually begins as a small, waxy knot with ruffled, translucent, pearly edges. It is less invasive than SCC. It does not metastasize through the blood or lymphatic system. SCC is a malignant proliferation originating in the epidermis.
244
q
The nurse helps remove a Tzanck swab. What is the suspected diagnosis of PT?
- fungal infection
- Herpes zoster
- soriasis
- seborrheic dermatosis
A
Herpes zoster
246
q
An intensive care unit nurse uses a computerized decision support system to correctly position her ventilated patients to reduce pneumonia caused by accumulated secretions in the airways. This is an example of what QSEN (Quality and Safety in Nursing Education) competency?
A. Non-patient centered care
B. Security
C. Teamwork and collaboration
D. IT
A
D. IT
The use of decision support systems is an example of the use and acquisition of computer skills.
248
q
The nurse cares for a patient with a severe respiratory illness. Which of the following is a late sign of hypoxia in the patient?
a) Hypotonia
b) restlessness
c) drowsiness
d) Zianosis
Show response
A
d) Zianosis
Explanation:
Cyanosis is a late sign of hypoxia. Hypoxia can cause restlessness and an initial increase in blood pressure, followed by hypotension and drowsiness.
249
q
A patient is being treated for chronic venous stasis ulcers in the lower extremities. What medications does the caregiver believe will increase peripheral blood flow, reduce blood viscosity, and aid ulcer healing?
- heparin
- warfarin
- Aspirin
- Pentoxifilina (Trental)
A
pentoxifylline
251
q
A patient involved in a motor vehicle accident sustained a blunt force injury to the chest wall and was taken to the emergency department. Does the nurse examine the patient for what clinical manifestation would indicate the presence of a pneumothorax?
a) Suction sound at the lesion site.
b) Decreased respiratory rate
c) Productive bloody cough
d) Decreased breath sounds.
A
a) Suction sound at the lesion site.
.Explanation:
Open pneumothorax is a form of traumatic pneumothorax. It occurs when a wound in the chest wall is large enough to allow air to flow freely in and out of the chest cavity with each attempt to breathe. Because airflow through the chest wall wound creates a sucking sound, these injuries are called chest suction wounds.
252
q
A middle-aged man presents to the emergency room complaining of severe chest discomfort. Which of the following patient findings is most indicative of a possible myocardial infarction?
a) Intermittent nausea and vomiting for 3 days
b) Appearance cold, clammy, sweaty and pale
c) Chest discomfort that does not improve with rest or nitroglycerin
d) Anxiety, restlessness and drowsiness
A
c) Chest discomfort that does not improve with rest or nitroglycerin
Explanation:
Chest pain or discomfort that is not relieved by rest or nitroglycerin is associated with an acute myocardial infarction. The other findings, although associated with ACS (acute coronary syndrome) or MI, can also occur in angina and are not indicative of MI alone.
254
q
The nurse provides the patient with an endotracheal (ET) tube. Which of the following nursing measures is contraindicated?
a) Make sure that humidified oxygen is always introduced through the tubing
b) Deflate the cuff before removing the tube
c) Regular cuff deflation
d) Check the cuff pressure every 6 to 8 hours
A
c) Regular cuff deflation
Explanation:
Routine cuff deflation is not recommended due to the increased risk of aspiration and hypoxia. The cuff is deflated before removing the ET. Cuff pressure should be checked every 6 to 8 hours. Humidified oxygen should always be introduced through the tubing.
255
q
Crohn's disease is a malabsorptive condition caused by which of the following pathophysiological processes?
a) General resection
b) Disaccharidasemangel
c) Inflammation of all layers of the intestinal mucosa
d) infectious disease
A
c) Inflammation of all layers of the intestinal mucosa
Explanation:
Crohn's disease, also known as regional enteritis, can occur anywhere in the GI tract, but is most common in the distal ileum and colon. Infectious diseases cause problems such as the overgrowth of bacteria in the small intestine, leading to malabsorption. Disaccharidases deficiency leads to lactose intolerance. Postoperative malabsorption occurs after gastric or intestinal resection.
257
q
The nurse discusses immediate postoperative communication strategies with the patient scheduled for total laryngectomy. Which of the following information will the caregiver receive?
a) "After the operation, you must use an electric larynx to communicate."
b) "After the operation he has a sore throat, but he can talk."
c) "A speech therapist will examine you and recommend a postoperative communication system."
d) "You can use writing or a blackboard to communicate."
A
d) "You can use writing or a blackboard to communicate."
Explanation:
When planning a total laryngectomy, the patient must understand that the natural voice is lost, but that special training can provide a means of communication. The patient should know that until the start of the training, communication is possible via the call light, in writing or via a special communication card. The use of an electronic device is a long-term postoperative goal. The speech therapist will examine the patient before the operation and will establish a method of immediate postoperative communication.
258
q
The father of a 16-year-old patient asks the nurse: "How can the surgeon operate without my consent?" What is the best response from the nurse?
a) "The surgical intervention performed does not require consent."
b) "Two doctors decided that your son needed surgery, so we didn't need to ask for consent."
c) "Your son suffered life-threatening injuries that required immediate surgery."
d) "We obtained your son's consent after he requested the surgery."
A
b) "Your son had life-threatening injuries that required immediate surgery."
Explanation:
In an emergency, the surgeon may be required to operate as a life-saving measure without the informed consent of the patient or parent. Informed consent must be obtained before any invasive procedure. A minor cannot consent to a surgical intervention. The opinions of two physicians do not deny the need for informed consent.
260
q
A nurse prepares dietary recommendations for a patient with a lung abscess. Which of the following statements would be included in the plan of care?
a) "You should eat a low-calorie diet such as skim milk, fresh fruits and vegetables."
b) "You should eat a diet high in carbohydrates such as bread, potatoes and pasta."
c) "You must have a low-fat diet, limiting dairy products and concentrated sweets."
d) "You must have a diet rich in protein such as chicken, fish and beans."
A
d) "You must have a diet rich in protein such as chicken, fish and beans."
Explanation:
For a patient with a lung abscess, the nurse recommends a diet high in protein and calories to ensure adequate nutritional intake. A high carbohydrate diet or low fat diets are not recommended for a patient with lung abscess.
262
q
Which of the following terms refers to a condition characterized by the destruction of melanocytes in localized areas of the skin?
a) Lichenification
b) Teleangiektasien
c) Vitiligo
(d) Hirsutism
A
c) Vitiligo
Explanation:
Vitiligo leads to the development of white patches that can be localized or generalized. Hirsutism is the condition of excessive hair growth. Lichenification is a leathery thickening of the skin. Telangiectasias refer to red patches on the skin caused by stretching of the superficial blood vessels.
263
q
If you are teaching a patient with rheumatic carditis and a history of recurrent rheumatic fever, which of the following statements by the patient indicates that the teaching was successful?
a) "You may need to take prophylactic antibiotics for up to 10 years."
b) "I will avoid milk, yogurt and other dairy products."
c) "I will avoid any type of activity."
d) "I will take non-steroidal anti-inflammatory drugs (NSAIDs) every day."
A
a) "You may need to take prophylactic antibiotics for up to 10 years."
Explanation:
Antibiotic prophylaxis for recurrent rheumatic fever with rheumatic carditis may require 10 or more years of antibiotic coverage (eg, intramuscular (IM) penicillin G every 4 weeks, oral penicillin V twice daily (BID), daily oral sulfadiazine or oral erythromycin BID) a history of rheumatic fever are susceptible to infective endocarditis and should be advised to take prophylactic antibiotics before any invasive procedure, including dental procedures. Steroids are prescribed to suppress the inflammatory response and aspirin to prevent formation of blood clots to control the surrounding heart valves. myocardial workload and combat the boredom of weeks of bed rest, activities that require a minimum of activity are recommended.
264
q
The nurse educates the parents of a child with head lice. Which of the following statements should the nurse include?
a) Shampoo with piperonylbutoxide (RID).
b) Wash clothes in cold water.
c) Shampoo with Kwell.
d) Disinfect brushes and combs with bleach.
A
a) Shampoo with piperonylbutoxide (RID).
Explanation:
Instructions from the nurse should include washing with RID, laundering clothes in hot water, and disinfecting brushes and combs with RID shampoo.
265
q
Which of the following statements about air pressure fluctuations is correct?
a) During inspiration, air enters the alveoli through the trachea and bronchi.
b) Air flows from an area of lower pressure to an area of higher pressure during inspiration.
c) The thoracic cavity contracts during inspiration.
d) The diaphragm contracts during inspiration.
A
a) During inspiration, air enters the alveoli through the trachea and bronchi.
Explanation:
During inspiration, movements of the diaphragm and intercostal muscles enlarge the thoracic cavity, reducing the pressure within the thorax to levels below atmospheric pressure. This pushes air through the trachea and bronchi into the alveoli.
266
q
A nurse teaches a patient who is waiting for a heart transplant. Which of the following statements shows that the patient understands what is needed to minimize rejection?
a) "There is no risk of rejection if the donor heart is exactly the same."
b) "I will receive medication before and during the operation that will eliminate the risk of rejection."
c) "I need medication after surgery to prevent rejection, and if my body doesn't reject the new heart, I don't need to take any medication at home."
d) "I'm going to have to take three different types of medications for the rest of my life to avoid rejection."
A
d) "I'm going to have to take three different types of medications for the rest of my life to avoid rejection."
Explanation:
Patients who have undergone a heart transplant constantly weigh the risk of rejection against the risk of infection. They must adhere to a complex regimen of diet, medications, activity, laboratory tests, biopsies of the transplanted heart (to diagnose rejection), and clinic visits. There are three classes of drugs prescribed to a transplant patient to minimize rejection: corticosteroids (eg [Rapamune]).
267
q
The nurse observes a patient during a stress test (bike). Which of the following findings indicates a positive test and the need for further diagnostic testing?
a) Changes in BP; 148/80 mmHg to 166/90 mmHg
b) Dizziness and leg cramps
c) changes in heart rate; 78bpm to 112bpm
d) ST segment changes on ECG
A
d) ST segment changes on ECG
Explanation:
During the test, the following are monitored: two or more ECG leads for heart rate, rhythm, and ischemic changes; blood pressure; skin temperature; Appearance; perceived exertion; and symptoms, including chest pain, dyspnea, dizziness, leg cramps, and fatigue. The test ends when the target heart rate is reached or if the patient develops signs of myocardial ischemia. Other diagnostic tests, such as a cardiac catheterization test, for example, may be warranted if the patient develops chest pain, extreme fatigue, decreased blood pressure or pulse rate, severe arrhythmias, or ST-segment changes on the ECG. during the effort. . The other findings would not justify stopping testing.
268
q
A nurse cares for patients with a variety of injuries. What would be most likely to heal through primary intention?
1. Make cuts in the skin with a kitchen knife
2. Worn perineal area
3. Skin abrasion
4. decubitus
A
1. Make cuts in the skin with a kitchen knife; A cut in the skin caused by a sharp instrument with minimal tissue loss can mostly heal when the edges of the wound come together slightly (approximately).
Abrasions, abrasions, and pressure sores heal secondarily, not primarily. Secondary intention healing occurs when the edges of the wound do not unite due to complete loss of tissue; The wound remains open until it is filled with new tissue. Abrasions and abrasions are injuries to the surface of the skin.
269
q
Graduate nurses must pass a qualifying exam
managed by:
1. State pavilions
2. National League for Nursing
3. Accredited School of Nursing
4. American Nurses Association
A
state nursing offices
270
q
An emergency room nurse examines a patient who complains of dyspnea. Which of these signs would indicate the presence of a pleural effusion?
a) Spots on the skin upon inspection
b) Resonance in the battery
c) wheezing on auscultation
d) Decreased excursion of the chest wall on palpation
A
d) Decreased excursion of the chest wall on palpation
Explanation:
Symptoms of a pleural effusion include shortness of breath, pain, taking a pain-relieving position, absent breath sounds, decreased thrill, a dull percussion sound, and decreased chest wall excursion. The caregiver may also hear a rubbing noise. Chest x-ray and computed tomography (CT) scan show fluid in the affected area.
271
q
Nurses at a community hospital participate in a training program to learn how to use a new pressure relief device for patients at risk of pressure ulcers. What kind of education is this?
A. Continuing Education
B. Graduate education
C. On-the-job training
D. Training to become a registered nurse
A
C. On-the-job training
In-service education programs are training or education provided by a health authority or facility. An in-service program is carried out in the unit, which aims to develop the knowledge, skills and competencies of nurses and other health professionals employed in the unit.
272
q
A client has a diabetic stasis ulcer on his lower leg. The nurse uses a hydrocolloid bandage to cover it. The application process includes:
1. Cleaning the skin and wound with Betadine
2. Remove all traces of old dressing residue.
3. Choose a bandage that is no more than 1/4 inch larger than the size of the wound
4. Hold in place for one minute to allow it to set
A
4. Hold in place for one minute to allow it to set; The skin is cleansed with normal saline or a mild detergent. The remnants of old associations are dissolving. The size of the dressing should be 3 to 4 cm (1.5 inches) larger than the size of the wound.
273
q
What physiological conditions are contraindicated for the use of heat as therapy? (Choose all that apply.)
1. The first 24 hours of the injury
2. Active bleeding
3. Non-inflammatory edema
4. Localized malignant tumor
A
All previous; The heat causes vasodilation and increases blood flow to the affected area, providing oxygen, nutrients, antibodies and white blood cells. A possible disadvantage of heat is that it increases capillary permeability, allowing extracellular fluids and substances to penetrate the capillary walls and cause edema or an increase in existing edema.
Contraindications include: the first 24 hours after the injury, active bleeding, non-inflammatory edema, localized malignancies, and skin diseases that cause redness or blistering.
275
q
The nurse observes that the patient has general paleness and recognizes that this finding may indicate this
a) albinism
b) Anemia
c) Vitiligo
d) Local arterial insufficiency
A
b) Anemia
Explanation:
In fair-skinned people, general paleness is a manifestation of anemia. In individuals with brown and black skin, anemia presents as a dull complexion. Albinism is a state of complete lack of pigment, in which the skin appears whitish-pink. Vitiligo is a condition characterized by the destruction of melanocytes in localized areas of the skin, resulting in mottled milky white patches. Local arterial insufficiency is characterized by pronounced localized pallor.
276
q
Which of the following is the main ingredient for strengthening hair and nails?
a) Merkel cell
b) Melanin
c) Talgdrüse
d) Keratin
A
d) Keratin
Explanation:
Keratin is the main hardening ingredient for hair and nails. Melanin is the substance responsible for skin color. A Merkel cell is found in the epidermis and plays a role in transmitting sensory messages. A sebaceous gland secretes sebum to keep the skin soft and supple.
278
q
Which of the following is the most reliable and accurate method for delivering precise oxygen concentrations non-invasively?
a) piece I T
b) Teilrückatmungsmaske
c) Venturi mask
d) Nasenkanüle
A
c) Venturi mask
Explanation:
The Venturi mask is the most reliable and accurate method of delivering a precise oxygen concentration non-invasively. The mask is designed to mix a constant flow of ambient air with a constant flow of oxygen. Nasal prongs, T-pieces, and partial rebreathing masks are not the most reliable or accurate methods of delivering oxygen.
279
q
The nurse examines the nails of a patient in the clinic. The nurse observes the pitting of the nail plate. What disorder does this finding indicate?
- soriasis
- vitiligo
- Diabetes
- Melanoma
A
soriasis
280
q
Which of the following statements identifies the severe grinding caused by abrasion of the pericardial surfaces during the cardiac cycle?
a) Eject click
b) marbles
c) friction friction
d) opening pressure
A
c) friction friction
Explanation:
In pericarditis, a scratchy, grating sound heard in both systole and diastole is called frictional friction. Turbulent blood flow produces a noise. One cause of the turbulence may be a critically restricted valve. An orifice snapshot is caused by high left atrial pressure with abrupt displacement of a stiff mitral valve. An ejection click is caused by very high pressure in the ventricle displacing a stiff, calcified aortic valve.
281
q
Open abdominal surgery is planned for an overweight patient. What priority training should the nurse provide to this patient?
a) Prevention of venous thromboembolism
b) Prevention of respiratory complications
c) Prevention of wound dehiscence
d) treat wounds and prevent infection
A
b) Prevention of respiratory complications
Explanation:
All answers are correct, but the obese patient is more prone to airway complications and maintaining a patent airway would be a priority.
282
q
Hypertension that is traceable to an underlying cause is known as which of the following?
a) Secondary
b) Primary
(c) Essential
d) isolated systolic
A
a) Secondary
Explanation:
Secondary high blood pressure can be caused by a tumor of the adrenal gland (eg, pheochromocytoma). Primary hypertension has no known underlying cause. Essential hypertension has no known cause. Isolated systolic hypertension is demonstrated by readings in which the systolic pressure is greater than 140 mm Hg and the diastolic reading is normal or near normal (less than 90 mm Hg).
283
q
How soon should percutaneous transluminal coronary angioplasty (PTCA) be performed to be effective, beginning with arrival in the emergency department after diagnosis of myocardial infarction (MI)?
a) 60 minutes
b) 30 minutes
c) 9 Roofs
d) 6 a 12 meses
A
a) 60 minutes
Explanation:
The 60-minute interval is known as the "door-to-balloon time" for performing a PTCA in a patient diagnosed with myocardial infarction. The 30 minute interval is known as the "door-to-needle time" for post-MI administration of thrombolytics. The 9-day period refers to the time to onset of vasculitis after administration of streptokinase for thrombolysis in a patient with acute myocardial infarction. The period of 6 to 12 months refers to the period in which streptokinase is not re-administered in the same patient for acute myocardial infarction.
284
q
A nurse discusses the laboratory results of a patient diagnosed with coronary artery disease (CAD). The patient's low-density lipoprotein (LDL) level is 115 mg/dL. The caregiver interprets this value as which of the following?
a high
b) Critically high
c) low
d) Within normal limits
A
a high
Explanation:
The normal range for LDL is between 100 mg/dL and 130 mg/dL. A value of 115 mg/dl is considered high. The goal of treatment is to bring LDL levels below 100 mg/dL (less than 70 mg/dL for very high-risk patients).
286
q
The nurse performs a preoperative examination on a patient scheduled for gallbladder surgery. The patient reports a frequent cough with greenish sputum for the past 3 days and denies fever. On auscultation, the nurse notes rumbling sounds throughout the right lung with an occasional whooshing sound. Respiratory rate is 20, temperature is 99.8 orally, heart rate is 87, and blood pressure is 124/70. What is the best action of the nurse?
a) Notify the surgeon to eventually postpone the operation.
b) Notify the family doctor of the test results.
c) Document the findings and monitor the patient before the operation.
d) Wait 1 hour and retest.
A
a) Notify the surgeon to eventually postpone the operation.
Explanation:
A respiratory infection can delay non-emergency surgery because the infection can increase the risk of respiratory complications. Therefore, the nurse must inform the surgeon of any delay in the operation. The family doctor can be called to deal with the test results, but this should only be done after notifying the surgeon. It is not convenient to continue with the preoperative period without notifying the surgeon and waiting 1 hour.
287
q
You are seeing a designated client and notice a superficial ulcer on the client's buttocks that appears as a shallow crater involving the epidermis and dermis. Which of the following stages do you think best describes this breach in skin integrity?
1. Filling I
2. Stage II
3. Stove 3
4. Stage IV
A
2. Stage II; Stage I pressure ulcers involve intractable erythema of intact skin, whereas stage II pressure ulcers involve partial loss of skin with epidermis, dermis, or both, with the ulcer being superficial and presenting as an abrasion, blister, or crater. little deep.
288
q
A surgical nurse is diagnosed with a skin infection on the right forearm. What is the caregiver's priority action?
a) Request a change of role to home care worker.
b) Return to work after 24 hours of antibiotic therapy.
c) Report the infection to a direct superior.
d) Make sure the infection is covered with a bandage.
A
c) Report the infection to a direct superior.
Explanation:
Infection must be reported immediately due to the aseptic environment of the operating room. The usual barriers may not protect the patient when there is infection. The employee must follow the operating room guidelines regarding infection. Infections may need to be covered with a bandage, but the infection must be reported first. Depending on the policy, the operating room nurse may still be able to work; Therefore, returning to work after 24 hours is not the priority measure. Even if the nurse requests a role change to outpatient nurse, the guidelines for infection in the operating room should be followed; therefore, you must inform yourself first.
289
q
Nurse teaches health promotion class on osteoporosis prevention. The nurse determines that the participants understand the teaching when they realize the amount of sun exposure needed to synthesize enough vitamin D, which is which?
a) 90 to 120 minutes twice a week
b) 5 to 30 minutes twice a week
c) 60 to 90 minutes a week
d) 30 to 60 minutes per week
A
b) 5 to 30 minutes twice a week
Explanation:
It is estimated that most people need 5 to 30 minutes of sun exposure twice a week to synthesize enough vitamin D to prevent osteoporosis.
290
q
The clinical finding of pink, frothy sputum may indicate which of the following causes?
a) pulmonary edema
b) an infection
c) A lung abscess
d) Bronchiektasen
A
a) pulmonary edema
Explanation:
Profuse pink foamy material that usually swells in the throat may indicate pulmonary edema. Foul-smelling sputum and bad breath may indicate a lung abscess, bronchiectasis, or infection caused by a fusospirochetal or other anaerobic organism.
291
q
The nurse attends to a patient who has undergone a laryngectomy. Which of the following interventions will the nurse perform first to meet the patient's nutritional needs?
a) Encourage sweet foods.
b) Season food generously.
c) Start enteral feeding.
d) Offer plenty of light fluids.
A
c) Start enteral feeding.
Explanation:
After the operation, the patient cannot eat or drink for at least 7 days. Alternative sources of nutrition and fluids include intravenous fluids, enteral nutrition by nasogastric or gastrostomy tube, and parenteral nutrition. Once the patient can resume oral feeding, light liquids are offered and sweet foods are avoided as they increase salivation and decrease the patient's appetite. The patient's taste sensations are disturbed for some time after the operation, since the inhaled air goes directly into the trachea, bypassing the nose and final olfactory organs. However, over time, the patient tends to get used to this change and the olfactory sensation adapts; So spices are based on personal preference.
292
q
A patient with an altered state of consciousness is being tube fed. Patients receiving tube feeding should be placed in which of the following positions?
a) Trendelenburg
b) Lateral position
c) Semi-Fowler o superior
d) supine position
A
c) Semi-Fowler o superior
Explanation:
Patients receiving tube feeding are positioned so that the head of the bed is 30 degrees or higher during feeding and for 30 to 45 minutes after tube feeding. Orally fed patients are placed with the head of the bed upright for 30 to 45 minutes after feeding. In patients with a nasogastric or gastrostomy tube, tube placement and residual gastric volume should be checked before each feeding.
293
q
One patient's orders include an autolytic debridement dressing. Which of the following information should the nurse provide when instructing the patient to discharge?
a) Always keep dressing very moist.
b) Do not wear a bandage 6 hours a day.
c) Clean the wound with Dakin's solution.
d) The wound may have a putrid odor.
A
d) The wound may have a putrid odor.
Explanation:
During autolytic debridement therapy, a foul odor is created by the breakdown of cellular debris. This odor does not indicate that the wound is infected. The wound is kept moist at all times during autolytic debridement therapy. The wound should be cleaned with normal saline.
294
q
One of the goals is to maintain an oxygen concentration of 70%. What is the best way for the caregiver to convey this focus?
- a nasal cannula
- An oropharyngeal catheter
- A partial rebreather mask
- A venturi mask
A
A partial rebreather mask
295
q
A patient is operated on the chest. What priority training should the caregiver provide to prevent respiratory complications?
a) Splint the incision site with a pillow during deep breathing and coughing exercises.
b) Pain relievers should be taken before completing the breathing and coughing exercises.
c) Deep breathing and coughing exercises can be used as relaxation techniques.
d) Breathing and coughing exercises should be performed every 8 hours.
A
a) Splint the incision site with a pillow during deep breathing and coughing exercises.
Explanation:
Placing a splint on the incision site helps to relieve pain and support the incision. This increases adherence to deep breathing and coughing exercises that help prevent respiratory complications. Pain relievers should be taken regularly and not just before breathing and coughing exercises. Breathing and coughing exercises should be done at least every 2 hours, more often if possible. While some patients find the exercises relaxing, most patients find the exercises painful to perform.
296
q
Which of the following does the nurse correctly identify as an example of blood pressure and heart rate measurements in a patient with postural hypotension?
a) Supine position: blood pressure 120/70 mm Hg, HR 70 bpm; sitting: BP 100/55 mmHg, HR 90 bpm; Standing: BP 98/52 mm Hg, HR 94 bpm
b) supine position: blood pressure 130/70 mm Hg, HR 80 bpm; sitting: BP 128/70 mmHg, HR 80 bpm; Standing: BP 130/68 mm Hg, HR 82 bpm
c) supine position: blood pressure 140/78 mm Hg, HR 72 bpm; sitting: BP 145/78 mmHg, HR 74 bpm; Standing: BP 144/78 mm Hg, HR 74 bpm
d) supine position: blood pressure 114/82 mm Hg, HR 90 bpm; sitting: BP 110/76 mmHg, HR 95 bpm; Standing: blood pressure 108/74 mm Hg, HR 98 bpm
A
a) Supine position: blood pressure 120/70 mm Hg, HR 70 bpm; sitting: BP 100/55 mmHg, HR 90 bpm; Standing: BP 98/52 mm Hg, HR 94 bpm
Explanation:
Postural (orthostatic) hypotension is a sustained decrease in systolic blood pressure of at least 20 mmHg or diastolic blood pressure of 10 mmHg within 3 minutes of moving from a supine or sitting position to a standing position. The following is an example of blood pressure and heart rate measurements in a patient with postural hypotension: supine: blood pressure 120/70 mm Hg, heart rate 70 bpm; sitting: BP 100/55 mmHg, HR 90 bpm; Standing: BP 98/52 mm Hg, HR 94 bpm. Normal postural responses that occur when a person moves from a supine to a standing position include (1) an increase in heart rate to 5 to 20 bpm above the resting rate; (2) an unchanged systolic pressure or a slight drop of up to 10 mm Hg; and (3) a slight increase in diastolic pressure of 5 mm Hg.
297
q
A client has a partial thickness flat skin pressure ulcer, an abraded area, but no areas of necrosis. What bandage would the nurse use to treat the area?
1. Alginate
2. Gaze dry
3. hydrocolloids
4. No membership is displayed.
A
3. hydrocolloid; Hydrocolloid dressings protect superficial ulcers and provide a proper healing environment.
Alginates (option 1) are used for highly draining wounds; dry gauze (option 2) adheres to the granulation tissue and causes further damage. A bandage is needed to protect the wound and promote healing.
298
q
A nurse caring for a patient finds that the patient's lower extremities are covered with very dry skin and that the stratum corneum of the skin is thickened. Which of the following findings does the nurse notice?
a) Lichenification
b) acantholize
c) Dermatitis
d) Piodermo
A
a) Lichenification
Explanation:
The nurse should record this as lichenification. Dermatitis is inflammation of the skin. Acantholysis is a separation of epidermal cells from each other and pyoderma is a bacterial infection of the skin.
299
q
On skin examination, the nurse finds a purple macular lesion on the patient's right upper extremity. The nurse distinguishes the lesion as petechiae or ecchymosis based on which of the following statements?
a) Erythema
(b) exudate
c) location
d) size
A
d) size
Explanation:
The nurse distinguishes between petechiae and ecchymosis based on the size of the affected area. Other distinguishing factors are shape, color and etiology.
300
q
A nurse cares for a patient receiving radiation therapy for throat cancer. Which of the following complications are late complications of radiation therapy?
a) laryngeal necrosis
b) Disfasia
c) Xerostomía
d) pain
A
a) laryngeal necrosis
Explanation:
Late complications of radiotherapy are laryngeal necrosis, edema, and fibrosis. Pain, xerostomia, and dysphasia are not late complications of radiation therapy.
301
q
During a community health fair, a nurse teaches a group of seniors about health promotion and infection prevention. Which intervention would best promote infection prevention for older adults at risk for pneumococcal infections and influenza?
a) Drink six glasses of water a day
b) Exercising every day
c) Take all prescribed medications
d) receive vaccinations
A
d) receive vaccinations
Explanation:
Identification of patients at risk of developing pneumonia provides an opportunity to practice preventive care. The nurse encourages patients at risk for pneumococcal and influenza infections to get vaccinated against these infections.
302
q
A nurse attends to a patient who has been prescribed diuretics to treat high blood pressure. Does the nurse recognize which of the following medications preserves potassium?
a) Spironolactone (Aldacton)
b) Clortalidona (Hygroton)
c) Clorotiazida (Diuril)
d) Furosemide (Lasix)
A
a) Spironolactone (Aldacton)
Explanation:
Aldactone is known as a potassium-sparing diuretic. Lasix causes the loss of potassium from the body. Diuril causes mild hypokalemia. Hygroton causes mild hypokalemia.
303
q
A nurse conducts discharge training for a patient with diverticulosis. Which of the following should the nurse include in the lesson?
a) Drink 8 to 10 glasses of liquid a day.
b) Avoid raw bran.
c) Use laxatives weekly.
d) Avoid daily exercise.
A
a) Drink 8 to 10 glasses of liquid a day.
Explanation:
The nurse should instruct a patient with diverticulosis to drink at least 8 to 10 large glasses of fluids per day. The patient should include raw bran in the diet as it increases the volume and should avoid the use of laxatives or enemas unless recommended by the doctor. In addition, the patient should exercise regularly if her current lifestyle is rather inactive.
304
q
A patient is scheduled for elective surgery. To prevent the complication of hypotension and cardiovascular collapse, the caregiver should report the use of which medication?
a) Hydrochlorothiazide (HydroDIURIL)
b) Prednison (Deltason)
c) Eritromicina (Ery-Tab)
d) Varfarin (Coumadin)
A
b) Prednison (Deltason)
Explanation:
Patients receiving corticosteroids are at risk of adrenal insufficiency. Corticosteroid-related insufficiency can lead to circulatory collapse and hypotension. Hydrochlorothiazide and erythromycin can cause respiratory complications. Warfarin increases the risk of bleeding.
306
q
The nurse leads a community awareness program on malignant melanoma. If participants identify which of the following risk factors, does the caregiver know they understood the teaching?
a) Mediterranean descent
b) dark skin
c) History of tanning
d) Family history of pancreatic cancer
A
d) Family history of pancreatic cancer
Explanation:
A family history of pancreatic cancer is a risk factor for malignant melanoma. Additional risk factors include lighter skin, freckles, blue eyes, blonde hair, Celtic or Scandinavian descent, a history of sunburn, prior melanoma, a family history of melanoma, and a family or personal history of multiple atypical moles.
307
q
A nursing student employed as a registered nurse
The wizard can perform all functions that:
1. Learned at school
2. What is expected of a nurse at this level
3. They are indicated in the job description
4. It requires technical skills rather than professionals
A
3. They are indicated in the job description
3. You only need to complete the tasks that appear on the
the job description for a nursing assistant or assistant
308
q
All of the following patients are at risk of
Development of severe fluid and electrolyte imbalances.
during and after the operation, except:
1. Patient F, who is 1 year old and has a cleft
palate repair
2. Patient H, 79 years old and history
of congestive heart failure
3. Patient G, 55 years old, with a history
a chronic respiratory disease
4. Patient E, who is 81 years old and suffers
Emergency surgery for intestinal obstruction after
4 days of vomiting and diarrhea
A
Patient G, 55 years old, with a history
a chronic respiratory disease
All other patients are predisposed to a
Imbalance due to existing leaks, fluid overload or
the impossibility of obtaining PO fluids.
309
q
A patient is operated on with a brachial plexus block on the right wrist. The patient expresses concern about knowledge of anesthesia. What is the best caregiver response?
a) "Due to the type of anesthesia used, you may be aware of what is happening around you."
b) "Knowledge of anesthesia does not play any role in the type of surgery you perform."
c) “Advances in the drugs used are reducing the probability of sensitization to anesthesia. What are your biggest concerns?”
d) "The entire surgical team will control the anesthetic knowledge and treat accordingly."
A
a) "Due to the type of anesthesia used, you may be aware of what is happening around you."
Explanation:
Anesthesia awareness is a complication of general anesthesia. The patient undergoes local line block surgery, not general surgery. An honest discussion of conscience is needed so that patients know what to expect in the operating room. Although the entire surgical team should be monitored for anesthetic awareness, this is not relevant to the surgical procedure being performed. To tell the patient that anesthetic awareness is not a problem is to belittle the patient's feelings.
310
q
76-year-old patient who underwent surgery for an abdominal hernia. The PACU nurse judges that the patient is confused and attempts to get out of bed and pull the cords on the heart monitor. What caregiver interventions are appropriate at this time? Select all that apply.
a) Walk with the patient.
b) Administer opioid analgesics as indicated.
c) Reorient the patient.
d) Assess diuresis.
e) Check for hypoxia.
f) Place wives.
A
c) Reorient the patient., d) Control diuresis., e) Control hypoxia.
Explanation:
The nurse should reassure and reorient the patient as necessary. Hypoxia and urinary retention can cause acute confusion in elderly postoperative patients, so it would be appropriate for the caregiver to assess for hypoxia and urine output. Opioid pain relievers can be more confusing; You should consult your doctor about the type and dosage of pain relievers. Patient wandering can be a safety concern, especially if the patient is bleeding or hypoxic. Handcuffs should only be used as a last resort.
311
q
If you are working with an older person, would you consider the older person more likely to experience which of the following changes with age?
1. Thinning of the epidermis
2. Thickening of the epidermis
3. Oiliness of the skin
4. Increased skin elasticity
A
1. Thinning of the epidermis. The epidermis thins with age and skin firmness and elasticity are reduced, dryness and flaking of the skin are increased, and pain perception is reduced due to reduced pressure and light touch.
313
q
Modern care requires the caregiver to have knowledge and skills for a variety of professional roles and responsibilities. Which of the following are examples? (Choose all that apply.)
a supervisor
B. Autonomy and responsibility
C. Patient Advocate
D. Health Promotion
E. Cabildero
A
A B C D
314
q
You notice that your newly assigned client has very shiny skin on their legs, has little to no hair on their legs, and the client reports that their skin is easily damaged. Would you suspect that these signs and symptoms are related to:
1. Excessive use of caustic products to remove hair from the legs.
2. Chronic neurological pathology.
3. Impaired peripheral arterial circulation.
4. Hereditary reduction of sweat glands and hair follicles.
A
3. Impaired peripheral arterial circulation; Shiny skin on the legs, reduced or absent leg hair, and easily damaged skin are often associated with compromised peripheral arterial circulation.
315
q
Which of the following actions should be included in the patient education plan to prevent deep vein thrombosis?
a) Using roof rollers to lift the bottom ends
b) Leg exercises per hour
c) Liquid restriction
d) Prolonged stay at the edge of the bed
A
b) Leg exercises per hour
Explanation:
The benefits of early amb) Leg exercises per hour
Explanation:
The benefits of early ambulation and hourly leg exercises to prevent deep vein thrombosis cannot be underestimated. It is important to avoid using rolled blankets, rolled pillows, or any form of elevation that constricts the vessels below the knees. Prolonged slouching can be dangerous and is not recommended in susceptible patients, as pressure below the knees can reduce circulation. Dehydration increases the risk of thrombosis formation. Bulation and leg exercises every hour to prevent deep vein thrombosis cannot be underestimated. It is important to avoid using rolled blankets, rolled pillows, or any form of elevation that constricts the vessels below the knees. Prolonged slouching can be dangerous and is not recommended in susceptible patients, as pressure below the knees can reduce circulation. Dehydration increases the risk of thrombosis formation.
317
q
A patient asks why there is a drain that draws fluid from the surgical wound. What is the best caregiver response?
a) “It will reduce the number of dressing changes needed.”
b) "Drainage will remove necrotic tissue."
c) "Most surgeons now use wound drains."
d) “Helps prevent infections.”
A
d) “Helps prevent infections.”
Explanation:
Wound drainage helps prevent infection by removing the medium in which bacteria would grow. The goal of wound drainage is not to remove necrotic tissue or reduce the number of dressing changes. To say that most surgeons use wound drains does not adequately answer the patient's question.
318
q
A patient with a recent myocardial infarction was admitted with a new diagnosis of mitral regurgitation. Which of the following assessment data received from the caregiver should be immediately shared with the health professional?
a) The patient has 4+ peripheral edema in both legs.
b) The patient has a crackling sound that is audible in all lungs.
c) The patient presents a loud systolic murmur throughout the precordium.
d) The patient has a palpable tingling sensation in the left anterior chest region.
A
b) The patient has a crackling sound that is audible in all lungs.
Explanation:
Acute mitral regurgitation resulting from myocardial infarction usually manifests as severe congestive heart failure. Dyspnea, fatigue, and weakness are the most common symptoms. Palpitations, shortness of breath on exertion, and cough due to lung congestion also occur. Audible crackles throughout the lungs indicate that the patient has severe left ventricular failure with pulmonary congestion and requires immediate interventions such as diuretics.
319
q
The nurse performs a gastrointestinal examination. If the patient complains of mucus and pus in the stool, the nurse investigates other signs/symptoms for which of the following diseases/conditions?
a) Diseases of the colon
b) Ulcerative colitis
c) Dharma absorption
d) disease of the small intestine
A
b) Ulcerative colitis
Explanation:
The presence of mucus and pus in the stool indicates ulcerative colitis. Watery stools are characteristic of diseases of the small intestine. Loose and semi-solid stools are most commonly associated with colon disease. Bulky, greasy stools indicate intestinal malabsorption.
321
q
A nurse assists a patient with recurrent hemoptysis undergoing bronchoscopy. Which of the following tasks should the nurse perform immediately after the procedure?
a) Ensure that the patient remains moderately sedated to reduce anxiety.
b) Examine the patient for a cough reflex.
c) Inform the patient that bed rest should be maintained for 2 hours.
d) Offer ice chips to the patient.
A
b) Examine the patient for a cough reflex.
Explanation:
After the procedure, the patient cannot take anything by mouth until the cough reflex returns, as preoperative sedation and local anesthesia interfere with the protective gag reflex and swallowing. Once the patient has a cough reflex, the nurse may offer ice chips and possibly fluids. The patient is sedated during the procedure, not after. The patient does not need to rest after the procedure.
322
q
What is the main action of the operating room nurse when the surgeon begins to close the surgical wound?
a) Find a sponge number.
b) Label the tissue sample.
c) Prepare the necessary seams.
d) Deliver the necessary equipment to the surgeon.
A
a) Find a sponge number.
Explanation:
The standards require that the surgical nurse and circulating nurse receive a sponge count at the start of surgery, when the surgical wound is sutured, and when the skin is sutured. Tissue samples should be labeled after collection. The sutures must be ready before the surgeon needs them. While the OR nurse hands over the equipment to the surgeon, the gauze count has higher priority.
323
q
A patient arrives at the hospital with a Tegaderm dressing on her buttocks. Does the nurse document this as what kind of bandage?
a) inactive
b) interactive
c) active
d) Passive
A
d) Passive
Explanation:
There are three categories of dressings: active, passive, and interactive. The nurse refers to Tegaderm as a passive dressing, a dressing that serves as a moist, protective environment for natural healing. Interactive dressings absorb wound exudate and contain hydrocolloids. Active dressings reduce healing time to enhance the healing process and contain biological skin substitutes.
324
q
The nurse triages surgical patients. Which patient would the nurse document as urgent for surgery?
a) A patient with profuse bleeding
b) A patient scheduled for cosmetic surgery
c) A patient who needs cataract surgery
d) A patient with an acute infection of the gallbladder
A
d) A patient with an acute infection of the gallbladder
Explanation:
Acute inflammation of the gallbladder is considered an urgent surgical intervention. Plastic surgery and cataract surgery are not considered urgent surgical procedures. Heavy bleeding can be considered an emergency surgical procedure.
326
q
Health reform will bring changes in the emphasis placed on care. Which of the following models is expected from health reform?
A. Transition from an acute illness to a model of health promotion and disease prevention
B. Transition from the disease prevention model to a health promotion model
C. Transition from an acute illness to a disease management model
D. Transition from a chronic care model to a disease prevention model
A
A. Transition from an acute illness to a model of health promotion and disease prevention
Health care reform also affects the delivery of health care. Greater emphasis is placed on health promotion, disease prevention, and disease management.
327
q
Which of the following terms will the caregiver use to describe a patient's inability to breathe easily when not upright?
a) Dyspnea
b) Hamoptisa
c) orthopnea
d) hypoxemia
A
c) orthopnea
Explanation:
Orthopnea is the term used to describe the patient's inability to breathe easily except in an upright position. Orthopnea can be found in patients with heart disease and occasionally in patients with COPD. Patients with orthopnea are placed in Fowler's high position to facilitate breathing. Dyspnea refers to shortness of breath or shortness of breath. Hemoptysis refers to the expulsion of blood from the airways. Hypoxemia refers to low levels of oxygen in the blood.
329
q
The nurse listens to a crack in a patient with a respiratory illness. With what disturbance do you usually hear crackles?
- Asma
- bronchospasm
- collapsed alveoli
- pulmonary fibrosis
A
collapsed alveoli
330
q
A nurse reviewing a patient's echocardiogram report reads the following statements: “The heart muscle is asymmetrical and shows an increase in overall size and mass, particularly along the septum. The ventricular walls thicken, reducing the size of the ventricular cavities. Several areas of the myocardium show signs of anxiety. Does the nurse know that these manifestations indicate what type of cardiomyopathy?
a) hypertrophy
(b) extended
c) Restrictive
d) Arrhythmogenic right ventricular cardiomyopathy
A
a) hypertrophy
Explanation:
In hypertrophic cardiomyopathy (HCM), the heart muscle increases in size and mass asymmetrically, particularly along the septum. It often affects non-adjacent areas of the ventricle. The increased thickness of the heart muscle reduces the size of the ventricular chambers and makes it take longer for the ventricles to relax after systole. The walls of the coronary arterioles also thicken, reducing the internal diameter of the arterioles. The narrowed arterioles restrict the blood supply to the myocardium, causing numerous small areas of ischemia and necrosis. The necrotic areas of the myocardium eventually fibrotase and heal, further impeding ventricular contraction. Due to the structural changes, HCM is also known as idiopathic hypertrophic subaortic stenosis (IHSS) or asymmetric septal hypertrophy (ASH). CMR is characterized by diastolic dysfunction caused by rigid ventricular walls that compromise ventricular stretch and diastolic filling. Arrhythmogenic right ventricular cardiomyopathy (ARVC) occurs when the right ventricular myocardium progressively infiltrates and is replaced by fibrous scar tissue and adipose tissue.
331
q
Which of the following diagnostic tests is used to look for herpes zoster cells?
a) Skin biopsy
b) Tzanck smear
c) Patch tests
d) skin scrapes
A
b) Tzanck smear
Explanation:
A Tzanck swab is a test used to examine the cells of blistering skin conditions, such as shingles. Biopsies are performed on skin nodules, plaques, blisters, and other lesions to rule out malignancy and provide an accurate diagnosis. Skin scrapings are used to diagnose spores and hyphae. A patch test is used to identify the substances to which the patient has developed an allergy.
332
q
Which of the following is the primary acne-related lesion caused by a blockage of sebum in hair follicles?
a) carbunco
b) Comedón
c) hervir
d) Schlieren
A
b) Comedón
Explanation:
A comedone is the primary acne lesion caused by a blockage of sebum in the hair follicle. A boil is a localized skin infection of a single hair follicle. A carbuncle is a localized skin infection that affects multiple hair follicles. Stretch marks are band-like ridges on the skin that differ in color, texture, indentation, or elevation from the tissue on which they lie.
333
q
The nurse will assess a patient's skin if the patient indicates a mole. Does the nurse alert the doctor about the mole if any of the following situations are observed?
a) Uniform light brown color
b) diameter greater than 6 mm
c) Symmetrical appearance
d) Clear limits
A
b) diameter greater than 6 mm
Explanation:
The nurse alerts the doctor to the mole if features of a melanoma are observed, such as: B. a diameter greater than 6 mm. Other features of melanoma include an asymmetrical appearance; jagged and indistinct edges; and red, white, or blue coloring.
334
q
During the preoperative evaluation, the patient reports being allergic to avocado, banana, and hydrocodone (Vicodin). What is the caregiver's priority action?
a) Notify the nutrition department.
b) Notify the surgical team to remove all latex-based items.
c) Notify the Nursing Manager to proceed with the procedure.
d) Inform the doctor about medication for postoperative pain.
A
b) Notify the surgical team to remove all latex-based items.
Explanation:
Allergies to avocados and bananas may indicate a latex allergy. Although it is necessary to notify the nutrition department and the doctor, this is not an immediate threat as the patient is NPO (nothing by mouth) and post-operative pain medication is prescribed. The nursing supervisor does not need to be informed about the patient's allergies.
335
q
A nurse cares for a patient with end-stage lung disease. The patient wants to go home with oxygen and feel good. The family wants the patient to undergo a new surgical procedure. The nurse explains the risks and benefits of surgery to the family and discusses the patient's wishes with the family. The nurse acts as the patient:
an educator
B.Abogado
C.Supervisor
D. Administrador de caídas
A
B.Abogado
A lawyer protects the patient's human and legal right to make decisions about their care. An attorney can also provide additional information to help a patient decide whether or not to accept treatment, or find an interpreter to help family members communicate their concerns.
336
q
Your client has a pressure ulcer in the sacral area that is believed to be caused by shear forces. The client's family asks you to explain the shear force. Would it be more accurate if I told the family that shear forces involve:
1. Rupture of muscle tissue due to significant downward force.
2. A sudden violation of the integrity of the skin due to pulling against bedding.
3. A superficial skin fold is compressed and the tissue is irritated by the pressure.
4. Superficial surface of the skin relatively immobile in relation to the surface of the bed.
A
3. A superficial skin fold is pinched and the tissue is irritated by the pressure; Shear force is a combination of friction and pressure in which the skin surface does not move relative to the bed surface, whereas the deeper tissues attached to the skeleton tend to move with the body.
337
q
A PACU nurse receives a postoperative patient who has received general anesthesia with a rigid plastic oral airway. The patient has clear lung sounds, steady, effortless breaths of 16, and an oxygen saturation of 98%. The patient responds minimally to painful stimuli. What caregiver measures make the most sense?
a) Continue with frequent assessments of patients.
b) Remove the oral airway.
c) Notify the physician of an impaired neurological condition.
d) Obtain vital signs, including pulse oximetry, every 5 minutes.
A
a) Continue with frequent assessments of patients.
Explanation:
An immediate postoperative patient can be transferred to the PACU with a rigid plastic oral airway. The airway should not be removed until the patient shows signs of gagging or retching. The neurological status is appropriate for a patient who has received general anesthesia. There is no information that requires the patient to measure vital signs more frequently than the standard 15 minutes. The nurse must continue to make frequent assessments of the patient.
338
q
Which of the following causes decreased gas exchange in older adults?
a) The elasticity of the lungs increases with age.
b) The alveolar walls become thicker.
c) The number of alveoli decreases with age.
d) The alveolar walls contain fewer capillaries.
A
d) The alveolar walls contain fewer capillaries.
Explanation:
Although the number of alveoli remains stable with age, alveolar walls are thinner and contain fewer capillaries, reducing gas exchange. The lungs also lose elasticity and become stiffer. Lung elasticity does not increase with age and the number of alveoli does not decrease with age.
339
q
The nurse attends to a patient whose ventilator is disconnected. Which of the following patient findings would require completion of the weaning process?
a) Heart rate below 100 bpm
b) PaO2 superior a 60 mm Hg con FiO2 inferior al 40 %
c) vital capacity of 12 ml/kg
d) Increase in blood pressure of 20 mm Hg from baseline
A
d) Increase in blood pressure of 20 mm Hg from baseline
Explanation:
Working with the rescuer, the caregiver would terminate the weaning process if side effects occurred, including an increased heart rate of 20 beats/min, an increased systolic blood pressure of 20 mm Hg, decreased oxygen saturation at less than 90%. , respiratory rate less than 8 or greater than 20 breaths/min, ventricular arrhythmias, fatigue, panic, cyanosis, irregular or labored breathing, and paradoxical chest movements. Vital capacity 10 to 15 mL/kg, Peak Inspiratory Pressure (MIP) of at least -20 cm H2O, Tidal volume: 7 to -9 mL/kg, Minute ventilation: 6 L/min, and rate of rapid respiration/ superficial less than 100 breaths/min/1; PaO2 greater than 60 mm Hg with FiO2 less than 40% are criteria, if the patient meets them, this indicates that the patient is ready to be weaned from the ventilator. A normal vital capacity is
10 to 15 ml/kg.
340
q
A nurse attends to a patient after heart surgery. On evaluation, the patient appears restless and complains of nausea and weakness. The patient's ECG shows spiked T waves. The nurse checks the patient's serum electrolytes and expects which of the following abnormalities?
a) Hyperkalemia
b) Hipomagnesia
c) Hyponatremia
d) Hypercalcemia
A
a) Hyperkalemia
Explanation:
Hyperkalemia is indicated by mental confusion, restlessness, nausea, weakness, and arrhythmias (high, sharp T waves). Asystole is likely to demonstrate hypercalcemia. Hypomagnesemia would likely present with hypotension, lethargy, and vasodilation. Hyponatremia would probably be indicated by weakness, fatigue, and confusion with no change in T wave formation.
341
q
One patient underwent a laryngectomy. The nurse notes signs of wound damage. Does the caregiver understand that the patient is at high risk of developing which of the following conditions?
a) pneumonia
b) dehydration
c) pulmonary embolism
d) Karotisblutung
A
d) Karotisblutung
Explanation:
The carotid artery is close to the stoma and can rupture due to erosion if the wound does not heal properly. If wound injury occurs, the patient should be carefully monitored and identified as being at high risk for carotid hemorrhage. Pulmonary embolism is associated with immobility. Dehydration can lead to poor wound healing and collapse. Pneumonia is a risk for all postoperative patients.
342
q
The nurse ties up a patient without the
With the permission of the patient and without the consent of the physician.
organize. The nurse may be guilty of:
1. Battery
second strike
3. Negligence
4. Invasion of privacy
A
Drums
accidental touching without consent
344
q
A patient with newly diagnosed emphysema is admitted to the medical-surgical service for evaluation. Which of the following does the nurse recognize as a chest wall deformity occurring as a result of lung hyperinflation in this patient population?
a) Tonnenkiste
b) Taubenbrust
c) puffy chest
d) Cifoescoliosis
A
a) Tonnenkiste
Explanation:
A barrel chest occurs as a result of hyperinflation of the lungs. There is an increase in the anteroposterior diameter of the thorax. It occurs with increasing age and is a typical sign of emphysema and chronic obstructive pulmonary disease (COPD). In a patient with emphysema, the ribs are farther apart and the intercostal spaces tend to increase with expiration. Pectus cupping occurs when there is a depression at the bottom of the sternum, which can cause a bruit. Pigeon breast occurs as a consequence of displacement of the sternum, which causes an increase in the anteroposterior diameter. Kyphoscoliosis is characterized by elevation of the scapula and associated S-shaped spine. This deformity limits lung expansion into the thorax.
345
q
The nursing professor teaches students about the types of lung cancer. What type of lung cancer is characterized by rapid growth and can develop in the periphery?
a) Adenocarcinoma
b) Squamous cell carcinoma
c) Large cell carcinoma
d) Carcinoma broncoalveolar
A
c) Large cell carcinoma
Explanation:
Large cell carcinoma is a fast-growing tumor that tends to arise on the periphery. Bronchoalveolar cell cancer arises from the terminal bronchus and alveoli and usually grows slowly. Adenocarcinoma presents as peripheral masses or nodules and often metastasizes. Squamous cell carcinoma originates from the bronchial epithelium and is more centrally located.
346
q
The nurse cares for a patient diagnosed with pneumonia. Will the nurse assess the patient's tactile emotion by following the steps below?
a) Ask the patient to say "one, two, three" while listening to the lungs
b) Ask the patient to repeat “ninety-nine” as the nurse's hands slide down the patient's chest
c) Place the thumbs along the costal arch of the chest wall and instruct the patient to inhale deeply.
d) Instruct the patient to inhale deeply and hold while striking the diaphragm
A
b) Ask the patient to repeat “ninety-nine” as the nurse's hands slide down the patient's chest
Explanation:
As the nurse verifies tactile emotion, the patient is asked to repeat "ninety-nine" or "one, two, three" or "eeee, eee, eee" as the nurse's hands move across the patient's chest. Vibrations are felt with the palmar surfaces of the fingers and hands or with the ulnar side of the hands extended toward the chest. The hand or hands move across the chest one at a time. The corresponding areas of the thorax are compared. Asking the patient to say "one, two, three" while listening to the lungs is not the correct technique to assess for tactile thrill. The nurse assesses the anterior respiratory excursion by placing the thumbs along the costal arch of the chest wall and directing the patient to take a deep breath. The nurse assesses the diaphragmatic excursion by instructing the patient to take a deep breath and hold it while striking the diaphragm.
347
q
The nurse caring for a patient with repeated episodes of contact dermatitis provides instructions on how to prevent future episodes. Which of the following must the caregiver provide?
a) Avoid cosmetics with fragrance.
b) Use screen softener.
c) Wear gloves during the day.
d) Wash the skin with very hot water.
A
a) Avoid cosmetics with fragrance.
Explanation:
The nurse should teach the patient to avoid cosmetics, soaps, and detergents that contain fragrances. Other prevention methods include avoiding heat and fabric softeners. The use of gloves for cleaning and dishwashing should be limited to no more than 15-20 minutes per day and cotton-lined gloves should be worn.
348
q
What vitamin does the nurse caring for a tuberculosis patient expect to receive along with isoniazid (INH) to prevent INH-associated peripheral neuropathy?
a) vitamin B6
b) Vitamin D
c) Vitamin C
d) vitamin E
A
a) vitamin B6
Explanation:
Vitamin B6 (pyridoxine) is usually given with INH to prevent INH-associated peripheral neuropathy. Vitamins C, D and E are not suitable.
349
q
Which of the following infectious agents is the cause of scabies?
a) bacteria
b) Reactivated virus
c) Kratzmilbe
d) parasitic fungi
A
c) Kratzmilbe
Explanation:
Several skin diseases involve an infectious agent. Scabies is caused by a biting mite. Parasitic fungi cause dermatophytosis of the skin, scalp, and nails. Shingles is caused by a reactivated virus.
350
q
The nurse caring for a patient with diverticulitis prepares to administer medication. What kind of medication does the nurse expect to administer for the patient's diverticulitis?
a) antispasmodic
b) anti-inflammatory
c) Antiangustia
d) Antiemetic
A
a) antispasmodic
Explanation:
The nurse hopes to administer antispasmodic medication to reduce intestinal spasms associated with diverticulitis. The patient may also be prescribed an opioid analgesic to relieve the pain associated with it. There is no evidence that the patient requires anxiolytic, antiemetic, or anti-inflammatory medication at this time.
351
q
Listening to a patient's heart sounds, the nurse notices an S4. Does the caregiver recognize that an S4 is associated with which of the following?
a) Hypertensive heart disease
b) turbulent blood flow
c) heart failure
d) diseased heart valves
A
a) Hypertensive heart disease
Explanation:
Auscultation of the heart requires familiarity with normal and abnormal heart sounds. An additional sound just before S1 is an S4 heart sound or atrial gallop. An S4 tone is often associated with hypertensive heart disease. A sound that follows S1 and S2 is called an S3 heart sound or ventricular gallop. An S3 heart sound is usually indicative of heart failure in an adult. In addition to a heart murmur, auscultation can detect other abnormal sounds, such as murmurs and clicks, caused by turbulent blood flow through diseased heart valves.
352
q
A nurse cares for a patient in the cardiovascular intensive care unit (CVICU) after a coronary artery bypass graft (CABG) procedure. Which of the following clinical findings requires immediate caregiver intervention?
a) Heart rate: 66 bpm
b) Pain score: 5/10.
c) Blood pressure: 110/68 mmHg
d) ZVD measurement value: 1 mmHg
A
d) ZVD measurement value: 1 mmHg
Explanation:
A central venous pressure (CVP) of 1 is low (2 to 6 mmHg) and indicates reduced right ventricular preload, usually caused by hypovolemia. Hypovolemia is the most common cause of decreased cardiac output after cardiac surgery. Substitute fluids, such as colloids, red blood cell concentrates, or crystalloid solutions, may be prescribed. The other findings require follow-up by the nurse; However, treatment of CVP is the caregiver's priority.
353
q
The nurse examines the patient and observes reddish-purple to dark blue spots, plaques, and nodules. The caregiver recognizes that these manifestations are associated with which of the following conditions?
a) syphilis
b) platelet disorders
c) Allergic reactions
d) Kaposi's sarcoma
A
d) Kaposi's sarcoma
Explanation:
Kaposi's sarcoma is a common comorbidity in AIDS patients. For platelet disease, the nurse looks for ecchymosis (bruising) and purpura (bleeding into the skin). Urticaria (hives or hives) is the manifestation of allergic reactions. A painless chancre or ulcerated lesion is a typical finding in patients with syphilis.
354
q
What is the antibiotic of choice to treat acute bacterial rhinosinusitis (RBSA)?
a) Levofloxacin (Levaquin)
b) Keflex (cefalexina)
c) Amoxicillin (Augmentin)
d) Ceftin (Cefuroxima)
A
c) Amoxicillin (Augmentin)
Explanation:
Antibiotics should be given as soon as RSAB is diagnosed. Amoxicillin clavulanate (Augmentin) is the antibiotic of choice. For penicillin allergy, doxycycline (Vibramycin) or respiratory quinolones such as levofloxacin (Levaquin) or moxifloxacin (Avelox) may be used. Other antibiotics previously prescribed to treat ABRS, including cephalosporins such as cephalexin (Keflex) and cefuroxime (Ceftin), are no longer recommended because they are ineffective in treating organisms resistant to the antibiotics that are now more commonly implicated in ABRS.
355
q
The home nurse keeps a record of all medications administered during a surgical procedure. The anesthesiologist administers an opioid pain reliever. What medications would the nurse check when they were administered?
a) Fentanyl (Sublimaze)
b) Metocurina (Metubina)
c) Mivacúrio (Mivacron)
d) Etomidat (Amidat)
A
a) Fentanyl (Sublimaze)
Explanation:
Fentanyl is an opioid pain reliever. Mivacurium and Metocurine are muscle relaxants. Etomidate is an anesthetic
356
q
Which of the following is appropriate
Surgery to maintain the respiratory system.
of the immobilized patient?
1. Rotate the patient every 4 hours.
2. Maintain a maximum fluid intake of 1500ml/
Qualification.
3. During this, continuously apply an abdominal bandage
the patient is in bed.
4. Encourage the patient to take a deep breath and cough
every 1 to 2 hours
A
4. Encourage the patient to take a deep breath and cough
every 1 to 2 hours
This technique produces a powerful and productive
Cough without excessive exhaustion
357
q
What measures should the nurse take to prevent deep vein thrombosis in the care of a postoperative patient?
a) Maintain bed rest.
b) Reinforce the need to do leg exercises awake every hour.
c) Instruct the patient to place the pillow under the knees.
d) Administer heparin in high doses for prophylaxis.
A
b) Reinforce the need to do leg exercises awake every hour.
Explanation:
The nurse should emphasize the need to do leg exercises awake every hour. Maintaining bed rest increases blood pooling in the lower extremities and increases the risk of deep vein thrombosis. For prophylactic treatment, the patient may receive low-dose but not high-dose heparin. The nurse should instruct the patient not to place a pillow under the knees, as the patient may constrict the blood vessels.
358
q
A nurse cares for a patient with a systolic blood pressure of 135 mmHg. This finding would be classified as which of the following?
a) Stage 1 hypertension
b) prehypertension
c) normal
d) Stage 2 hypertension
A
b) prehypertension
Explanation:
A systolic blood pressure of 135 mm Hg is classified as prehypertension. A systolic blood pressure of less than 120 mm Hg is normal. A systolic blood pressure of 140 to 159 mm Hg is stage I hypertension. A systolic blood pressure greater than or equal to 160 is classified as stage II hypertension.
359
q
The nurse knows that there are three types of chronic pharyngitis. Numerous inflamed lymphoid follicles in the pharyngeal wall characterize which of the following?
a) Hoarseness
b) Granular chronicle
c) Atrophy
d) Hypertrophy
A
b) Granular chronicle
Explanation:
Chronic granular pharyngitis is characterized by inflammation of numerous lymphoid follicles in the pharyngeal wall. Hoarseness refers to the inability to use the voice. Atrophic pharyngitis is characterized by a thin, white, shiny, and sometimes wrinkled membrane. Hypertrophic pharyngitis is characterized by a general thickening and congestion of the pharyngeal mucosa.
360
q
A nurse interprets the blood gases of a patient with Acute Respiratory Distress Syndrome (ARDS). What blood gas values indicate respiratory acidosis?
a) pH 7,47, Paco2 28, HCO3 30
b) pH 7,49, Paco2 34, HCO3 25
c) pH 7,25, Paco2 48, HCO3 24
d) pH 7,87, Paco2 38, HCO3 28
A
c) pH 7,25, Paco2 48, HCO3 24
Explanation:
pH 7.25, Paco2 48, HCO3 24 = respiratory acidosis pH 7.87, Paco2 38, HCO3 28 = metabolic alkalosis pH 7.47, Paco2 28, HCO3 30 = respiratory alkalosis pH 7.49, Paco2 34, HCO3 25 = respiratory alkalosis
361
q
🇧🇷 In a situation where there are not enough staff
To implement competent care, a nurse must:
1. Organize a strike
2. Reject the order
3. Inform patients about the situation
4. Accept assignment but file a protest
write to the administration
A
4. Accept assignment but file a protest
write to the administration
Need to follow the policies of the institution and
Procedures to deal with these situations and
Use the chain of command
362
q
A nurse caring for a patient with cardiomyopathy diagnoses anxiety related to the fear of death. Which of the following patient behaviors would indicate to the nurse that the patient's anxiety level has decreased?
a) The patient looks forward to family visits.
b) The patient is lying in bed and watching television.
c) The patient responds without problems to the questions about the physical state.
d) The patient can freely discuss the prognosis.
A
d) The patient can freely discuss the prognosis.
Explanation:
When anxiety subsides, patients are free to discuss the prognosis, express fears and concerns, and participate in support groups.
363
q
The nurse attends to a patient undergoing a thoracentesis. In which of the following positions does the nurse place the patient in preparation for the procedure?
a) Vulnerable
b) supine position
c) Sit on the edge of the bed
(d) lateral decubitus
A
c) Sit on the edge of the bed
Explanation:
When possible, it is best to place the patient standing or at the edge of the bed, with feet supported, arms, and head on a padded nightstand. Other positions in which the patient may be placed include lying in a chair with the arms and head resting on the back of the chair, or lying on the unaffected side with the head of the bed elevated 30 to 45 degrees if the Patient cannot assume a comfortable sitting position.
364
q
A PACU nurse prepares to discharge a patient after ankle surgery. The patient continues to look at the ceiling while receiving discharge instructions. What actions are appropriate for the caregiver?
a) Review the instructions with the patient and accompanying adult.
b) Ask the patient: "Did you understand?"
c) Deliver the written instructions to the patient's 16-year-old son.
d) Repeat the instructions continuously until the patient repeats them.
A
a) Review the instructions with the patient and accompanying adult.
Explanation:
The effects of anesthesia can affect the patient's memory or concentration. It is important that the discharge instructions are discussed with the patient and an accompanying adult. Giving instructions to a 16 year old is not appropriate. Repeating the instruction until the patient repeats it does not guarantee that the patient will remember it, as anesthesia can affect memory. Asking if the patient understood the instructions only elicits a yes or no response, but does not indicate whether the patient understood the instructions.
366
q
On the second postoperative day, a patient requires treatment of a surgical wound with healing by secondary intention. What kind of dressing changes should the caregiver expect?
a) Cleaning the wound with sterile saline solution and application of cyanoacrylate tissue glue
b) Unpack the wound bed with a saline soaked sterile dressing and cover with a dry dressing.
c) Clean the wound with soap and water and allow it to air out
d) Cover the very close edges of the wound with a dry dressing.
A
b) Unpack the wound bed with a saline soaked sterile dressing and cover with a dry dressing.
Explanation:
Postoperative surgical wounds, which may heal by secondary scarring, are usually wrapped with a sterile saline dressing and covered with a dry dressing. The edges of a wound healed by secondary intention do not approximate. The wound can be cleaned with sterile saline, but the nurse would not apply cyanoacrylate tissue glue. The wound should not be left open to air, as this can expose it to microorganisms and dry out the wound bed, making healing more difficult.
367
q
A patient in the emergency room developed wheezing and shortness of breath. The nurse administers the prescribed nebulizer treatment now and in 4 hours. What standard of practice is performed?
A. Planning
B. Evaluation
C. Classification
D. Implementation
A
D. Implementation
Implementation includes care coordination and the prescribed care plan.
369
q
During the skin exam, the nurse finds a skin infection located in a single hair follicle. Does the caregiver document the presence of which of the following?
a) Queilita
b) Carbunco
c) comedones
d) hereafter
A
d) hereafter
Explanation:
Boils are localized skin infections of a single hair follicle. They can appear anywhere on the body, but are more common in areas prone to irritation, pressure rubbing, and excessive sweating, such as the neck, armpits, or buttocks. A carbuncle is a localized skin infection that affects multiple hair follicles. Cheilitis refers to dry cracks at the corners of the mouth. Comedones are the primary acne lesions caused by a blockage of sebum in the hair follicle.
370
q
A nurse attends to a patient with a oozing skin lesion. The surrounding skin is very inflamed. What type of bandage should the caregiver apply?
a) protector
b) Plosive
c) interactive
d) Passive
A
c) interactive
Explanation:
The caregiver should apply an interactive bandage to absorb exudate while providing a moist environment for wound healing. Protective and passive dressings do not absorb exudate; They only have a protective function. Occlusive dressings are used to cover topical medications.
372
q
The nurse instructed a patient to breathe through pursed lips. The nurse recognizes that the purpose of this type of breathing is to perform which of the following tasks?
a) Promote the patient's ability to absorb oxygen.
b) Promote more efficient and controlled ventilation and reduce the work of breathing
c) promote the strengthening of the patient's diaphragm
d) Improve oxygen transport, induce a slow, deep breathing pattern, and help the patient control breathing.
A
d) Improve oxygen transport, induce a slow, deep breathing pattern, and help the patient control breathing.
Explanation:
Lip breathing, which improves oxygen transport, helps induce a slow, deep breathing pattern and helps the patient control breathing even during times of stress. This type of breathing helps prevent airway collapse due to loss of lung elasticity in emphysema.
373
q
A patient diagnosed with myocardial infarction (MI) started an active rehabilitation program. Which of the following does the nurse use as the overall goal of rehabilitation for a patient with myocardial infarction?
a) Limit the impact and progression of atherosclerosis
b) Prevention of another cardiac event
c) Return of the patient to work and lifestyle prior to the disease
d) Improve the quality of life
A
d) Improve the quality of life
Explanation:
In general, cardiac rehabilitation is a complete program dedicated to prolonging and improving the quality of life. An immediate goal of rehabilitation for the patient with myocardial infarction is to limit the impact and progression of atherosclerosis. An immediate goal of the rehabilitation of the patient with myocardial infarction is the return of the patient to work and to the lifestyle prior to the illness. An immediate goal of rehabilitation for the patient with myocardial infarction is to prevent another cardiac event.
375
q
The nurse treats a postoperative patient with an indwelling urinary catheter. Urine output per hour at 9 am is 80 ml. The nurse estimates that the urine output per hour at 10 am is 20 ml. What is the most important action of the nurse?
a) Document the results.
b) Flush the catheter with sterile saline.
c) Re-evaluate leaving at 11:00 a.m.
d) Notify the doctor.
Show response
A
d) Notify the doctor.
Explanation:
If the patient has an indwelling catheter, deliveries per hour are monitored and rates less than 30 ml/h are reported. Any diuresis less than 30 ml/h should be reported immediately to the physician. Urine output will be reassessed at 11:00 a.m. m., but waiting for the doctor's notification can harm the patient. The results must be documented, but this is not the highest priority. A urinary catheter may need irrigation, but a postoperative patient with low urine output has a complication that should be reported immediately.
376
q
If the patient has subcutaneous emphysema along the chest bandage or suture line 2 hours after chest surgery, which of the following is the best action for the nurse to take?
a) Communicate the finding to the doctor immediately.
b) Apply a compression bandage to the area.
c) Write the observation.
d) Measure the patient's pulse oximetry.
A
b) Apply a compression bandage to the area.
Explanation:
Subcutaneous emphysema is a typical postoperative finding in patients after breast surgery. During surgery, air from the pleural cavity is expelled through the opening in the tissue created by the surgical procedure. Subcutaneous emphysema is spontaneously absorbed by the body after treating or stopping the underlying leak.
377
q
A patient was transported to the operating room for emergency surgery. Which statement from the nurse best supports the need for emergency surgery?
a) "The patient was unconscious, with abdominal distension and unstable vital signs after a car accident."
b) "The patient experienced severe pain and a facial laceration with minimal bleeding after being attacked by a dog 1 hour ago."
c) "The patient had epigastric abdominal pain, elevated leukocytosis, and vomiting for 1 day."
d) "The patient presented tachycardia, progressive weight loss and insomnia secondary to hyperthyroidism."
A
a) "The patient was unconscious, with abdominal distension and unstable vital signs after a car accident."
Explanation:
Emergency surgery means that the patient needs immediate attention and the condition can be fatal. The patient with unstable vital signs and a distended abdomen after a car accident requires immediate attention. The patient with abdominal pain on the left side may not need surgery. Upper abdominal pain with vomiting for 1 day is generally not an indication for emergency surgery. Cuts on the face require sutures, not emergency surgery. A thyroidectomy to treat hyperthyroidism is necessary surgery, not emergency surgery.
377
q
What is the priority action when the nurse performs a second review of the surgical procedure and surgical site?
a) Discuss complications and allergies with the anesthesiologist.
b) Call the attention of all members of the surgical team.
c) Ask the surgeon if the marked surgical site is correct.
d) Discuss the surgical procedure and the surgical site with the patient.
A
b) Call the attention of all members of the surgical team.
Explanation:
The second review of the surgical procedure and site should include all members of the surgical team. This review should be done simultaneously with all team members involved. Surgical site marking is confirmed by all members of the surgical team, not just the surgeon or patient. Expected complications, allergies, and problems are also discussed with the entire surgical team.
378
q
A patient comes to the clinic and asks the nurse why the skin on his forehead, palms and soles has a yellow-orange tint. There is no yellowing of the sclera or mucous membranes. What should the nurse ask the patient?
- “He has consumed large amounts of alcohol”
- "Have you been diagnosed with Addison's disease?"
- "Have you been in the sun a long time?"
- "Did you eat large amounts of foods rich in carotene?"
A
"Did you eat large amounts of foods rich in carotene?"
379
q
Which of the following is the area of the heart that is located in the third intercostal space (IC) to the left of the sternum?
a) upper abdominal region
b) Aortenbereich
c) lung area
d) Erb's point
A
d) Erb's point
Explanation:
The Erb point is located in the third IC space to the left of the sternum. The aortic area is in the second IC site to the right of the sternum. The lung area is located in the second IC space to the left of the sternum. The epigastric area is below the xiphoid process.
380
q
A nurse attends to a patient with a wound in the late regeneration phase of tissue repair. The wound can be protected by applying:
1. Transparent film
2. Hydrogel band
3. Collagenase dressing
4. Wet-dry bandage
A
1. Transparent film; Wounds in the regenerative phase of healing must be protected as new tissue grows. Answers 2, 3 and 4 are bandages to remove non-viable tissue.
381
q
For which of the following reasons is it important for the caregiver to encourage the patient diagnosed with hypertension to rise slowly from a sitting or lying position?
a) Gradual position changes give the heart time to increase the rate of contraction in order to replenish oxygen to the brain.
b) Gradual changes in position will help reduce the work of the heart, restore oxygen to the brain.
c) Gradual position changes help lower blood pressure to reoxygenate the brain.
d) Gradual position changes give the heart time to reduce its rate of contraction in order to replenish oxygen to the brain.
A
a) Gradual position changes give the heart time to increase the rate of contraction in order to replenish oxygen to the brain.
Explanation:
It is important for the caregiver to encourage the patient to rise slowly from a sitting or lying position, as gradual changes in position give the heart time to increase its rate of contraction to reoxygenate the brain, not blood pressure or rate. heart rate from blood pressure.
383
q
Your client has a Braden scale score of 17. What is the most appropriate nursing intervention?
1. Re-evaluate the client within 24 hours. The score is in the normal range.
2. Implement a restructuring plan; the client is at increased risk of skin damage.
3. Affix a clear wound barrier to key pressure points; The client is at moderate risk of skin damage.
4. Ask for a special low air loss bed; the client is exposed to a very high risk of skin damage.
A
2. Implement a restructuring plan; the client is at increased risk of skin damage; A score in the range of 15 to 18 is considered vulnerable and a recovery plan is appropriate.
Option 1 requires a score greater than 18 (continuous normal score shown). Option 3, medium risk, for which a transparent barrier would be appropriate, applies to individuals with scores of 13-14. Option 4, very high risk, is assigned to individuals with scores of 9 or less.
384
q
A health issue often becomes an ethical issue
dilemma because:
1. Decisions must be made based on value systems
2. The options seem ambiguous
right or wrong
3. Decisions must be made quickly, often under
stressful conditions
4. The legal rights of the patient coexist with health
professional duties
A
2. The decisions involved do not appear to be clearly right or wrong
Ethical problems arise from controversies and
Conflict
385
q
A nurse examines a patient for obstructive sleep apnea (OSA). Which of the following are signs and symptoms of OSA? Select all that apply.
a) Polycythemia
b) insomnia
c) Ronco alto
d) Pulmonary hypotonia
e) Nocturnal headache
A
a) polycythemia, b) insomnia, c) loud snoring
Explanation:
Signs and symptoms include excessive daytime sleepiness, frequent nocturnal awakenings, insomnia, loud snoring, morning headaches, mental deterioration, personality changes, irritability, impotence, systemic hypertension, arrhythmias, pulmonary and pulmonary hypertension, polycythemia, and enuresis.
386
q
A nurse working in a doctor's office teaches parents of a child diagnosed with tinea captis (tinea captis). How often should the nurse instruct the parents to wash the child's hair with Nizoral or a selenium sulfide shampoo?
a) twice a day
b) weekly
c) once
d) daily
A
d) daily
Explanation:
The nurse should instruct the parents to wash the child's hair with Nizoral or selenium sulfide shampoo two to three times a day.
388
q
An anxious preoperative surgical patient is encouraged to focus on a pleasurable experience or restorative scene. What cognitive coping strategy would the nurse document?
a) Photos
b) distraction
c) Progressive muscle relaxation
d) Optimistic self-recitation
A
a) Photos
Explanation:
Imaging has been shown to be effective for anxiety in surgical patients. Optimistic self-recitation is practiced when the patient is encouraged to recite optimistic thoughts such as "I know everything is going to be okay." Distraction is used when the patient is encouraged to think of a funny story or recite a favorite poem. Progressive muscle relaxation involves contracting and relaxing groups of muscles and is a physical coping strategy as opposed to a cognitive strategy.
389
q
A group that lobbies at the state and federal level.
promote the role of nursing personnel, economic interests,
and health is:
1. State pavilions
2. American Nurses Association
3. American Hospital Association
4. National Association of Nursing Students
A
2. American Nurses Association
ANA's goal is to improve
professional development and general well-being
nurses
390
q
Neck stiffness or inability to bend the neck is attributed to which of the following causes?
a) Dysphagia
b) Hoarseness
c) Stiff neck
d) Xerostomia
A
c) Stiff neck
Explanation:
Stiff neck is stiffness in the neck or the inability to bend the neck. Hoarseness is a deterioration in the ability to use the voice due to stress or injury to the larynx. Xerostomia is dry mouth for several reasons. Dysphagia is difficulty swallowing.
391
q
You are attending a conference to coordinate the clinical care of a terminal cancer patient. Discuss with your colleagues the use of the Code of Ethics for Professional Nurses to guide care decisions. An indifferent colleague asks for this code. Which of the following statements best describes this code?
A. Improves self-sufficiency
B. Protects patient confidentiality
C. Ensure equal care for all patients
D. Defines the principles of right and wrong in patient care
A
D. Defines the principles of right and wrong in patient care
When providing care, it is important to perform a specific service in accordance with standards of practice and follow a code of ethics. The Code of Ethics consists of the philosophical ideals of right and wrong that define the principles you apply in caring for your patients. The Code serves as a guide for compliance with the duties of care to guarantee quality care and the ethical obligations of the profession.
392
q
A patient who has recently had a myocardial infarction develops pericarditis and complains of grade 6 chest pain (on a scale of 0 to 10) with deep breathing. Which of these Pro-Re-Nata (PRN) prescription drugs are best for caregiver administration?
a) Acetaminophen (Tylenol) 650 mg orally (orally) alle 4 Stunden
b) Ibuprofen (Motrin) 800 mg po every 8 hours
c) Morphine sulfate 6 mg IVP every 2-4 hours
d) Fentanyl 2 mg intravenous pyelogram (IVP) every 2-4 hours
A
b) Ibuprofen (Motrin) 800 mg po every 8 hours
Explanation:
The pain associated with pericarditis is caused by inflammation, which is why non-steroidal anti-inflammatory drugs (NSAIDs), such as ibuprofen, are most effective. Opioid pain relievers are generally not used for pain associated with pericarditis.
393
q
The volume of air inspired and expired with each breath is called which of the following?
a) tidal volume
b) vital capacity
c) Resting volume
d) vital capacity
A
a) tidal volume
Explanation:
Tidal volume is the volume of air that is inhaled and exhaled with each breath. Residual volume is the volume of air remaining in the lungs after maximal expiration. The vital capacity is the maximum volume of air exhaled from the point of maximum inspiration. The expiratory reserve volume is the maximum volume of air that can be exhaled after a normal inspiration.
394
q
An 87-year-old man is hospitalized with phlegmon on his left arm. He walks with a walker and takes a diuretic to control the symptoms of fluid retention. Which procedure is most important to protect you from injury?
1. Leave the bathroom light on.
2. Discontinue the client's diuretic medication.
3 Offer a nightstand.
4. Hold the rails up.
A
3. Offer a nightstand.
Rationale: Placing the nightstand next to the bed helps reduce the number of steps you have to take. This will help protect you from injury from falls. Option 1: Leaving the light on would help the client find the bathroom, but would not reduce the risk of falling when running to the bathroom. Option 2: The nurse cannot withhold a patient's medication without consulting the GP. Option 4 – If the customer is asked to stand up with assistance and the railings are up, they are at risk of falling and falling a distance.
395
q
A nurse prepares to connect EKG electrodes to a male patient who needs continuous cardiac monitoring. Which of the following should the caregiver do to improve adherence to the skin and conduction of electrical current from the heart?
a) Trim the patient's chest hair before placing the electrodes.
b) Once the electrodes are in place, change them every 72 hours.
c) Apply talcum powder to the patient's chest before placing the electrodes.
d) Before applying the electrodes, clean the patient's chest with alcohol.
A
a) Trim the patient's chest hair before placing the electrodes.
Explanation:
The nurse should follow these steps when applying the cardiac electrodes: trim (do not shave) the hair around the electrode site, if necessary; If the patient is diaphoretic (sweating), apply a small amount of benzoin to the skin, avoiding the area below the center of the electrode. Clean the surface of the skin of dead cells with soap and water and dry thoroughly (or as recommended by the manufacturer). Change electrodes every 24 to 48 hours (or as recommended by the manufacturer); Examine the skin for irritation and place the electrodes in different locations.
396
q
When you receive a report at the beginning of your shift, you learn that your designated client has a surgical incision that is healing by first intention. Did you know that your client's incision:
1. Very rough, with little or no drainage.
2. Takes a little longer than normal to heal.
3. It will leave more scars than most incisions.
4. Drain serosanguineous secretions.
A
1. Very rough, with little or no drainage; Primary intention means that the wound edges are closely approximated with little or no tissue loss and minimal granulation and scar tissue formation.
398
q
Which of the following is the correct cuff pressure for the endotracheal tube?
a) 25 mmHg
b) 17 mmHg
c) 13 mmHg
d) 21 mmHg
A
b) 17 mmHg
Explanation:
Cuff pressure should be checked every 6 to 8 hours with an aneroid manometer calibrated to maintain cuff pressure between 15 and 20 mm Hg. Other values are not within the normal range for adequate cuff pressure.
399
q
The nurse takes the medical history of the patient with a prosthetic heart valve and new symptoms of infective endocarditis. Which caregiver question is the most appropriate?
a) Do you have a family history of endocarditis?
b) Do you live with pets in your home?
c) Have you recently taken a vacation outside of the United States?
d) Have you been to the dentist recently?
A
d) Have you been to the dentist recently?
Explanation:
Invasive procedures, particularly those involving mucosal surfaces (eg, those involving manipulation of gingival tissue or periapical dental regions), can cause bacteremia that rarely lasts longer than 15 minutes. However, if a patient has anatomic heart defects or implanted cardiac devices (eg, prosthetic valve, pacemaker, implantable cardioverter-defibrillator [ICD]), bacteremia can cause bacterial endocarditis.
400
q
A patient has developed a boil on his face and the nurse watches as the patient bursts the boil. What does the caregiver understand by a potentially serious complication of this manipulation?
- scars
- brain abscess
- Erythema
- cellulitis
A
brain abscess
401
q
The nurse treated a postoperative patient for nausea. What medications would the nurse document as administered?
a) Ondansetron (Zofran)
b) Prednison (Deltason)
c) Propofol (Diprivan)
d) Varfarin (Coumadin)
A
a) Ondansetron (Zofran)
Explanation:
Odansetron (Zofran) is an antiemetic and one of the most widely prescribed medications for nausea and vomiting. Warfarin (Coumadin) is a blood thinner. Prednisone (Deltason) is a corticosteroid. Propofol (Diprivan) is an anesthetic.
402
q
The nurse attends to a critical patient in the intensive care unit). The nurse records the patient's respiratory rate as bradypnea. Does the caregiver recognize that bradypnea is associated with which of the following conditions?
a) metabolic acidosis
b) pneumonia
c) Increased intracranial pressure
d) pulmonary edema
A
c) Increased intracranial pressure
Explanation:
Bradypnea is associated with increased intracranial pressure, brain injury, and drug overdose. Breathing is slower than normal (
403
q
The nurse is developing a treatment plan for a patient with toxic epidermal necrolysis (TEN) or Stevens-Johnson syndrome. Which of the following must the caregiver provide?
a) Use of friction when repositioning the patient
b) Continuous flow of hot air
c) Frequent inspection of the oral cavity
d) Limitation of liquids
A
c) Frequent inspection of the oral cavity
Explanation:
The nurse must frequently inspect the oral cavity of a patient with TEN or Stevens-Johnson syndrome. In addition, care must be taken to reduce friction and shear when rotating or repositioning the patient. Fluid intake should not be restricted as these patients are prone to dehydration. A continuous stream of hot air on exposed skin can worsen dehydration.
404
q
Which of the following are used to irrigate wound care? Select all that apply.
1. Clean gloves
2. Sterile gloves
3. Cold rinse solution
4. 60ml syringe
A
1, 2 and 4; To irrigate a wound, the nurse uses clean gloves to remove the old dressing and holds the basin where the irrigation fluid is collected, and sterile gloves to apply the new dressing. A 60 mL syringe is the correct size to hold the volume of the irrigating solution and deliver safe irrigating pressure. Wash fluid should be at room or body temperature, never refrigerated. 🇧🇷
405
q
A nurse visits a client who sprained her ankle in the morning. The client has an ice pack on his ankle. Which of the client's chronic medical conditions is an argument against the use of ice?
1. Gastritis
2.Diabetes
3. Glaucoma
4. Osteoporosis
A
2.Diabetes; Diabetes contradicts the use of ice. Clients with neurological or circulatory problems are at risk of injury when using ice.
406
q
The nurse shows a patient with a stoma how to change the pouching system. Which of the following should the nurse include when instructing a patient without peristomal skin irritation?
a) Dry the skin well after washing.
b) Sprinkle with nystatin powder
c) Apply Kenalog spray
d) Apply barrier powder
A
a) Dry the skin well after washing.
Explanation:
Without peristomal skin irritation, the caregiver should instruct the patient to dry the skin thoroughly after washing. Barrier powder, kenalog spray, and nystatin powder are used when peristomal skin irritation and/or fungal infections are present.
407
q
How do classic impetigo lesions manifest next?
a) Comedones in the facial area
b) Abscess of skin and subcutaneous tissue
c) Lesions with honey crusts on an erythematous base
d) patches of grouped vesicles on red, swollen skin
A
c) Lesions with honey crusts on an erythematous base
Explanation:
Classic impetigo lesions are honey-crusted lesions on an erythematous base. Facial comedones are representative of acne. A carbuncle is an abscess of the skin and subcutaneous tissue. Shingles appears as patches of clustered blisters on red, swollen skin.